Vous êtes sur la page 1sur 226

Parte I: Ecuaciones Diferenciales de Primer

Orden
1

Ejemplos de Problemas que involucran Ecuaciones Diferenciales.

Ejemplo 1 Considere un objeto de masa m cayendo libremente. Cmo determinar la posicin del objeto en
cada instante t?
En primer lugar se necesita un sistema de coordenadas apropiado. Considere una recta vertical con coordenadas de manera tal que el sentido positivo queda hacia arriba. Como la posicin del objeto cambia en el tiempo,
resulta claro que para describirla en relacin al sistema de coordenadas se requiere un listado de nmeros que
indique la posicin en cada instante. Esto se expresa diciendo que la posicin est determinada por la funcin
y (t), y el problema consiste en determinar la funcin y (t). Ahora bien, como el objeto cae libremente se puede
argumentar que la nica fuerza que acta sobre l es su propio peso P , que se relaciona, segn Newton, con la
aceleracin de gravedad por la ecuacin
P = mg,

donde g > 0, es la aceleracin de gravedad. Por otra parte, la aceleracin a (t) experimentada por el objeto en
el instante t, se expresa desde el punto de vista de y (t), o mejor de clculo diferencial, por
a (t) =

d2 y (t)
= y 00 (t) ,
dt2

y nuevamente Newton afirma que


P =m

d2 y (t)
= my 00 (t) .
dt2

Entonces, se puede concluir que


y 00 (t) = g.

De esta manera se ha determinado una relacin que debe ser satisfecha por la funcin y (t) que representa
la situacin de inters. Esta relacin es una ecuacin diferencial; ecuacin por cuanto involucra la magnitud
desconocida y (t), que se busca determinar, y diferencial porque la incgnita aparece bajo el signo de la derivada.
Se puede observar que cualquier funcin que satisfaga esta relacin puede representar la posicin del objeto
en el instante t. Esto tiene sentido: note que se puede dejar caer un objeto de muy diferentes formas, por
ejemplo se puede lanzar hacia abajo o hacia arriba, an cuando pareciera que esta ltima forma no es la manera
ms natural de "caer", luego, cualquiera sea la forma en que el fenmeno ocurra, se espera poder describir el
movimiento con una funcin que satisfaga la ecuacin.
En efecto, integrando un par de veces ambos miembros de la ecuacin diferencial se obtiene:
1
y (t) = gt2 + at + b
2
De qu manera se puede seleccionar la solucin que representa la situacin que se est estudiando?.
Para responder a esta pregunta es necesaria informacin adicional que de alguna forma caracterice la situacin
que est bajo estudio, por ejemplo, conocer la posicin en que se encuentra en algn instante t y, naturalmentem
la velocidad en ese mismo instante. As, esta situacin particular queda descrita por la ecuacin
y 00 (t) = g
1

junto con las llamadas condiciones inciales


y 0 (t0 ) = v0
y (t0 ) = c0 .
Esto lleva a la solucin particular

1 2
1 2
y (t) = gt + (v0 + gt0 ) t + c0 v0 t0 + gt0 .
2
2
Ejemplo 2 La velocidad con que se desintegra una sustancia radiactiva es, en cada instante, proporcional a la
cantidad de sustancia que permanece no desintegrada. Cmo determinar la cantidad de sustancia radiactiva
en el instante t?.
Anotemos A (t) la cantidad de sustancia radiactiva en el instante t. La desintagracin en un intervalo de
tiempo [t, t1 ] est dada por la variacin A (t1 ) A (t) en la cantidad de sustancia radiactiva y la rapidez con
que se produce la desintegracin en este intervalo es
A (t1 ) A (t)
t1 t
de manera que la rapidez con que cambia la cantidad A (t), en el instante t no es otra cosa que la derivada de A
A0 (t) = lim

tt1

A (t1 ) A (t)
t1 t

y entonces la cantidad A (t) necesariamente satisface la ecuacin diferencial


A0 (t) = kA (t)
donde k < 0 es la constante de proporcionalidad (Por qu k < 0?).
Ejemplo 3 Supngase que un cable cuelga suspendido de sus extremos, sometido a su propio peso. Cmo
determinar la forma que asume el cable colgante?
Se situa el sistema de coordenadas de manera que el punto ms bajo de la curva, P1 , est sobre el origen
del sistema de coordenadas. Considere P2 un punto cualquiera sobre la curva. Sobre el arco P[
1 P2 actan tres
fuerzas, a saber:
1.- El peso del segmento de cuerda P[
1 P2 ,
2.- la tensin T1 en el extremo P1 , y
3.- la tensin T2 en el extremo P2 .

Figura 1: Cable colgante


Ahora bien, si w representa el peso del cable por unidad lineal y s (x) representa la longitud de arco de cable
entre P1 y P2 , entonces
P eso de P[
1 P2 = ws (x)
y por otra parte, como el sistema est en equilibrio, entonces las tensiones en los extremos P1 y P2 del arco en
consideracin satisfacen

|T1 | + |T2 | cos ( (x)) = 0, 0 (x)


2
ws (x) + |T2 | sen ( (x)) = 0
y podemos escribir
|T1 | = |T2 | cos ( (x)) , 0 (x)

ws (x) = |T2 | sen ( (x))


dividiendo miembro a miembro
tg ( (x)) =
Por otra parte, del curso de clculo
s (x) =

1+

ws (x)
|T1 |

dy (t)
dt

(1.1)
2

dt,

reemplazando en (1.1) y derivando, para eliminar la integral se obtiene finalmente


s
2
w
dy
d2 y
=
1+
2
dx
|T1 |
dx
Por lo tanto, si se necesita determinar la funcin cuya grfica representa la forma que asume el cable colgante,
se debe encontrar una aplicacin y (x) que satisfaga esta ltima ecuacin.
3

Ejemplo 4 Una masa m de peso W se suspende del extremo de una varilla de longitud constante l, la que
puede rotar libremente por el extremo opuesto. Cmo determinar el movimiento de la masa m?.
Considere P , el punto ms bajo del recorrido del pndulo y sea s (t) la longitud de arco entre el punto P
y la posicin de la masa en el extremo del pndulo en el instante t. Si (t) es el ngulo determinado por el
pndulo respecto a la vertical, se puede afirmar que:
(t) =

s (t)
,
l

o
s (t) = l (t) .

Segn la segunda ley de Newton, la componente del peso m en la direccin del movimiento F (t) satisface

ml00 (t) = ms00 (t) = F (t) = W sen ( (t)) = mgsen ( (t)) ,


y entonces
l00 (t) = gsen ( (t)) ,
o bien

g
00 (t) + sen ( (t)) = 0
(1.2)
l
Consecuentemente el movimiento del extremo del pndulo estar descrito por una funcin que satisfaga la
ecuacin diferencial (1.2).
Ejemplo 5 Considere una masa m que pende del extremo de un resorte cuyo segundo extremo est fijo. Cmo
determinar el movimiento de la masa m?.
Para responder a esta pregunta se debe considerar la ley de Hook: La fuerza ejercida por un resorte al
ser deformado longitudinalmente es proporcional a la diferencia de longitud experimentada por el resorte. La
formulacin respectiva de la ley es:
FR = k (lf li )
donde k > 0, es la constante de proporcionalidad o constante del resorte, lf es la longitud del resorte deformado
y li es la longitud natural del resorte.
Ahora bien, si por el extremo libre del resorte se suspende una masa m, y se deja al sistema oscilar libremente,
luego de un tiempo el sistema "masa-resorte" alcanzar un estado de equilibrio. Sea le la longitud del resorte
en estado de equilibrio y s = le li . Aplicando la ley de Hook se puede afirmar que
ks + mg = 0

(1.3)

Ahora bien, si en el instante t la masa se encuentra en la posicin y (t), entonces la fuerza resultante aplicada
a la masa m en ese instante ser

F (t) = k (s + y (t)) + mg,


y segn la ley de Newton

F (t) = my 00 (t)

de manera que
ks ky (t) + mg = my 00 (t)
4

y empleando (1.3), obtenemos


k
y (t) = 0
(1.4)
m
Se puede concluir que para determinar el movimiento de la masa m en el extremo del resorte es suficiente
encontrar una funcin y (t) que satisfaga la ecuacin diferencial (1.4). Observe que el razonamiento anterior
tiene un pequeo "descuido". En efecto, se observa que para determinar le se admite un hecho intuitivamente
discutible: despus de un tiempo el sistema alcanza un estado de equilibrio, esta est muy bien, pero rpidamente "ha sido olvidado". Se puede argir como defensa que, si bien es cierto se ha olvidado este hecho
en el razonamiento posterior, an as el sistema "masa-resorte" puede estar razonablemente bien representado
por aquellas funciones y (t) que satisfacen la ecuacin (1.4), pero, qu tan bien representado?...eso se pede
establecer experimentalmente en un laboratorio.
Si se quisiera mejorar la descripcin del sistema, se debera considerar la razn por la cual es sistema alcanza
el equillbrio: el roce con el aire. Experimentalmente se establece que la fuerza que se opone al movimiento
de la masa m es proporcional a la velocidad de m en cada instante. Esta consideracin adicional, junto con
un razonamiento anlogo al anterior permite concluir que el movimiento de la masa m est descrito por una
funcin que satisface la siguiente ecuacin diferencial
y 00 (t) +

y 00 (t) +

q 0
k
y (t) + y (t) = 0
m
m

donde q es la constante de proporcionalidad determinada por el roce.


Ejemplo 6 Una persona M que parte del orgen, se mueve en la direccin positiva del eje x arrastrando un
peso a lo largo de la curva C. El peso, ubicado inicialmente sobre el eje y en el punto (0, s), es arrastrado por
medio de una cuerda de longitud constante s, que se mantiene tensa en el transcurso del movimiento. Cmo
determinar la curva C?.
Para cada x, sea (x, y (x)) el punto C con abscisa x, y sea (x) el ngulo obtuso formado por la recta
tangente a la curva C en el punto (x, y (x)) y el eje de abscisas. Entonces:
dy
sen ( (x))
y (x)
,
= tg ( (x)) =
=p
dx
cos ( (x))
s2 y 2 (x)

de manera que la curva queda determinada por la funcin que satisface la ecuacin diferencial

1.1

Ejercicios.

y (x)
dy
.
=p
dx
s2 y 2 (x)

1. Un cuerpo de masa m que cae encuentra una resistencia del aire proporcional a su velocidad instantnea
v (t). Cmo determinar la posicin y (t) en el instante t?.
2. El istopo radioactivo I 131 se usa en el tratamiento de la hipertiroides y tiene una vida media de 8
dias.
(a) Suponga que se requieren 72 horas para enviar el I 131 del productor al hospital. Qu porcentaje
de la cantidad originalmente enviada llega al hospital?
5

(b) Si el I 131 es almacenado en el hospital 48 horas adicionales antes de ser usado Qu tanto quedara
de la cantidad original enviada por el productor cuando el material radioactivo se utilice?
(c) Cunto tiempo le tomara al I 131 desintegrarse completamente de manera que el hospital pueda
deshacerse de los residuos sin precauciones especiales?.
3. La ley de enfriamiento de Newton dice que la rapidez con que un cuerpo se enfra es proporcional a
la diferencia entre su temperatura y la temperatura del ambiente. Un excursionista sometido a una
temperatura externa de 0o C nota que la temperatura de su caf descendi de 90o C a 80o C en 60 segundos.
Cunto tardar la temperatura del caf en descender hasta 20o C ?.
4. Un peso de 96 libras se desliza por una pendiente que forma un ngulo de 30o con la horizontal. Si el
coeficiente de rozamiento (o friccin) deslizante es , determine la ecuacin diferencial de la velocidad v (t)
del peso en cualquier instante. Se sabe que la fuerza de friccin que se opone al movimiento es N donde
N es la componente normal del peso.
5. Un tonel cilndrico de s pies de dimetro y peso w libras flota en el agua. Despues de un hundimiento parcial
en el momento inicial, el tonel se sigue moviendo verticalmente hacia arriba y hacia abajo. Determine la
ecuacin diferencial de los desplazamientos verticales y (t) sobre un eje vertical en el que el orgen se elige
en la superficie del agua cuando el tonel esta en equilibrio. Emplee el principio de Arqumides (el empuje,
vale decir, la fuerza ejercida
h
i hacia arriba que el agua ejerce sobre el tonel es igual a peso del agua que
lb
desplaza) y tome 62, 4 pie3 como peso especfico del agua. La direccin positiva es la direccin hacia
abajo.

Definiciones, Teoremas Bsicos y Mtodo grfico de soluciones

Definicin 1 Una ecuacin diferencial de primer orden, es una ecuacin de la forma


dy
= f (x, y)
dx

(2.1)

donde f (x, y) es una funcin conocida de las variables x e y.


La incgnita en la ecuacin es y = y (x), que es una funcin de la variable x. Una solucin para esta ecuacin
es cualquier funcin y = y (x) que al ser reemplazada en la ecuacin (2.1) satisface la igualdad determinada por
la ecuacin.
Definicin 2 El problema de resolver la ecuacin
dy
= f (x, y)
dx
sujeta a la condicin
y (x0 ) = y0
se denomina problema de valor inicial, y lo abreviamos P.V.I. La condicin y (x0 ) = y0 se denomina condicin
inicial.

Definicin 3 Una solucin del P.V.I.


dy
= f (x, y) ,
dx
y (x0 ) = y0
es cualquier funcin y = y (x) definida en un intervalo que contiene al punto x0 , que satisface la ecuacin
dy
dx = f (x, y) y que adems satisface y (x0 ) = y0 .
Dado un P.V.I., resulta natural plantearse algunas preguntas respecto a la existencia de su(s) solucin(es):
i) Existe solucin para este P.V.I.?, y supuesto que exista solucin,
ii) cuntas soluciones tiene este P.V.I.?,
iii) existe una nica solucin para el P.V.I.?.
dy
= f (x, y),
Observe que estas preguntas se resumen en la siguiente: Cuntas soluciones tiene la ecuacin dx
cuya grfica pasa por el punto de coordenadas (x0 , y0 )?.
dy
Los siguientes teoremas permiten asegurar que el P.V.I. dx
= f (x, y) , y (x0 ) = y0 tiene solucin:

Teorema 1 (PEANO) Suponga que f : [a, b] [c, d] R es una funcin contnua. Entonces, dado (x0 , y0 )
(a, b) (c, d), existe un intervalo x0 (a1 , b1 ) [a, b] y una funcin y : (a1 , b1 ) [c, d] que es solucin del
P.V.I. y 0 = f (x, y) , y (x0 ) = y0 .
Este teorema garantiza la existencia de al menos una solucin para el P.V.I. y 0 = f (x, y) , y (x0 ) = y0 , pero
no responde a la pregunta: Cuntas soluciones tiene el P.V.I.?. El siguiente ejemplo muestra que un P.V.I.
puede tener ms de una solucin.
1

Ejemplo 7 Considere el P.V.I. y 0 = x |y| 2 , y (0) = 0.


Para y 0, esta ecuacin se puede escribir
1

y 2 y0 = x
o bien
2
integrando

d 1
y2 = x
dx
1

y2 =

x2
+ c,
4

elevando al cuadrado

x2
x4
+ c + c2
16
2
y como es de inters determinar una solucin y = y (x) que satisfaga la condicin inicial y (0) = 0 , se puede
4
concluir que c = 0. De modo que y1 (x) = x16 es una solucin del P.V.I., sin embargo es claro que la funcin
y2 (x) = 0, tambin es solucin del P.V.I., de manera que se tiene al menos dos soluciones para el P.V.I. dado.
En la figura se grafican ambas funciones en un intervalo centrado en x0 = 0, (note que la grfica de y2 (x) = 0
coincide con el eje x).
y (x) =

Figura 2: Las dos soluciones de y 0 = x|y| 2


Ya que segn muestra el ejemplo, un P.V.I. puede tener ms de una solucin, es posible preguntar si existen
condiciones bajo las cuales un P.V.I. dado tiene solucin nica. La respuesta a esta pregunta est dada por el
siguiente teorema:
Teorema 2 (PICARD) Suponga que f : [a, b] [c, d] R y f
y : [a, b] [c, d] R, son funciones contnuas.
Dado (x0 , y0 ) (a, b) (c, d), existe un intervalo x0 (a1 , b1 ) [a, b] y una nica funcin y : (a1 , b1 ) [c, d]
que es solucin del P.V.I. y 0 = f (x, y) , y (x0 ) = y0 .
1

Observacin 1 Note que en el ejemplo anterior f (x, y) = x |y| 2 es una funcin contnua en cualquier rectngulo [a, b] [c, d] que contiene al punto (0, 0), pero no tiene derivada parcial en este punto, de manera que no
1
podemos aplicar el teorema anterior al P.V.I. y 0 = x |y| 2 , y (0) = 0.
Observacin 2 Este teorema asegura que bajo ciertas condiciones un P.V.I. dado tiene una nica solucin,
pero nada dice acerca de cmo determinar esa nica solucin.
Ejemplo 8 Cuntas soluciones tiene el P.V.I. y 0 = y 3 , y (0) = 1?
Es claro que f (x, y) = y 3 es una funcin contnua, al igual que
teorema de Peano, el P.V.I. dado tiene una nica solucin.
La ecuacin puede ser escrita de la siguiente forma:
y 3 y 0 = 1
o bien

1 d 2
= 1,
y
2 dx
de donde, multiplicando por -2 e integrando, se obtiene

y 2 = 2x + C,
8

f
y

(x, y) = 3y 2 , y por lo tanto, segn el

despejando y
y=

1
C 2x

Para determinar el valor de la constante C se utiliza la condicin inicial y (0) = 1. Reemplazando esta condicin
inicial en la ltima igualdad se obtiene
r
1
1=
C
luego C = 1
Ejemplo 9 La ecuacin

dy
dx

= xy es una ecuacin de primer orden en la que f (x, y) = xy.

Para resolver esta ecuacin, esto es, determinar alguna funcin y = y (x) que la satisfaga, se puede, supuesto
que y 6= 0, escribir la ecuacin en la forma
1 dy
=x
(2.2)
y dx
recordando

1 dy
d
(ln |y|) =
dx
y dx

y reemplazando en (2.2), se obtiene


d
(ln (|y|)) = x,
dx
integrando respecto a x, se obtiene
ln (|y|) =

xdx =

x2
+ c, c constante,
2

calculando exponencial
|y| = ec e

x2
2

y por lo tanto
y (x) = ke

x2
2

, k constante arbitraria.

Se puede observar que existen infinitas soluciones de la ecuacin dada, las que dependen del valor asignado a
la constante k. Para seleccionar una solucin en particular es necesario conocer alguna informacin adicional.
En efecto, si, por ejemplo, estamos interesados en la solucin de la ecuacin que satisface la condicin (inicial)
y (1) = 1, entonces el valor de la constante k debe ser tal que
1 = y (1) = ke
de manera que

12
2

= ke 2

k = e 2 ,
y se obtiene la solucin
1

y (x) = e 2 e

x2
2

pero, si es de inters determinar la solucin que satisface y (1) = 1, se obtiene y (x) = e


9

x2
2

12

Es posible determinar aproximadamente la grfica de una solucin de la ecuacin y 0 = f (x, y),


sin resolver la ecuacin?
Para responder a esta pregunta es necesario recordar que las soluciones de la ecuacin son funciones y = y (x)
que satisfacen y 0 (x) = f (x, y (x)), de manera tal que la ecuacin entrega informacin acerca de las pendientes
de las grficas de sus soluciones. De alguna forma, la ecuacin sugiere que por el punto (x, y) del plano se puede
trazar un pequeo segmento (pequeo por razones prcticas) cuya inclinacin sea exactamente el valor dado
por la expresin f (x, y). Si esto se realiza con un nmero conveniente de puntos, se obtiene un conjunto de
segmentos en el plano que puede emplearse como gua para un primer bosquejo de las grficas de las soluciones
de la ecuacin dada. La funcin que a cada punto (x, y) le asigna un segmento de recta con inclinacin f (x, y),
se denomina campo de direcciones. Procediendo de esta forma con la ecuacin y 0 = xy se obtienen las figuras
3 y 4.

Figura 3: Campos de direcciones de y 0 = xy

Figura 4: Curva solucin de y 0 = xy que pasa por (1, 1)

Ejemplo 10 Anotemos y (t) la cantidad de individuos de alguna especie de bacteria (o peces o animales) Cmo
crece el nmero de y (t), supuesto que la tasa de crecimiento sea proporcional a y (t)?
Vemos que
y 0 (t) = ky (t) = f (t, y)
o bien
y 0 = ky
el campo de direcciones de esta ecuacin se muestra en la figura 5

10

Figura 5: Campos de direcciones de y 0 = 3y


si f (t, y) = 0, entonces ky = 0 y 0 = 0 t (aqu las isoclinas, que son curvas con igual valor de y 0 , son
rectas paralelas al eje t).
Cmo determinar las curvas solucin?
1 0
y
y

d
(ln |y|) = k
dt
ln |y| = kt + C1
|y| = Cekt
=

Suponga ahora que los individuos de la especie anterior tienen que competir entre s por el alimento, se
puede pensar que este hecho produce una disminucin en el crecimiento de la poblacin.
2
En el instante t podran ocurrir y (t)y (t) = y (t) encuentros y se puede asumir que
y 0 (t) = ky (t) ay 2 (t)
o bien
y0
y0

= ky ay 2
= (k ay) y

Para simplificar, se supone que k = a = 1 de manera que el crecimiento (o decrecimiento) de la especie est
dada por
y 0 = (1 y) y = y y 2
interpretando geomtricamente, observe la figura 6,

11

Figura 6: Funcin y y 2
el campo de direcciones de la ecuacin diferencial asociado se muestra en la figura 7.

Figura 7: Campo de direcciones de y 0 = y y 2


12

Observe que la especie tiene un volumen de equilibrio natural dado por 1. Ahora, suponga que se trata de
una especie marina, por ejemplo, merluza y se decide extraer una parte C de este volumen de equilibrio natural
por unidad de tiempo, de manera que la poblacin est dada por
y 0 = (1 y) y C
Se extingue la especie o no?. Considere las grficas para los siguientes valores de C ::
Para C = 1, figura 8, el campo de direcciones se muestra en la figura 9, para C = 14 , figura 10, el campo
1
de direcciones se muestra en la figura 11, finalmente para C = 10
, figura 12, el campo de direcciones de la
ecuacin diferencial se muestra en la figura 13.

Figura 8: y y 2 1

Figura 9: Campo de direcciones de

dy
dt

= y y2 1

1
4

Figura 11: Campo de direcciones de

dy
dt

= y y2

Figura 10: y y 2

13

1
4

Figura 12: y y 2 +

2.1

1
10

Figura 13: Campo de direcciones de

dy
dt

= y y2 +

1
10

Ejercicios

1. Dada H (x, y, c) = 0, una familia de curvas a un parmetro. Muestre que existe una ecuacin diferencial
de primer orden que tiene a todas las curvas de esta familia entre sus soluciones. Esta ecuacin es llamada
ecuacin de la familia de curvas.
2. Determine la ecuacin diferencial de la familia de curvas a un parmetro x2 + y 2 = c, o bien x2 + y 2 c =
H (x, y, c) = 0 Grafique las curvas.
3. Determine la ecuacin diferencial de la familia de curvas a un parmetro x2 +y 2 = cy, o bien x2 +y 2 cy =
H (x, y, c) = 0 Grafique las curvas.
4. Cuando todas las curvas de una familia de curvas G (x, y, c) = 0 cortan ortogonalmente a todas las curvas
de otra familia H (x, y, c) = 0, se dice que las familias son, cada una, trayectorias ortogonales de la otra.
dy
Suponga que la ecuacin diferencial de la familia de curvas G (x, y, c) = 0 es la ecuacin dx
= f (x, y).
Determine la ecuacin diferencial de la familia H (x, y, c) = 0 ortogonal a la familia G (x, y, c) = 0.
5. Determine las ecuaciones de las familias de curvas ortogonales a las familias en los ejercicios 2 y 3 anteriores.
Esboce algunas curvas de las familias obtenidas.
6. Determine las soluciones de
y 0 = (1 y) y C, C [0, 1] , constante.
7. Suponga que se extrae una proporcin p de la poblacin y (t) en cada unidad de tiempo, dfe manera que
la ecuacin que determina el crecimiento (decrecimiento) de la poblacin de peces es
y 0 = (1 y) y py, p [0, 1]
Analice la situacin grficamente. Existe algn riesgo de extincin de la especie para algn valor de p?.
14

P
8. Considere el modelo de poblacin dP
dt = 0.4P 1 230 , donde P = P (t) representa el nmero de individuos presentes en la poblacin en el tiempo (instante) t y 230 es la capacidad de soporte del medio, esto
significa que el medio no es capaz de soportar mas de 230 individuos y por lo tanto si en algn instante el
nmero de individuos es mayor que 230, P = P (t) comenzar a disminuir, mientras que si el nmero de
individuos es menor que 230 es posible que la poblacin aumente.
(a) Explique por que la ecuacin representa el hecho que la poblacin crece cada vez que el nmero de
individuos es menor que 230 (y positivo) y disminuye cuando el nmero de individuos es mayor que
230.
(b) Para cules valores de P la poblacin se encuentra en equilibrio? Note que la poblacin se encuentra
en equilibrio cuando su nmero no experimenta cambio.
(c) Para cules valores de P la poblacin se encuentra creciendo?
(d) Para cules valores de P la poblacin est decreciendo?
(e) Observe que P (t) = 0 y P (t) = 230 son soluciones de la ecuacin. Grafique estas soluciones como
funciones de t.
(f) Si P = P (t) es una
solucin
de esta ecuacin, entonces la pendiente de P en t esta dada exactamente
P
por dP
=
0.4P
1

dt
230 en cada punto (Por qu?). Empleando esta observacin y sin resolver la
ecuacin, esboce, en forma aproximada, la grfica de una solucin de la ecuacin supuesto que i)
0 < P (0) < 230, ii) 230 < P (0).
(g) Resuelva la ecuacin y compare las grficas de estas soluciones con las grficas determinadas en el
punto anterior.
2

P
9. La poblacin de peces en una laguna est modelada por la ecuacin dP
dt = 50 + 2P . Suponga que las
autoridades han decidido autorizar la pesca con licencia de manera que cada pescador puede pescar tres
peces durante el ao.

(a) Si las autoridades contratan sus servicios para que usted determine el nmero de licencias que se
puede entregar sin que la poblacin de peces quede en riesgo de extincin, cul sera el nmero
mximo de licencias que usted recomendara?
(b) Suponga que las autoridades entregan el nmero mximo de licencias recomendado por usted. Cul
es su prediccin para el nmero de peces en la laguna a medida que el tiempo transcurre?. Esboce
en forma aproximada una posible solucin de la ecuacin diferencial.
(c) Resuelva analticamente la ecuacin y compare con su esbozo en b.
10. Determine si las siguientes ecuaciones tienen solucin nica:
(a)
(b)
(c)

dy
3
3
dx = x y , y (0)
x dx
dt + 4t = 0, x (2)
dy
y dx = x, y (1) = 0

= 6,
=

11. Considere la ecuacin diferencial

dy
= x + sen (y)
dx

15

(a) Bosqueje el campo de direcciones de la ecuacin.

(b) Una curva solucin pasa por el punto 1, 2 . Cul es su pendiente en ese punto?. Bosqueje la grfica
de esa curva.
(c) Justifique que cada curva solucin es creciente para x > 1.

(d) Muestre que la segunda derivada satisface


1
d2 y
= 1 + x cos (y) + sen (y) .
2
dx
2
(e) Una curva solucin pasa por (0, 0). Demuestre que la curva tiene un mnimo relativo en (0, 0) .
12. Para cada una de las siguientes ecuaciones determine una regin del plano de manera que por cada punto
(x0 , y0 ) pase una nica curva solucin (grfica de la solucin). Dibuje un esbozo del campo de pendientes
correspondiente a cada ecuacin.
dy
dx
0

= y 2/3

(b) y = xy
(a)

(c) (x2 + y 2 )y 0 = y 2
(d) y 0 = x3 cos(y)
(e)

dy
dx

= (x 1)ey/(x1)

13. Se puede afirmar que el P.V.I. y 0 = 3y 2/3 , y(0) = 0 tiene solucin nica?.
Si su respuesta no es
afirmativa determine por lo menos dos soluciones, en caso contrario encuentre la solucin nica.

Ecuaciones de Variables Separables.

En esta seccin se estudiar un tipo particular de ecuaciones de primer orden conocidas como ecuaciones de
variables separables o tambin de variables separadas, cuyas soluciones pueden encontrarse, luego de alguna
manipulacin de tipo algebraica, por integracin directa.
Definicin 4 Se dice que la ecuacin
g(x)
f (x, y) = h(y)
.
Proposicin 1 Una ecuacin

y tiene solucin:

dy
dx

dy
dx

= f (x, y) es de variables separables si f (x, y) puede expresarse como

= f (x, y) de variables separables, se puede escribir como:

g (x)
dy
=
dx
h (y)

(3.1)

(3.2)

h (y) dy =

g (x) dx + C

Observacin 3 La igualdad 3.2 sugiere escribir la ecuacin 3.1 en la forma h (y) dy = g (x) dx o tambin
g (x) dx h (y) dy = 0. La ventaja de esta notacin reside en el hecho que para resolver la ecuacin es suficiente
integrar en cada sumando respecto aR la variableR indicada por la respectiva diferencial. As, la solucin de
g (x) dx h (y) dy = 0 est dada por g (x) dx h (y) dy = C.
16

g(x)
dy
Observacin 4 La ecuacin diferencial dx
= h(y)
, se anotar como g (x) dx + h (y) dy = 0, y se dir que la
ecuacin est expresada en su forma diferencial. Habiendo adoptado esta notacin, se debe tener presente que
cada vez que se encuentre una ecuacin diferencial de la forma g (x) dx + h (y) dy = 0, se debe entender que la
g(x)
dy
ecuacin que se est resolviendo es, en realidad, la ecuacin dx
= h(y)
.

Ejemplo 11 Resolver el P.V.I.


dy
x2 y 2
=
dx
1+x
y (0) = 1
2 2

g(x)
y
x
Aqu se tiene f (x, y) = x1+x
= h(y)
, donde g (x) = 1+x
y h (y) = y12 , de manera que la ecuacin dada es del
tipo de variables separables. Para determinar las soluciones de esta ecuacin se procede de la siguiente forma.
Supuesto que y 6= 0, se puede escribir
x2
1
dy =
dx
2
y
1+x
e integrando miembro a miembro
Z
Z
x2
1
dy
=
dx + C
2
y
1+x
Z
x2
1
=
dx

y
1+x
Z
Z
1
=
(x 1) dx +
dx
1+x
2

(x 1)
+ ln |1 + x| + C
2

de manera que las soluciones de la ecuacin estn determinadas por


y=

(x1)2
2

(3.3)

+ ln |1 + x| + C

Para resolver el P.V.I. dado se debe determinar el valor de la constante C para la que se satisface y (0) = 1.
Reemplazando x = 0 en 3.3
1
1 = y (0) = (01)2
+ ln |1 + 0| + C
2
de donde se obtiene

1
+C
1
3
= 1 =
2
2

1 =
C

1
2

y la solucin del P.V.I. dado es


y=

(x1)2
2

+ ln |1 + x|

3
2

17

(x 1)2 + 2 ln |1 + x| 3

3.1

Ejercicios

1. Dado el P.V.I.
dy
x2 y 2
=
dx
1+x
y (0) = 1
determine si tiene una solucin o ms de una
2. Cuntas soluciones tiene el siguiente P.V.I.: y 0 = y 2 4, y (0) = 2?. Resuelva el P.V.I.
3. Resuelva las siguientes ecuaciones mediante separacin de variables.
dy
= (y + 1)2
(a) (x + 1)y 2 dx

(b)

dy
dx

xy+3xy3
xy2x+4y8

(c) sen(3x)dx + 2y cos3 (3x)dy = 0


dy
(d) (y yx2 ) dx
= (y + 1)2
dy
4. Muestre que una ecuacin de la forma dx
= f (ax + by + c), b 6= 0, puede reducirse a una ecuacin de
variables separables mediante la sustitucin u = ax + by + c.

5. resuelva las siguientes ecuaciones:


(a) y 0 = (x + y)2
(b) y 0 = sen2 (x y + 1)
(c)

dy
dx

= 1 + eyx+5

6. Resuelva las siguientes ecuaciones diferenciales y P.V.I.:


dv
(a) x dx
=

14v 2
3v
cos(x)

(b) y 1 dy + ye
sen (x) dx = 0

2
2
(c) x + xy dx + ex ydy = 0

(d) 1 + x2 + y 2 + x2 y 2 dy = y 2 dx,

(e) y 2 + yx dx + x2 dy = 0,

Ecuaciones Homogneas

Definicin 5 La funcin de las variables x e y, H (x, y), es una funcin homognea de grado , si para x, y
y t convenientemente restringidos, se tiene
H (tx, ty) = t H (x, y)
Observacin 5 En la definicin anterior, la expresin "para x, y y t convenientemente restringidos", debe
entenderse como "para x, y y t en intervalos donde las siguientes expresiones tengan sentido"
18

Ejemplo 12 H (x, y) = x2 y 2 , es homognea de grado dos, esto es, = 2.

xy es homognea de grado 1. En efecto:

M (tx, ty) = txty = |t| xy = tM (x, y) , supuesto que xy 0 y t 0.


p
Ejemplo 14 N (x, y) = x3 + y 3 , es homognes de grado 32 . En efecto
Ejemplo 13 M (x, y) =

N (tx, ty) =

q
3p
3
(tx)3 + (ty)3 = t 2 x3 + y 3 = t 2 N (x, y) .

Ejemplo 15 H (x, y) = x2 + y 3 , no es homognea de grado , cualquiera que sea a. En efecto, si para algn
, se satisface

t2 x2 + t3 y 3 = H (tx, ty) = t H (x, y) = ta x2 + y 3 ,


donde x, y y t estn en ciertos intervalos. Es claro que no puede ser ni cero, uno, dos ni tres (Por qu?).
Despejando se obtiene
2

t t x2 = t3 t y 3

donde x, y y t estn en ciertos intervalos. Derivando cuatro veces respecto a t, resulta

( 1) ( 2) ( 3) t4 x2 = ( 1) ( 2) ( 3) t4 y 3
y como ( 1) ( 2) ( 3) t4 6= 0, se puede cancelar y se tiene
x2 = y 3
lo que es absurdo (Por qu?).
Definicin 6 La ecuacin diferencial de primer orden
M (x, y) dx + N (x, y) dy = 0
es homognea de grado si ambos coeficientes, M (x, y) y N (x, y), son homogneos de grado .

Ejemplo 16 La ecuacin x2 2y 2 dx + xydy = 0, es homognea de grado 2.

Ejemplo 17 La ecuacin x2 y 0 3xy 2y 2 = 0,


de grado 2, ya que puede escribirse como
es homognea

dy
x2 dx
= 3xy + 2y 2 y en forma diferencial como 3xy + 2y 2 dx x2 dy = 0, ecuacin cuyos coeficientes son
homogneos de grado 2.
dy

Ejemplo 18 xsen xy dx
= ysen xy + x, es una ecuacin homognea de grado 1.
Ejemplo 19 xdx + y 2 dy = 0. Los coeficientes de esta ecuacin son homogneos pero tienen grados diferentes,
consecuentemente la ecuacin no es homognea.

Ejemplo 20 x2 + y dx+xdy = 0, no es una ecuacin homognea ya que el coeficiente de dx no es homogneo.

Proposicin 2 Si la ecuacin M (x, y) dx + N (x, y) dy = 0, es homognea, entonces el cambio de variables


y = zx transforma la ecuacin en una ecuacin de variables separables.
19

Observacin 6 El cambio de variables x = zy tambin transforma la ecuacin homognea en otra de variables


separables. Para verificar esto observe que
1
1
dy
.
= dy =
dz
dx
z
+
y dy
dx


Ejemplo 21 Resolver 2 xy y dx xdy = 0.

Esta ecuacin es homognea de grado 1 para x > 0. Consiodere el cambio de variables y = zx. Reemplazando
dy = zdx + xdz en la ecuacin, se obtiene
p

2 x (zx) zx dx x [zdx + xdz] = 0,


factorizando y cancelando x > 0,

separando las variables


2 z z dx [zdx + xdz] = 0

2 z z z dx xdz = 0

2 z z dx xdz = 0
1
1
dz = 0.
dx
x
2 ( z z)

Para resolver esta


ecuacin se integra directamente.
1
Considere u = z, du = 12 z 2 dz = 12 1z dz, de manera que
Z
Z
Z

1
1
1 1
1
dz =

dz =
du = ln z 1
2 (z z)
( z 1) 2 z
u1
Integrando en 4.1 se obtiene

Z
1
1

dz = 0,
dx
x
2 ( z z)

ln |x| + ln z 1 = c

como y = zx , se tiene z = xy , reemplazando en la relacin anterior


r

ln x
1 = c,
x

suponiendo c = ln |k|, calculando exponencial en ambos lados de la igualdad

r
y
1
= k
x
x

xy x = k, k constante real
despejando y, se obtiene facilmente
y=

(x + k)2
, k constante real.
x
20

(4.1)

Ahora, si se intenta el otro cambio de variables que se sugiere en el caso de una ecuacin homognea, a
saber, x = zy. Se tiene dx = zdy + ydz y reemplazando en la ecuacin, se obtiene

p
2 zy 2 y (zdy + ydz) zydy = 0

z 2 z 2 dy + y 2 z 1 dz = 0

1
2 z1

dy +
dz = 0
y
2z ( z 1)

1
z+ z1

dz = 0
dy +
y
2z ( z 1)

1
1
1
dy +
+
dz = 0
y
2 z ( z 1) 2z
integrando directamente
1

ln |y| + ln z 1 + ln |z| = c
2

ln y z z 1 = c

reemplazando z = xy , y calculando la exponencial

xy

x
1 =k
y

despejando y
2

(x k)
, k constante.
x

Ejemplo 22 Resolver la ecuacin 2x3 ydx + x4 + y 4 dy = 0.


y=

La ecuacin es homognea de grado 4. Haciendo x = zy, dx = zdy + ydz y reemplazando, se tiene

2z 3 y 4 (zdy + ydz) + z 4 y 4 + y 4 dy = 0

2z 3 y 5 dz + 3z 4 y 4 + y 4 dy = 0

2z 3 ydz + 3z 4 + 1 dy = 0

1
2z 3
dz + dy = 0
3z 4 + 1
y

1 4
ln 3z + 1 + ln |y| = c = ln k
6

1
y 3z 4 + 1 6 = k
y

y, como z = xy , se obtiene
y

p
6
3z 4 + 1 = k

s
6

3x4 + y 4
=k
y4
21

tomando la sexta potencia


3y 2 x4 + y 6 = k, k 0 constante.

Si M (x, y) y N (x, y) son homogneas de grado , entonces

y
y
M (x, y) = M x, x
= x M 1,
x

yx
y
N (x, y) = N x, x
= x N 1,
x
x

y entonces la ecuacin

M (x, y) dx + N (x, y) dy = 0o
se puede escribir como

De la misma forma, la ecuacin

M (x, y)
dy
=
dx
N (x, y)

y
M 1, y
dy
= yx = F
.
dx
x
N 1, x
y
dy
=F
dx
x

puede escribirse
F
que es una ecuacin homognea de grado 0.

y
x

dx + dy = 0,

dy
= y + xe x , y (1) = 1.
Ejemplo 23 Resolver el P.V.I. x dx

Dividiendo por x,

y
y
dy
y
= + ex = F
dx
x
x
ecuacin que en su forma diferencial es la ecuacin de grado 0 dada por

y
y
+ e x dx dy = 0
x

Para resolver esta ecuacin, cambiando variables y = zx, dy = zdx + xdz y reemplazando

zx
zx
+ e x dx zdx xdz = 0
x
ez dx xdz = 0
1
dx ez dz = 0
x
ln |x| + ez = c

y reemplazando z =

y
x

ln |x| + e x = c

(4.2)

relacin que determina y = y (x) en forma implcita. Para determinar la solucin del P.V.I. dado, se debe
determinar el valor de la constante c. Para esto, en 4.2 se reemplaza x = 1, y = 1
ln |1| + e1 = c
c = e1
22

y entonces la solucin del P.V.I. est dada en forma implcita por


y

ln |x| + e x = e1
explcitamente, la solucin es
y

= x ln e1 ln |x|

= x ln
1 e ln |x|

y
Ejemplo 24 Resolver la ecuacin x2 e x + y 2 dx = xydy
Dividiendo por xy (o multiplicando por

1
xy ),

ser obtiene

x y
y
e x+
dx = dy
y
x

y
y
e x
dx dy = 0
y +
x
x
esta ltima ecuacin es homognea con coeficientes de grado 0. Para resolverla considere y = zx, dy = zdx+xdz,
al reemplazar en la ecuacin

z
e
+ z dx (zdx + xdz) = 0
z
ez
dx xdz = 0
(4.3)
z
1
dx zez dz = 0
x
de manera que la solucin de la ecuacin 1.15 est dad en forma implcita por
Z
Z
1
dx zez dz = c, c R
x
R z
R
Integrando ze dz por partes se obtiene zez dz = (z 1) ez , de manera que la solucin de 1.15 est dada
implcitamente por
ln |x| (z 1) ez = c, c R.
(4.4)
2 y

Ahora bien, no se debe olvidar que es la ecuacin x e x + y 2 dx = xydy, la que se est resolviendo, lo que
an no se ha realizado. Para lograr este objetivo basta reemplazar z = xy en 1.16 y se obtiene
y
y
ln |x|
1 e x = c, c R,
x
relacin que determina la solucin de la ecuacin en forma implcita.

23

4.1

Ejercicios

1. Si ae 6= bd muestre que se puden elegir constantes h y k de modo tal que las sustituciones x = z h, y =
dy
ax+by+c
w k , transforman la ecuacin dx
= F ( dx+ey+f
) en una ecuacin homognea.
2. Resuelva las siguientes ecuaciones:
(a)

dy
dx

xy3
x+y1

(b) (2x + 3y 1)dx 4(x + 1)dy = 0


3. Resuelva las siguientes ecuaciones homogeneas
(a) ydx + (x +

xy)dy = 0

(b) 2x2 ydx = (3x3 + y 3 )dy


(c)

dy
dx

y
x

x
y

(d)

dy
dx

y
x

x2
y2

+1

(e) (y 2 + 3xy)dx = (4x2 + xy)dy , y(1) = 1

(f) y 3 dx = 2x3 dy 2x2 ydx, y(1) = 2


p
dy
= y , y( 12 ) = 1
(g) (x + y 2 xy) dx

Ecuaciones Exactas y Factores de Integracin

Antes de definir que clase de ecuaciones diferenciales son llamadas exactas, consideraremos el siguiente teorema:
Teorema 3 (De la funcin implcita) Suponga que f : R2 R, es de clase C 1 (las derivadas existen y son
contnuas) en un rectngulo de R2 que contiene al punto (x0 , y0 ) en su interior. Si f (x0 , y0 ) = c, c constante,
y f
y (x0 , y0 ) 6= 0, entonces existe un intervalo abierto I que contiene a x0 y una funcin y : I R, y = y (x),
tal que
i. f (x, y (x)) = c, para todo x I.
ii. y 0 (x) =

dy
dx

(x,y(x))

(x) = x
para todo x I.
f
(x,y(x))
y

Observe que la conclusin del teorema dice algo respecto a las soluciones de ciertas ecuaciones diferenciales.
En efecto, sea f (x, y) una funcin de las variables x e y, si se denota M (x, y) = f
x (x, y (x)) y N (x, y) =
f
y (x, y (x)) y se considera el P.V.I.
)
(x,y)
y 0 (x) = M
N (x,y)
,
y (x0 ) = y0
el teorema de la funcin implcita asegura que si N (x0 , y0 ) = f
y (x0 , y0 ) 6= 0, entonces el P.V.I. tiene una
nica solucin en un intervalo abierto que contiene al punto x0 y est dada implcitamente por la relacin
f (x, y (x)) = c. Este hecho justifica la siguiente definicin:

24

Definicin 7 Se dice que la ecuacin M (x, y) dx + N (x, y) dy = 0 es exacta, si existe una funcin de dos
f
variables f (x, y), tal que M (x, y) = f
x (x, y (x)) y N (x, y) = y (x, y (x)) .
Si la ecuacin M (x, y) dx + N (x, y) dy = 0 es exacta, y N (x0 , y0 ) 6= 0, entonces el teorema de la funcin
implcita garantiza que el P.V.I.

M (x, y) dx + N (x, y) dy = 0
y (x0 ) = y0
tiene una nica solucin, que est determinada implcitamente por la relacin
f (x, y (x)) = c0
donde c0 = f (x0 , y0 ). En otras palabras, la solucin est determinada por la curva de nivel c0 = f (x0 , y0 ) de
la funcin f .
Ejemplo 25 Resolver el P.V.I.:
2xdx + 2ydy = 0
y (2) = 4
Esta ecuacin es exacta, pues las funciones M (x, y) = 2x y N (x, y) = 2y coinciden con las derivadas parciales
de f (x, y) = x2 +y 2 respecta a x e y respectivamente. El teorema de la funcin implcita afirma que la solucin de
la ecuacin 2xdx + 2ydy = 0 est dada implcitamente por f (x, y) = c, esto es x2 + y 2 = c. Ahora bien, como es
de inters determinar aquella solucin que tiene la propiedad adicional y (2) = 4 (satisfacer la condicin inicial),
se determina c reemplazando estos valores en la relacin f (x, y) = c.
Se obtiene f (2, 4) = 22 + 42 = 20 = c0 .
Despejando y se obtiene la solucin al P.V.I.:
p
y (x) = 20 x2
Observe que, si bien es cierto que la ecuacin dada en este ejemplo es una ecuacin exacta,
tambin
es una
R
R
ecuacin de variables separadas, de modo que puede resolverse integrando directamente: 2xdx + 2ydy = c,
de donde x2 + y 2 = c.
Ejemplo 26 Resolver el P.V.I.
2xdx 2ydy = 0
y (0) = 2
Esta ecuacin es exacta, pues las funciones M (x, y) = 2x y N (x, y) = 2y, coinciden con las derivadas
parciales de f (x, y) = x2 y 2 respecto a x e y respectivamente. El teorema de la funcin implicita afirma que
la solucin de la ecuacin 2xdx 2ydy = 0, est dada implcitamente por f (x, y) = c, esto es x2 y 2 = c. Para
determinar la solucin que satisface la condicin inicial y (0) = 2, se determina c reemplazando estos valores
en la relacin f (x, y) = c. Luego: f (0, 2) = 02 22 = 4 = c0 , por lo tanto, la solucin del P.V.I. dado es:
x2 y 2 = 4. Despejando y se obtiene la solucin del P.V.I.
p
y = 4 + x2
Note que, tambin esta ecuacin es de variables separables, de modo que puede resolverse mediante integracin
directa. Por supuesto que no debe pensarse que toda ecuacin exacta es de variables separables. Fcilmente
25

se puede construir un ejemplo de una ecuacin exacta que no es de variables separables. En efecto, es suficiente considerar una funcin de dos variables f (x, y) de manera que la ecuacin exacta determinada por ella
f
f
x (x, y (y)) dx + y (x, y (x)) dy = 0, no sea de variables separables. Por ejemplo, si f (x, y) = x + sen (xy),
se obtiene la ecuacin exacta (1 + y cos (xy)) dx + x cos (xy) dy = 0, que no es de variables separables. Como
esta ecuacin es de variables separables, su solucin est determinada implcitamente por las curvas de nivel de
la funcin f (x, y) = x + sen (xy), esto es, las soluciones estn determinadas por la relacin x + sen (xy) = c,
donde c es una constante arbitraria.
Entonces, cmo se puede reconocer si una ecuacin dada M (x, y) dx + N (x, y) dy = 0 es exacta o no. Claraf
mente, si se puede determinar alguna funcin f (x, y) tal que M (x, y) = f
x (x, y (x)) y N (x, y) = y (x, y (x)),
entonces no existir ninguna duda sobre la naturaleza de la ecuacin, y se habr encontrado la solucin de la
ecuacin despejando y de la relacin f (x, y) = c.
f
Y si no es tan fcil determinar la funcin f (x, y) tal que M (x, y) = f
x (x, y (x)) y N (x, y) = y (x, y (x)),
entonces considere el siguiente teorema:
Teorema 4 Si f (x, y) es de clase C 2 (dos veces continuamente diferenciable), entonces las derivadas parciales
mixtas de orden dos son iguales; esto es:
2f
2f
(x, y) =
(x, y)
xy
yx
Ahora bien, suponga que la ecuacin M (x, y) dx + N (x, y) dy = 0, es exacta y que las funciones M (x, y) y
f
N (x, y) son de clase C 1 . Entonces, existe una funcin f (x, y) tal que M (x, y) = f
x (x, y) y N (x, y) = y (x, y),
y, como las derivadas parciales mixtas de segundo orden de f (x, y) son iguales, se puede afirmar que
2f
2f
N
M
(x, y) =
(x, y) =
(x, y) =
(x, y)
y
yx
xy
x
N
Se concluye que, si M (x, y) dx + N (x, y) dy = 0, es exacta, entonces M
y (x, y) = x (x, y).
N
Es natural la pregunta acerca de que si la igualdad M
y (x, y) = x (x, y) garantiza que la ecuacin
M (x, y) dx + N (x, y) dy = 0 es exacta. La respuesta la da el siguiente teorema:

Teorema 5 Si M (x, y) y N (x, y) son de clase C 1 , entonces


M
N
(x, y) =
(x, y)
y
x
si, y solamente si,
M (x, y) dx + N (x, y) dy = 0 es exacta.
Proposicin 3 Si M (x, y) dx + N (x, y) dy = 0 es exacta, entonces la solucin de la ecuacin es:
Z

Z
Z

f (x, y) :
M (x, y) dx +
N (x, y)
M (x, y) dx dy = C
y
Z

Z
Z

f (x, y) :
N (x, y) dy +
M (x, y)
N (x, y) dy dx = C
x

2
Ejemplo 27 Resolver la ecuacin 2xydx + x y dy = 0.
26

La ecuacin es exacta pues

2
(2xy) = 2x =
x y
y
x

la solucin de la ecuacin est determinada por f (x, y) = c, donde f


x (x, y) = 2xy y
bien, utilizando la proposicin se tiene:
Z

Z
Z

2
2xydx dy = C
2xydx +
x y
y

Z
2
2
2
x y
x y+
x y dy = C
y
Z
2

x2 y +
x y x2 dy = C
Z
x2 y + ydy = C
x2 y

y2
2

f
y

(x, y) = x2 y. Ahora

= C

Se puede despejar y para obtener finalmente y como funcin de x:


p
y = x2 + x4 2c

Observacin 7 Note que existen dos posibles soluciones al despejar la variable y en la solucin implcita, pero
una sla satisface la ecuacin diferencial.

Ejemplo 28 Resolver la ecuacin e2y y cos (xy) dx + 2xe2y x cos (xy) + 2y dy = 0.


Note que esta ecuacin no es de variables separables ni homognea. La ecuacin es exacta:

2y
e y cos (xy) = 2e2y cos (xy) + xysen (xy)
y

=
(2xexy x cos (xy) + 2y)
y

2y
y cos (xy) y f
la solucin de la ecuacin est determinada por f (x, y) = c, donde f
x (x, y) = e
y (x, y) =
2y
2xe x cos (xy) + 2y. Luego,

Z
Z
Z
2y

2y

e y cos (xy) dx +
2xe2y x cos (xy) + 2y
e y cos (xy) dx dy = C
y

2y
xe2y sen (xy) +
2xe2y x cos (xy) + 2y
xe sen (xy) dy = C
y
Z

xe2y sen (xy) +


2xe2y x cos (xy) + 2y 2xe2y + x cos (xy) dy = C
Z
2y
xe sen (xy) + 2ydy = C

xe2y sen (xy) + y 2

= C

No se intentar despejar el valor y = y (x) de esta relacin, pues resulta muy difcil de realizar o incluso
imposible.
27

Ejemplo 29 Resolver el siguiente PV.I.

2y

e y cos (xy) dx + 2xe2y x cos (xy) + 2y dy = 0


y (0) = 1
Como la ecuacin dada en este P.V.I. es la misma del ejemplo anterior, se puede recurrir a la solucin
obtenida para determinar el valor de la constante c correspondiente, pero para ejemplificar el hecho de que
podemos resolver la ecuacin exacta considerando la segunda forma de solucin, se determinar la solucin de
esta ecuacin a partir de este supuesto y posteriormente se proceder a evaluar el valor de la constante c.

Z
Z
Z

2y

2xe2y x cos (xy) + 2y dy +


e y cos (xy)
2xe2y x cos (xy) + 2y dy dx = C
x

2y
xe2y sen (xy) + y 2 +
e2y y cos (xy)
xe sen (xy) + y 2 dx = C
x
Z

xe2y sen (xy) + y 2 +


e2y y cos (xy) e2y + y cos (xy) dx = C
Z
xe2y sen (xy) + y 2 + 0dx = C
xe2y sen (xy) + y 2

= C

Luego, se ha obtenido el mismo resultado que en el ejercicio anterior. Para determinar la solucin del P.V.I.,
se reemplaza la condicin inicial en la relacin anterior. Se obtiene
0e21 sen (0 1) + 12 = C
de donde C = 1, de manera que la solucin del P.V.I. dado est determinado implcitamente por
xe2y sen (xy) + y 2 = 1

5.1

Factores de Integracin

Considere la ecuacin
M (x, y) dx + N (x, y) dy = 0

(5.1)

y suponga que su solucin pueda expresarse implcitamente por la relacin f (x, y) = c, para alguna funcin f
de clase C 2 . Existe alguna funcin = (x, y), tal que la ecuacin
(x, y) M (x, y) dx + (x, y) N (x, y) dy = 0

(5.2)

resulte exacta?. Si la solucin de (5.1) puede expresarse por f (x, y) = c, entonces, aplicando el teorema de la
funcin implcita y la ecuacin (5.1), se puede afirmar que
f
x
f
y

(x, y)
(x, y)

dy
M (x, y)
=
,
dx
N (x, y)

entonces resulta inmediato de verificar que si se define


f
(x, y)
y (x, y)
(x, y) =
=
M (x, y)
N (x, y)
f
x

entonces la ecuacin (5.2) es una ecuacin exacta.


28

Definicin 8 La funcin = (x, y) es un factor integrante para la ecuacin M (x, y) dx + N (x, y) dy = 0, si


la ecaucin (x, y) M (x, y) dx + (x, y) N (x, y) dy = 0 es una ecuacin exacta.
Proposicin 4 (x, y) es un factor de integracin de la ecuacin (5.1):
i. (x, y) depende slo de la variable x, s
1 (x)
=
(x) x
es decir, si el cuociente

M
y

(x,y) N
x (x,y)
N (x,y)

M
y

(x, y)

N
x

(x, y)

N (x, y)

(5.3)

= (x) es una funcin que slo depende de x.

ii. (x, y) depende slo de la variable y, s


1 (y)
=
(y) y
es decir, si el cuociente

N
x

(x,y) M
y (x,y)
M(x,y)

N
x

(x, y)

M
y

(x, y)

M (x, y)

(5.4)

(y) es una funcin que slo depende de y.

iii. si z = z (x, y) depende de las variables x e y, entonces (x, y) = (z), depende de z, s


1 d (z)
=
(z) dz
es decir, el cuociente

N (x,y)
M(x,y)
x
y
z
z
y M(x,y) x N(x,y)

N(x,y)
x
z
y M

(x, y)

M (x,y)
y
z
x
N (x, y)

(5.5)

= (z) es una funcin que slo depende de z.

Ejemplo 30 Determine un factor de integracin para la ecuacin (xy 1) dx + x2 xy dy = 0 y resuelva la


ecuacin.
La ecuacin dada no es exacta. Para determinar un factor de integracin se puede verificar si esta ecuacin
admite un factor de integracin que no dependa de y, es decir, que slo dependa de x. Para esto, se puede
proceder de dos formas: la primera es recurrir a la ecuacin (5.3) y verificar si la expresin
M
y

(x, y)

N
x

(x, y)

N (x, y)

no depende de y. Una simple evaluacin:


M
y

(x, y)

N
x

N (x, y)

(x, y)

x (2x y)
1
=
2
x xy
x

de manera que efectivamente el lado izquierdo de la ecuacin (5.3) no depende de y. Resolviendo esta ecuacin
se puede determinar un factor de integracin que slo depende de x :
1
1 (x)
=
(x) x
x
29

(5.6)

o bien

d
1
(ln | (x)|) =
dx
x
integrando, tomando la exponencial se obtiene el factor de integracin
(x) =

C
x

Antes de continuar la solucin de la ecuacin considere un segundo procedimiento para determinar = (x) :
Suponga que existe un factor de integracin = (x) que slo depende de x, entonces la ecuacin

(x) (xy 1) dx + (x) x2 xy dy = 0


es exacta, de manera que

y derivando parcialmente

esto es


( (x) (xy 1)) =
(x) x2 xy
y
x

2
(x)

(x) 2
(xy 1) + (x)
(xy 1) =
x xy + (x)
x xy
y
y
x
x
(x) x =

(x) 2
x xy + (x) (2x y)
x

1 (x)
y despejando el producto (x)
x se obtiene la ecuacin

x (2x y)
1 (x)
=
(x) x
x2 xy
que luego de alguna simplificacin algebrica se transforma en la ecuacin
1 (x)
1
=
(x) x
x
que es la ecuacin (5.6) de donde
C
x
Ahora que el factor de integracin ha sido determinado, resulta relativamente fcil resolver la ecuacin dada.
En efecto, al multiplicar por (x) = Cx , se obtiene la ecuacin exacta

C x2 xy
C (xy 1)
dx +
dy = 0
x
x
(x) =

y simplificando

1
y
dx + (x y) dy = 0.
x

30

Luego, resolvemos:

Z

Z
Z
1

1
y
dx +
(x y)
y
dx dy
x
y
x

yx ln |x| +
(x y)
(yx ln (x)) dy
y
Z
yx ln |x| + [(x y) x] dy
Z
yx ln |x| + (y) dy
yx ln |x|

y2
2

= C
= C
= C
= C
= C

y la solucin de la ecuacin dada est determinada en forma implcita por


yx ln |x|

y2
= C, C constante real.
2

Se deja como ejercicio despejar el valor de la funcin y en trminos de la variable x.


En el ejemplo anterior fue posible encontrar un factor de integracin que slo depende de la variable x. Por
supuesto, esto no es siempre as, como lo muestra el siguiente ejemplo.
Ejemplo 31 Determine un factor de integracin para la ecuacin

2xydx + 3x2 y 2 dy = 0
y resuelva la ecuacin.

Primero observe que la ecuacin dada no es exacta. Se verifica si la ecuacin admite un factor de integracin
= (x). Segn la discusin anterior, si existe un factor puede determinarse como solucin de la ecuacin (5.3)
siempre que la expresin
M (x,y)
(x,y)
Nx
y
N (x, y)

slo depende de la variable x. Evaluando esta expresin con los valores M (x, y) = 2xy y N (x, y) = 3x2 y 2 ,
se obtiene
M (x,y)
N(x,y)
8x
y
x
= 2
N (x, y)
3x y 2

que es una funcin de las variables x e y, por lo tanto la ecuacin dada no admite un factor de integracin que
slo depende de x. Entonces, utilizando (5.4), se tiene:
4
1 (y)
=
(y) y
y

Resolviendo esta ecuacin se obtiene (y) = y 4 . Note que no se ha considerado la constante de integracin
(Por qu?). Para resolver la ecuacin dada, se multiplica por el factor de integracin para obtener la ecuacin
exacta:

2xy 3 dx + 3x2 y 4 y 2 dy = 0.
31

Se procede como de costumbre.

Z
Z
Z

2 4

3
2
3
dx +
3x y y

dx dy
2xy
2xy
y

Z
2 4

2 3
2 3
2
x y +
3x y y
dy

x y
y
Z
2 4

3x y y 2 3x2 y 4 dy
x2 y 3 +
Z
2
y
dy
x2 y 3 +
x2 y 3 +

1
y

= C
= C
= C
= C
= C

de donde se tiene que la solucin de la ecuacin dada est determinada implcitamente por
1 x2
3 = C, donde C es constante.
y
y

Ejemplo 32 Sabiendo que la ecuacin y 2 dx + x2 + xy dy = 0, tiene un factor de integracin que depende


de x2 y, determine sus soluciones.
Sea z = z (x, y) = x2 y. Entonces, el lado derecho de la ecuacin (5.5) debe poder expresarse como una
funcin de la variable z, y si este es el caso, el factor puede determinarse resolviendo (5.5):
N (x,y)
x
z
y M

(x, y)

M (x,y)
y
z
x N (x, y)

x2

(2x + y) (2y)
(y 2 ) 2xy (x2 + xy)

2x + 3y
2x3 y + 3x2 y 2
2x + 3y
= 2
x y (2x + 3y)
1
= 2
x y

de manera que el lado derecho de la ecuacin (5.5) se puede expresar como una funcin (z) de la variable
z = x2 y, a saber (z) = z1 . Para determinar el factor de integracin se resuelve la ecuacin (5.5) expresada
en trminos de la variable z:
1 d (z)
1
= .
(z) dz
z

1
1
Se obtiene (z) = z = x2 y . Ahora, se multiplica la ecuacin y 2 dx + x2 + xy dy = 0 por (z) para obtener
la ecuacin exacta

y
1 1
2 dx +
+
dy = 0.
x
y x

32

Para resolver esta ltima ecuacin se procede de la forma habitual:

Z
Z
Z
1
1

y
y
+

2 dx dy
2 dx +
x
y x
y
x

Z
1
1
y
y
+
+

dy
x
y x
y x

Z
1 1
1
y
+
+

dy
x
y x
x
Z
1
y
+
dy
x
y
y
+ ln |y|
x

= C
= C
= C
= C
= C

y la solucin de la ecuacin dada est determinada en forma implcita por


y
+ ln |y| = C, donde C es una constante.
x
Cmo se procedera para resolver el ejemplo anterior, si no se recuerda la frmula (5.5)?.

5.2

Ejercicios

1. Determine si la ecuacin dada es exacta, y si lo es resulvala:


(a) (2x + y) dx (x + 6y) dy = 0
(b) ( sen(y) ysen(x)) dx + (cos(x) + x cos(y) y) dy
(c) (x3 + y 3 )dx + 3xy 2 dy = 0

(d) (x + y)2 dx + (2xy + x2 1)dy = 0


2. Determine M (x, y) N (x, y) para que la ecuacin dada resulte exacta:
(a) M (x, y)dx + (xexy + 2xy + x1 )dy = 0
(b) (y 1/2 x1/2 +

x
x2 +y )dx

+ N (x, y)dy = 0

3. Resuelva los siguientes P.V.I.:


(a) (ex + y) dx + (2 + x + yey ) dy = 0, y (0) = 1.

(b) x3 + xy dx + y 2 + ln (x) dy = 0, y (1) = 1


(c) (cos (x) + ln (y)) dx + xy + ey dy = 0, y 2 = 1

4. Muestre que la funcin (x, y) dada es un factor de integracin para la correspondiente ecuacin:
(a) 6xydx + (4y + 9x2 )dy = 0, (x, y) = y 2 Cmo determinara este factor?
(b) y 2 dx + (x2 + xy)dy = 0,

(x, y) =

1
x2 y

33

(c) (xysen(x) + 2y cos(x))dx + 2x cos(x)dy = 0, (x, y) = xy


5. Determine si las siguientes ecuaciones admiten o n factores de integracin de la forma = (x)
= (y) . Si es el caso resuelva la ecuacin.
(a) xdy + ydx + 3x3 y 4 dy = 0

(b) 3x2 y 2 dy 2xydx = 0

(c) ex dx + (ex cot(y) + 2y csc(y))dy = 0

(d) ydx + (x 2x2 y 3 )dy = 0


(e) ydx + (2x yey )dy = 0

(f) (y 2 + xy + 1)dx + (x2 + xy + 1)dy = 0

R.-.

= exy , exy (x + y) = c

6. Bajo qe condiciones la ecuacin M (x, y)dx + N (x, y)dy = 0 tendr un factor integrante que sea funcin
de la suma z = x + y ?

Ecuaciones Lineales de Primer Orden.

Se llama ecuacin lineal de primer orden a la ecuacin


dy
+ p (x) y = q (x)
dx

(6.1)

Esta ecuacin puede escribirse en la forma diferencial


(p (x) y q (x)) dx + dy = 0,
luego tiene el factor de integracin
(x) = e

pxdx

Multiplicando la ecuacin (6.1) y resolviendo se obtiene:

Z R
R
p(x)dx
p(x)dx
q (x) + c , c R ,
e
y (x) = e

(6.2)

(6.3)

que es la solucin general de la ecuacin lineal de primer orden (6.1).


Ejemplo 33 Determine la solucin general de la ecuacin
y 0 + y = e3x .
La ecuacin es lineal de primer orden y est en la forma normal, de manera que admite el factor de integracin
R

(x) = e

dx

= ex .

Multiplicando la ecuacin por este factor se obtiene:


d x
(e y) = e4x ,
dx
34

de donde

1 4x
e + c,
4
y entonces, la solucin general de la ecuacin est dada por
ex y =

y (x) =

1 3x
e + cex , c R.
4

Ejemplo 34 Determine la solucin general de la ecuacin


xy 0 + 4y = x3 x
La ecuacin dada es lineal de primer orden, pero no se encuentra en la forma normal. Para determinar
un factor de integracin que slo depende de la variable x, es necesario normalizar la ecuacin. Para esto se
multiplica la ecuacin por x1 :
4
y 0 + y = x2 1
x
y el factor de integracin es
R 4
(x) = e x dx = e4 ln(x) = x4 .
Se multiplica la ecuacin normalizada por el factor de integracin, para obtener la ecuacin exacta
x4 y 0 + 4x3 y = x6 x4

(6.3)

(Investigue que ocurre si se multiplica la ecuacin original por el factor de integracin. Resulta exacta?Cmo
explica lo que ocurre?).
Observe que la ecuacin (6.3) puede escribirse como
d 4
x y = x6 x4
dx

integrando se obtiene

x4 y =

x7 x5

+c
7
5

y, despejando y, se encuentra la solucin general


y=

6.1

x
c
x3
+ 4 , c R.
7
5 x

La ecuacin de Bernoulli.

La ecuacin de Bernoulli normalizada es la ecuacin


y 0 + p (x) y = q (x) y n
donde n es un nmero real arbitrario fijo.
Para resolver esta ecuacin se multiplican ambos miembros por y n , para obtener
y n y 0 + p (x) y n+1 = q (x) ,

35

(6.4)

y se observa que y n y 0 "casi coincide" con la derivada de y n+1 . Esta observacin sugiere el cambio de variable
z = y n+1 ,
de donde se obtiene

dz
= (1 n) y n y 0 ,
dx

o bien

1
z0,
1n
de manera que, reemplazando en la ecuacin (6.4) resulta
y n y 0 =

1
z 0 + p (x) z = q (x) ,
1n
y normalizando, se obtiene la ecuacin de primer orden
z 0 + (1 n) p (x) z = (1 n) q (x) .
Ejemplo 35 Resolver la ecuacin
xy 0 + y = y 2 .
La ecuacin dada no es una ecuacin de Bernoulli normalizada, pero multiplicando por x1 se obtiene la
ecuacin de Bernoulli normalizada
1
1
y 0 + y = y 2 .
x
x
Multiplicando esta ecuacin por y (2) = y 2 , se obtiene
1 3
1
y =
x
x

y2 y0 +

(6.5)

Haciendo el cambio de variable


z

= y (2)+1 = y 3 ,

z0

= 3y 2 y 0 , o bien y 2 y 0 =

1 0
z
3

reemplazando en la ecuacin (6.5)


1 0 1
1
z + z= ,
3
x
x
y normalizando, se obtiene la ecuacin lineal
z0 +
que admite el factor de integracin

3
3
z=
x
x
R

(x) = e

3
x dx

= x3 .

Multiplicando la ecuacin (6.6) por el factor de integracin resulta


d 3
x z = 3x2 ,
dx
36

(6.6)

integrando
x3 z = x3 + c,
y como z = y 3 se obtiene, finalmente
y (x) =

p
3
1 + cx3 , c R.

Ejemplo 36 Determine la solucin de la ecuacin

y 0 + x1 y = xy 2 .
Multiplicando la ecuacin por y 2 se obtiene
y 2 y 0 + x1 y 1 = x,
haciendo z = y 1 , se tiene y 2 y 0 = z, y reemplazando en la ecuacin anterior
z 0 +
Normalizando
z0

1
z = x.
x

1
z = x.
x

Un factor de integracin para esta ecuacin es


R

(x) = e

1
x
dx

= e ln(x) =

1
.
x

Multilplicando ambos lados de la ecuacin se tiene



d 1
z = 1,
dx x
integrando
1
z = x + c, c R,
x
despejando z y reemplazando z = y 1 , se obtiene la solucin
1
y

= x2 + xc, c R

1
, c R.
cx x2

Ejemplo 37 Determine la solucin de la ecuacin

2 1 x2 y 0 1 x2 y = xy 3 ex
Normalizando

1
xex
y0 y =
y3 ,
2
2 (1 x2 )

37

multiplicando por y 3

1
xex
y 3 y 0 y 2 =
,
2
2 (1 x2 )

haciendo z = y 2 , se tiene y 3 y 0 = 12 z 0 y luego

1
1
xex
z0 z =
,
2
2
2 (1 x2 )
y normalizando
z0 + z =
R

se multiplica por el factor de integracin (x) = e

dx

xex
,
x2 1

= ex :

x
d x
(e z) = 2
,
dx
x 1
integrando
ex z =
despejando z y reemplazando por y 2 se tiene
1
y2
y

6.2

1 2
ln x 1 + c, c R,
2

1 x 2
e ln x 1 + cex , c R
2
r
ex

, c R.
=
ln x2 1 + c
=

La ecuacin de Riccati.

La ecuacin de Riccati es la ecuacin


dy
= p (x) + q (x) y + r (x) y 2 .
dx

(6.7)

Si se conoce una solucin y1 (x) de esta ecuacin, entonces el cambio de variable


y (x) = y1 (x) +

1
z (x)

transforma esta ecuacin en una ecuacin lineal de primer orden. En efecto, reemplazando
y
y0
en la ecuacin
y10

1
= y1 + , e
z
z0
= y10 2
z

2
z0
1
1
=
p
(x)
+
q
(x)
y
+
+
+
r
(x)
y
1
1
z2
z
z
38

(6.8)

y desarrollando

z0
1
y1
1
= p (x) + q (x) y1 + q (x) + r (x) y12 + 2r (x)
+ 2
z2
z
z
z
y como y1 es solucin de la ecuacin (6.7), se tiene que
y10

1
z0
1
1
= q (x) + 2r (x) y1 + r (x) 2 ,
z2
z
z
z

normalizando, se obtiene la ecuacin lineal de primer orden


z 0 = (q (x) + 2r (x) y1 ) z r (x)
o bien
z 0 + (q (x) + 2r (x) y1 ) z = r (x)
Ejemplo 38 Determine las soluciones de la ecuacin
y 0 xy 2 + (2x 1) y = x 1,
sabiendo que y1 (x) = 1 es una solucin.
Reemplazando en la ecuacin se verifica fcilmente que y1 (x) es efectivamente una solucin. Luego de
verificar este hecho, se realiza el cambio de variables
y (x) = 1 +
y0

1
z (x)

z0
z2

reemplazando en la ecuacin, se tiene

z0
1
1
2 x 1+
+ (2x 1) 1 +
= x 1,
z
z
z
desarrollando

2x
2x
x
z0
x
2 + 2x +
1
z2
z
z
z
0
x
z
2 2
z
z

1
z
1
z

= x 1,
= 0,

normalizando
z 0 + z = x

La ecuacin lineal de primer orden tiene como factor de integracin (x) = ex


d x
(e z) = xex ,
dx

39

integrando, y reemplazando z:
ex z
z

= xex + ex + c, c R
= 1 x + cex , c R,

1
y (x) 1

= 1 x + cex , c R,

y (x) = 1 +

1
, c R,
1 x + cex

que es la solucin pedida.


Ejemplo 39 Determine la solucin de la ecuacin
y 0 + xy 2 2x2 y + x3 = x + 1,
sabiendo que la y1 (x) = x 1, es una solucin particular de la ecuacin.
Por inspeccin se verifica que y1 (x) = x 1 es una solucin. Considere
1
y = (x 1) + .
z
Reemplazando en la ecuacin, se tiene

1
1
z0
2x2 (x 1) +
1 2 + x (x 1) +
+ x3 = x + 1,
z
z
z
desarrollando

z0
2x2 2x
2x2
x
3
2
3
2
+
x

2x
+
x
+

2x
+
2x

+
+ x3 = x + 1,
z2
z
z
z2
z
cancelando y sumando trminos semejantes
1

z0
2x
x

+ 2 = 0,
z2
z
z

normalizando
z 0 + 2xz = x,
R

multiplicando por el factor integrante: (x) = e

integrando

2xdx

= ex , se tiene

2
d x2
e z = xex ,
dx
2

ex z =

1 x2
e + c, c R,
2

y como y = x 1 + z1 , se obtiene
y =x1+

2
, c R.
1 + cex2

que es la solucin pedida.


40

6.3

Ejercicios

1. Resolver las ecuaciones lineales:


dy
3y = x4
(a) x dx

(b) y 0 + y = (1+e1 2x )

(c) 2y x3 dx = xdy

(d) (1 + x2 )dy + 2xydx = cot(x)dx


(e) (y 2xy x2 )dx + x2 dy = 0

2. Resuelva las siguientes ecuaciones:


(a) y 0 y = ex y 2

(b) xy 0 (1 + x)y = xy 2

(c) 3(1 + x2 )y 0 = 2xy(y 3 1)

(d) y 1/2 y 0 + y 3/2 = 1 , y(0) = 4


3. Resolver las siguientes ecuaciones:
(a)
(b)
(c)
(d)

dy
dx
dy
dx
dy
dx
dy
dx

= 2 y + y 2 , note que y = 2 es una solucin,


= 1 x y + xy 2 , y = 1
= 6 + 5y + y 2

= sec2 (x) tan(x)y + y 2 , y = tan(x)

4. Resuelva los siguientes P.V.I.

(a) x3 y dx + xdy = 0, y (1) = 3

(b) (t + x + 3) dt + dx = 0, x (0) = 1
dy
(c) (x + 1) dx
+ y = ln (x) , y (1) = 10

dy
2xy = 3y 4 , y (1) =
(d) x2 dx

1
2

Aplicaciones de las Ecuaciones de Primer Orden.

Ahora se darn ejemplos de aplicaciones de algunas de las ecuaciones diferenciales estudiadas hasta ahora.
Tambin se invita al lector a revisar la bibliografa, en la que encontrar una gran cantidad de otros ejemplos.
La vida media de una sustancia radiactiva es el tiempo necesario para que se desintegre la mitad de los
tomos de una cantidad inicial de la sustancia en cuestin. Es sabido que la tasa de desintegracin de una
sustancia radiactiva es proporcional a la cantidad de sustancia en cada instante.
Ejemplo 40 Inicialmente haba 100 miligramos de una sustancia radiactiva. Despus de 6 horas la masa
disminuy en un 3%. Si la rapidez de desintegracin es, en un instante cualquiera, proporcional a la cantidad
de sustancia en dicho instante, hallar la cantidad que queda despus de 24 horas.
41

Sea y (t) la cantidad de dicha sustancia en el instante t. Como la rapidez de desintegracin y 0 (t) es, en un
instante cualquiera, proporcional a la cantidad y (t) de sustancia en dicho instante, se tiene
y 0 (t) = ky (t) ,
para alguna constante de proporcionalidad k, de donde
y (t) = Cekt , C R.
La cantidad inicial de sustancia es de 100 = y (0) = Ce0k = C miligramos.
Luego de 6 horas, la cantidad de sustancia disminuy una tres por ciento, de manera que 97 = y (6) = 100e6k ,
y entonces

1
97
k = ln
.
6
100
Consecuentemente

y (t) = 100
Ahora bien, despus de 24 horas quedarn:
y (24) = 100

97
100

24
6

97
100

6t

= 100

97
100

es decir, despus de 24 horas quedarn aproximadamente 88,5 miligramos de sustancia radiactiva.


Ejemplo 41 La cantidad de bacterias en un cultivo crece, en un instante cualquiera, con una rapidez proporcional al nmero de bacterias en dicho instante. Despus de tres horas se observa que hay 400 bacterias, y al
cabo de 10 horas, hay 2.000. Cul es el nmero inicial de bacterias?.
Sea y (t), el nmero de bacterias presentes en el cultivo en el instante t. Como en un instante cualquiera
y 0 (t) -la rapidez con que crece la poblacin de bacterias- es proporcional al nmero de elllas, se puede escribir
y 0 (t) = ky (t) .
Esta es una ecuacin lineal de primer orden, la que tiene soluciones de la forma
y (t) = Cekt .
Para responder a la pregunta formulada, es necesario determinar los valores de las cosntantes C y k. Para
esto, se recurre a la informacin que aporta el enunciado del problema. Si el tiempo se mide en horas, entonces
se puede afirmar que
Ce3k = y (3) = 400
y
Ce10k = y (10) = 2000.
De donde, dividiendo miembro a miembro,
e7k

5, k =
3

C = 5 7
42

1
ln (5)
7
400

y entonces el nmero de bacterias en el instante t est determinado, aproximadamente, por


3

y (t) = 5 7 400 5 7 = 400 5

t3
7

(por qu?). Entonces, el nmero inicial de bacterias es, aproximadamente,


3

y (0) = 400 5 7 200.6787642.


La ley de enfriamiento de Newton afirma que en un cuerpo que se est enfriando, la rapidez con que la
temperatura T (t) cambia es proporcional a la diferencia entre la temperatura del cuerpo y la temperatura
constante del medio que lo rodea. Por supuesto este enunciado presume que el medio que rodea al cuerpo en
cuestin es suficientemente grande para que su temperatura no se vea afectada de manera perceptible.
Ejemplo 42 Un termmetro que est en el interior de una habitacin se lleva al exterior, en donde la temperatura del aire es de 15 grados celsius. Despus de un minuto el termmetro marca 13 grados celsius, y
despus de 5 minutos marca 1 grado celsius. Cul es la temperatura de la habitacin al momento de sacar el
termmetro al exterior?
Denote la temperatura de termmetro, medida en grados celsius, en el tiempo t, medido en minutos, por
T (t). Se puede suponer que el tiempo 0 corresponde al momento en que el termmetro es transportado al
exterior, de manera que la temperatura de la habitacin est dada por el valor de T (t) en t = 0. Como la
temperatura del aire fuera de la habitacin es de 15 grados celsius (temperatura constante del medio que
rodea al termmetro), se puede afirmar -aplicando la ley de enfriamiento de Newton- que la temperatura T (t)
del termmetro satisface la ecuacin lineal de primer orden
dT
(t) = k [T (t) (15)] ,
dt
o, identicamente
1
dT kdt = 0,
T + 15
ecuacin de variables separables, con solucin
ln |T + 15| kt = c, c R,
despejando T , se tiene
|T + 15| = Cekt , C R.

(7.1)

Ahora, como la temperatura del termmetro no puede ser en ningn caso menor que la temperatura del
medio ambiente en el que se enfra, entonces T (t) 15, de manera que T +15 0 y entonces |T + 15| = T +15.
Reemplzanado esto en 7.1 se tiene
T + 15 = Cekt ; C R,
de donde
T (t) = Cekt 15, C R.
Para que esta funcin represente la temperatura del termmetro en las condiciones dadas en el problema, es
necesario determinar los valores de las constantes C y k, de manera que T (1) = 13 y T (5) = 1, esto es
Cek 15 = 13
Ce5k 15 = 1
43

o bien

Cek = 28 (1)
Ce5k = 14 (2)

despejando C en (1) y reemplzando en (2), se tiene


e4k =

14
1
=
28
2



1
luego k = 14 ln 12 = ln 4 2 y por lo tanto C
= 28 C = 28 4 2 y la temperatura de termmetro en el
4
2
tiempo t est dada por
t+1
T (t) = 28 2 4
De manera que la temperatura de la habitacin al momento de transportar el termmetro fuera de ella es,
aproximadamente:

4
T (0) = 28 2 33.29779922 grados celsius.
Ejemplo 43 Un termmetro se saca de una habitacin, en donde la temperatura del aire es de 21 grados
celsius, al exterior donde la temperatura es de 12 grados celsius. Despus de medio minuto el termmetro
marca 16 grados celsius. Cunto marca el termmetro despus de estar un minuto a la intemperie?Cuanto
tiempo demorar el termmetro en alcanzar los 10 grados celsius?.
Sea T (t) la temperatura en grados celcius del termmetro en el instante t medido en segundos. Supondremos
que el instante t = 0 es el instante en que el termmetro es llevado al exterior. Siendo as, T (0) = 21. La
temperatura del medio exterior es 12 grados celcius, de manera que la ley de enfriamiento de Newton aplicada
a nuestro termmetro nos da
T 0 = k(T (12)),
o lo que es lo mismo
dT
= k dt,
T + 12
de donde
ln |T + 12| = kt + c,
y como en el ejemplo anterior
T (t) = Cekt 12.

(7.2)
0k

Ahora bien, como 21 = T (0), reemplazando en (1.43) resulta 21 = Ce 12, y entonces C = 33. Por otra
parte, como a los treinta segundos la temperatura del termmetro
es 16 grados celcius, reemplazando en (7.2),

28
1
se tiene 16 = 33 e30k 12, de manera que k =
ln , y por lo tanto la temperatura del termmetro en el
30 33
instante t est dada por
1
28
T (t) = 33e 30 ln| 33 |t 12,
o bien
T (t) = 33

28
33

30t

12.

60
2
30
Finalmente T (60) = 33 28
12 = 33 28
12 11, 8, de manera que la temperatura del termmetro un
33
33
minuto despues de expuesto a la intemperie es T (60) 11, 8 grados celcius. Para contestar la segunda pregunta
44

hacemos 10 = T (t) = 33

28 30t
33

12 y despejamos t. Obtenemos

segundos, esto es, aproximadamente 8, 6 minutos.

2
=
33

28
33

30t

de donde t = 30

ln

33
ln| 28
33 |

513

Ejemplo 44 Se disuelven inicialmente 25 kilos de sal en un tanque que contiene 1200 litros de agua. Se bombea
salmuera al tanque a razn de 12 litros por minuto; y luego la solucin, adecuadamente mezclada, se bombea
fuera del tanque tambien a razn de 12 litros por minuto. Si la concentracin de la solucin que entra es de
0, 25 kilos de sal por litro, determine la cantidad de sal que hay en el tanque en un instante cualquiera. Cunta
sal hay en el tanque despues de 50 minutos?, Cunta sal queda en el tanque despues de un perodo prolongado
de tiempo?.
Sea y(t) la cantidad de kilos de sal que se encuentran en el tanque en el instante t, medido en minutos.
Considere un pequeo intervalo de tiempo (t0 , t1 ). Qu ocurre con la solucin salina en el tanque durante este
intervalo de tiempo?. Evidentemente la solucin salina en el tanque se est modificando, ya que por un lado
entra sal al tanque y por otro sale del tanque. Observe que durante el mencionado intervalo de tiempo entran al
tanque 12 t litros de salmuera, que llevan consigo 0, 25 12 t kilos de sal al interior del tanque. Por otra parte,
en el mismo intervalo de tiempo, salen del tanque 12 t litros de solucin, los que llevan consigo...cierta cantidad
de sal. Cmo determinar esta cantidad de sal que sale del tanque?. Imagine que se tiene algn instrumento que
permite medir la cantidad de sal en el tanque. Ciertamente, cualquiera que sea el diseo de este instrumento,
este tendr ciertas limitaciones: Por muy preciso que resulte ser, no podr detectar las variaciones en la cantidad
de sal en el tanque si t es arbitrariamente pequeo, de manera que desde un punto de vista prctico, se puede
asumir que para intervalos de tiempo muy pequeos, la cantidad y(t) de sal en el tanque, t (t0 , t1 ), permanece
constante. Para hacer este razonamiento mas aceptable, suponga que en intervalos de tiempo muy pequeos,
la cantidad de sal en el tanque permanece practicamente constante. Aceptado este supuesto, se puede afirmar
y(t)
que durante el intervalo de tiempo (t0 , t1 ) la concentracin de sal en el tanque es 1200
, donde t es cualquier
tiempo en el intervalo (t0 , t1 ). De esta discusin se concluye que: durante el intervalo de tiempo (t0 , t1 ), salen
y(t)
kilos de sal. Ahora bien, la variacin de la cantidad de sal en el tanque durante el intervalo
del tanque 12 t
1200
(t0 , t1 ) es exactamente la cantidad de sal que entra durante este intervalo de tiempo menos la cantidad de sal
que sale del tanque durante este intervalo. En trminos matemticos:
y(t1 ) y(t0 ) = 0, 25 12 t 12

y(t)
t , t (t0 , t1 ) ,
1200

dividiendo por t,

considerando lim

t1 t0

y(t1 )y(t0 )
t1 t0

y(t)
y(t1 ) y(t0 )
=3
,
t1 t0
100

t (t0 , t1 ) ,

y observando que t (t0 , t1 ) se obtiene finalmente la ecuacin diferencial:


y 0 (t0 ) = 3

1
y(t0 )
100

y como t0 es arbitrario, se puede escribir la ecuacin en trminos de la variable t :


y 0 (t) = 3

45

1
y(t) .
100

Para responder las preguntas


formuladas en el problema, se debe resolver la ecuacin. Considere el factor de
R dx
x
integracin (x) = e 100 = e 100 . Multiplicando la ecuacin por este factor, se obtiene
x
x
d
(e 100 y(t)) = 3 e 100 ,
dx

integrando
de manera que

e 100 y(t) = 300 e 100 + c , c R


x

y(t) = 300 + ce 100 , c R

Inicialmente el tanque contiene 25 kilos de sal, de modo que

25 = y(0) = 300 + ce 100 = 300 + c ,


por lo tanto c = 275, y la cantidad de sal en el tanque esta determinada, en el instante t, por
x

y(t) = 300 275 e 100 .


50

Consecuentemente, despues de 50 minutos, el tanque contendra y(50) = 300 275 e 100 = 300 275 e 2 128, 1
x
kilos de sal, y como lim e 100 = 0, se concluye que luego de un perodo prolongado de tiempo quedarn en el
x
tanque aproximadamente 300 kilos de sal, ... siempre que la salmuera que entra al tanque no se agote antes!!!,
esta informacin no es conocida por nuestra ecuacin, asi que ella no puede advertirnos de este hecho prctico.
Ejemplo 45 Determine la forma de un espejo curvado de tal manera que la luz de una fuente situada en el
orgen se refleja en l como un haz de rayos paralelos al eje x.
Suponga que la superficie del espejo est determinada por la rotacin de una seccin transversal al espejo
pasando por el centro del mismo. En otras palabras, suponga que la superficie del espejo se obtiene rotando la
grfica de alguna funcin conveniente en torno a algun eje de rotacin (eje x). Sea y(x) la curva que representa
una seccin del espejo como en la figura.

Figura 14: Espejo Curvado


Sean , , y los angulos en la figura. Observe que = + , y como = resulta = + . Por
otra parte, segun la ley de reflexin = , de manera que = 2 . De la figura se observa que:
y
2 tg()
= tg() = tg(2 ) =
x
1 tg 2 ()
46

y es claro que
y 0 (x) = tg()
de donde se obtiene la ecuacin diferencial:
y
2 y 0 (x)
=
x
1 (y 0 (x))2

Lo primero que se observa es que la derivada de y aparece al cuadrado en esta ecuacin. Esto sugiere tratar
la ecuacin como una ecuacin cuadrtica en la incgnita y 0 , de manera que se intentar despejar y 0 y luego
y
determinar y. Para esto se escribe z = y 0 y u = . Con estas notaciones la ecuacin se expresa por
x
u=

2z
1 z2

o bien,
z2 +

2
z1=0 .
u

Despejando z se obtiene:
1
z=
u
y reemplazando z = y 0 y u =

y
, resulta
x

1 + u2
,
u2

v
u
y 2
u
x u1 + (x)
0
,
y = t y
y
( )2
x

de donde

x
y =
y
0

x2 + y 2
,
y2

y como se est interesado en y > 0 (Por qu?), se obtiene finalmente la ecuacin diferencial homogenea de
grado 1
p
x x2 + y 2
0
y =
.
(7.3)
y
Note que en realidad se trata de dos ecuaciones, cada una de ellas determinada por el signo asignado al radical
en el lado derecho de la ecuacin. Para resolver esta ecuacin considere y = w x, de manera que y 0 = w0 x + w
y al reemplazar en la ecuacin anterior se obtiene
p
x x2 + (w x)2
w0 x + w =
,
wx
de donde

x |x| 1 + w2
w x+w =
.
wx
0

47

Cancelando x en el lado derecho, resulta:


1
|x|
w x+w =
w
x
0

1 + w2
.
w

Antes de continuar, observe que la presencia de la variable x en el lado derecho de esta ltima ecuacin es
|x|
superflua ya que
= 1. Restando w a ambos lados resulta
x

1 + w2
|x| 1 + w2

,
w0 x =
w
x
w
o, lo que es igual,

1 + w2
(1 + w2 ) |x|
dw
x
x=
dx
w

y separando las variables


w
(1 +

w2 )

|x|
1
x

w2

dw =

1
dx .
x

(7.4)

Si v = 1 + w2 entonces dv = 2w dw de manera que


R

w dw
(1 +

w2 )

|x|
1
x

w2

1
2

dv

v |x|
v
x
R
dv
=

2 v( v |x|
x )

|x|
=
ln | v x |

=
ln | 1 + w2 |x|
x |
|x|
(recuerde que x = 1)

de manera que integrando directamente en la ecuacin (7.4) se obtiene


p

|x|
2

ln 1 + w
= ln |x| + c
x

y "exponenciando"

1 + w2

= C |x| , C = ec > 0.

|x|
x

y
en esta
Relacin que determina a la solucin de la ecuacin (7.4) en forma implcita. Reemplazando w =
x
ecuacin se obtiene
1
r
= C |x|

y2

|x|
1 + 2 x

x
o bien

1 p
2
2

|x| x + y
48

= C |x| ,

|x|
x

y como

|x|
x
=
se puede escribir
x
|x|

|x|
p
= C |x|

2
x + y 2 x

de donde la solucin de la ecuacin (7.3) est dada en forma implcita por


1
p
=C ,

2
x + y 2 x

Ahora bien, como se ha comentado, la ecuacin (7.3) corresponde a dos ecuaciones. Si el signo del radical en la
ecuacin (1.44) es + la solucin est dada por
1
p
= C , (Por qu?).

2
x + y 2 x

(7.5)

p
x2 + y 2 x. Se
Para poder eliminar las barras del valor absolutoen (7.4)
p y despejar y, considere el signopde
2
2
2
2
observa que si x R {0} , entonces x |x| = x
x + y , y consecuentemente
x + y 2 x 0, de
manera que (7.5) se puede escribir como
a=
y por lo tanto

x2 + y 2 x , a =

a+x=
elevando al cuadrado y despejando y 2 se obtiene

1
>0,
C

x2 + y 2 ,

y 2 = 2ax + a2 .
Se deja de ejercicio analizar que ocurre si en la ecuacin (7.3) se considera el signo negativo para el radical del
lado derecho. (Indicacin: si x < 0, entonces x < 0 < x = |x|). Compare la solucin que obtiene en este caso
con la anterior. En qu consiste la diferencia entre ambas soluciones desde un punto de vista geomtrico?.
Ejemplo 46 Suponga que un aeroplano despega desde el punto (a, 0) situado al este de su destino, un aeropuerto
localizado en el orgen (0, 0). El aeroplano vuela con rapidez constante v0 relativa al viento, el cual esta soplando
hacia el norte con velocidad constante. Supuesto que durante todo el vuelo el piloto mantiene la nave en direccin
hacia el origen, determine la trayectoria del avin.
Denote Va.r.t. la velocidad del avin relativa al terreno, Va.r.v. la velocidad del avin relativa al viento, y
Vv. la velocidad del viento (relativa al terreno), entonces es claro que Va.r.t = Vv + Va.r.v. . Como la velocidad
del viento es constante en la direccin norte, entonces Vv = (0, w). Denote (t) = (x(t), y(t)) la posicin
del avin respecto al sistema de coordenadas con orgen en el aeropuerto de destino y con punto de despegue
en el punto de coordenadas (a, 0). Sea (t) el ngulo que forma el vector velocidad Va.r.t respecto al eje x.
Entonces (x0 (t), y 0 (t)) = Va.r.t. (t) = Vv + Va.r.v. = (0, w) + Va.r.v. y entonces x0 (t) = k Va.r.t. (t) k cos((t)) e
y 0 (t) = k Va.r.t. (t) k sen( (t)) +w y como k Va.r.t. (t) k= v0 se obtiene
x0 (t) = v0 cos((t)) e y 0 (t) = v0 sen( (t)) + w
49

w
resulta la ecuacin homogenea
v0

y dividiendo y 0 (t) por x0 (t) y anotando k =

dy
p
y
x2 + y 2
dy
dt
= k
=
.
dx
dx
x
x
dt
Sea y = z x, entonces

p
x2 + (z x)2
dy
zx
dz
=
=
k
z+x
dx
dx
x
x
y como en este problema x > 0 (por qu?), cancelando resulta
x

separando las variables

p
dz
= k 1 + z 2 ,
dx

dz
dx

+
=0,
x
k 1 + z2
integrando

esto es

dz

+
k 1 + z2

dx
=c
x

p
1

ln z + 1 + z 2 + ln |x| = c
k

k1
p

=c
ln x z + 1 + z

o bien

"exponenciando" y anotando C = ec resulta

y como z =

k1
p
x z + 1 + z2
=C

y
, reemplazando se obtiene
x
x

o tambin
x

y
+
x

y
+
x

1+

y 2

1+

! k1

=C

=C

y 2
x

Finalmente, como el avin despega del punto con coordenadas (a, 0) = (0) = (x(0), y(0)), se observa que
x(0) = a e y(0) = 0, y reemplazando estos valores en la ltima ecuacin se obtiene

s
2
0
0
=C
ak + 1 +
a
a
50

, esto es C = ak , y la trayectoria del avin queda determinada en forma implcita por la ecuacin:

w
w
p
1
x v0
y + x2 + y 2 = a v0 .

Es posible que el avin se quede sin combustible antes de llegar al aeropuerto?, En cules condiciones
p esto
sera inevitable?, Puede obtener su conclusin a partir de esta ltima ecuacin? (Recuerde que x2 + y 2
representa la distancia del avin al aeropuerto (origen de coordenadas), y observe que para que el avin se
acerque al aeropuerto es necesario que la componente x se aproxime a 0).
Ejemplo 47 Suponga que un cable cuelga suspendido de sus extremos, sometido a la accin de su propio peso.
Si la densidad lineal del cable es constante igual w, detemine la forma que asume el cable.
Ubique el eje Y pasando por el punto ms bajo de la cadena, como es observa en la figura:

Figura 15: Cable colgante


Denote por s la longitud de arco desde ese punto a uno variable (x, y) y por w (s) la densidad lineal de la
cadena. Se obtiene la ecuacin de la curva del hecho de que la porcin de la cadena entre el punto ms bajo y
(x, y) est en equilibrio bajo la accin de tres fuerzas: la tensin horizontal T0 en el punto ms bajo; la tensin
variable T en (x, y), que acta a lo largo de la tangente debido a la flexibilidad de la cadena; y una fuerza hacia
abajo igual al peso de la porcin de la cadena entre esos dos puntos. Igualando la componente hortizontal de T
y T0 y la vertical de T al peso resulta:
T cos () = T0 ,
Z s
T sen () =
w (s) ds
0

Se deduce de la primera de estas ecuaciones que:


T sen () = T0 tan () = T0
as que:
0

T0 y =

51

w (s) ds

dy
,
dx

Se elimina la integral derivando con respecto a x:


Z s
Z s
d
d
ds
T0 y 00 =
w (s) ds =
w (s) ds
dx 0
ds 0
dx
q
2
= w (s) 1 + (y 0 )
Por consiguiente:

T0 y 00 = w (s)

q
2
1 + (y 0 )

(7.6)

es la ecuacin diferencial de la curva pedida, y sta se halla resolviendo la ecuacin. Para continuar se necesita
informacin adicional sobre la funcin w (s) . Se resuelve (7.6) para el caso en que w (s) es una constante w0 ,
de modo que:
q
w0
2
(7.7)
y 00 = a 1 + (y 0 ) , a =
T0
Sustituyendo y 0 = p e y 00 =

dp
dx ,

la ecuacin (7.7) se reduce a:


dp

1 + p2

= adx

(7.8)

Ahora se integra (7.8) y se utiliza el hecho de que p = 0 cuando x = 0 para llegar a

ln p + 1 + p2 = ax
despejando p,

dy
1 ax
=
e eax .
dx
2
Si se ubica el eje x a la altura adecuada para que y = a1 cuando x = 0 se tiene
p=

y=

1
1 ax
e + eax = cosh (ax)
2a
a

como la ecuacin de la curva que adopta una cadena uniforme flexible bajo su propio peso. Esta curva se llama
catenaria.

7.1

Ejercicios

1. Un cuerpo de masa m que va cayendo encuentra una resistencia del aire proporcional a su velocidad
instantanea v(t) . Determine la posicin del cuerpo en el instante t .
2. Supongase que un cable cuelga suspendido de sus extremos, sometido a la accin de su propio peso.
Determine la funcin cuya grfica representa la forma del cable colgante.(Vea SIMMONS, ECUACIONES
DIFERENCIALES con aplicaciones y notas histricas
3. Una persona M que parte del origen, se mueve en la direccin positiva del eje x arrastrando un peso a
lo largo de la curva C. El peso, ubicado inicialmente sobre el eje y en el punto de coordenadas (o, s), es
arrastrado por medio de una cuerda de longitud constante s, que se mantiene tensa en el transcurso del
movimiento. Determine la curva C. (Vea SIMMONS, ECUACIONES DIFERENCIALES con aplicaciones y notas histricas
52

4. Hallar la forma de un espejo curvado de tal manera que la luz de una fuente situada en el orgen se refleja
en l como un haz de rayos paralelos al eje x. (Vea SIMMONS, ECUACIONES DIFERENCIALES con
aplicaciones y notas histricas, Captulo 1, Factores integrantes).
5. Un depsito contiene 40 galones de agua pura. Salmuera con 3 libras de sal por galn fluye en l a razn
de 2 galones por minuto, y la mezcla escapa a razn de 3 galones por minuto.
a) Calcular la cantidad de sal en el depsito cuando ya solo quedan en l 20 galones.
b) Cundo es mxima la cantidad de sal en el depsito?
6. Un cultivo tiene inicialmente una cantidad N0 de bacterias. Una hora ms tarde se observa un aumento
de 50% en el nmero de bacterias del cultivo. Si la rapidez con que crece el cultivo es proporcional al
nmero de bacterias presentes, determine el tiempo necesario para que el nmero de bacterias se triplique.
7. Un magnate posee una fortuna que crece a un ritmo proporcional al cuadrado de su valor en cada momento.
Si tena 10 millones de dlares hace un ao y hoy tiene 20 millones Cul ser su fortuna dentro de 6
meses?, y dentro de un ao?.
8. Cul curva situada por encima del eje x tiene la propiedad que la longitud del arco que une dos puntos
cualesquiera sobre ella es proporcional al area bajo dicho arco?
9. Un conejo parte del orgen y corre por el eje y positivo con velocidad a. Al mismo tiempo, un perro que
corre con rapidez b sale del punto de coordenadas (c, 0) y persigue al conejo. Cul es la trayectoria que
sigue el perro?.(Vea SIMMONS, ECUACIONES DIFERENCIALES con aplicaciones y notas histricas,
Captulo 2, Cadena colgante, curvas de persecucin)
10. El eje y y la recta x = c son las orillas de un ro cuya corriente fluye con una velocidad uniforme a en la
direccin negativa del eje y. Una barca entra en el ro por el punto de coordenadas (c, 0) y desplazandose
con rapidez b relativa al agua se dirige a la orilla opuesta. Cul es la trayectoria que seguir la barca
si durante todo el trayecto la proa permanece orientada hacia el orgen de coordenadas?. Es claro que
el destino de la barca depende de la relacin entre a y b. Determine condiciones para llegar a la orilla
opuesta. Es posible que la barca llegue al punto correspondiente al orgen de coordenadas?. (La misma
referencia del problema anterior)
11. Cierta informacin dudosa relativa a los efectos de la feniletilamina en el agua potable comenz a extenderse
un da en una ciudad de 100.000 habitantes. Despus de una semana 10.000 personas haba odo el rumor.
Suponga que la razn de aumento del nmero de las que han odo el rumor es proporcional al nmero de
los que an no escuchan el rumor. Cunto tiempo pasar hasta que la mitad de la ciudad haya escuchado
el rumor?
12. La ley de Torricelli se puede enunciar de la siguiente forma: la velocidad de cada de una gota de agua
es proporcional al cuociente entre la raz cuadrada de la altura sobre el rea de la seccin transversal

horizontal del tanque. La constante de proporcionalidad est dada por la expresin: k = a 2g. Un
tanque tiene la forma de un cilindro vertical; inicialmente contiene agua pura con una profundidad de 9
pies y un tapn en el fondo es retirado en el momento t = 0 (horas). Despus de 1 hora la profundidad
del agua ha descendido a 4 pies. Cunto tiempo tardar el agua en salir por completo?

53

13. Un tanque de 400 galones contiene inicialmente 100 galones de salmuera, la cual contiene a su vez 50 libras
de sal. Entra salmuera, cuya concentracin es de 1 libra de sal por galn a razn de 5 galones por segundo,
y la salmuera mezclada en el tanque se derrama a razn de 3 galones por segundo. Qu cantidad de sal
contendr el tanque cuando est lleno de salmuera?

Ecuaciones Diferenciales Lineales de Orden


Superior.
Ecuaciones lineales de 2o orden.

Definicin 1 La ecuacin lineal de 2o orden es la ecuacin


y 00 (x) + p (x) y 0 (x) + q (x) y (x) = r (x) ,

(1.1)

Teorema 1 Sean p (x) , q (x) y r (x) funciones contnuas en un intervalo cerrado [a, b]. Dado x0 [a, b], y0
e y1 R, existe una y slo una solucin y (x) de (1.1) definida para todo x [a, b], tal que y (x0 ) = y0 e
y 0 (x0 ) = y1 .
Este teorema afirma que la ecuacin (1.1) siempre tiene solucin, ms an afirma que el P.V.I.

y 00 (x) + p (x) y 0 (x) + q (x) y (x) = r (x)


y (x0 ) = y0

y 0 (x0 ) = y1

tiene exactamente una solucin.

1.1

Ecuaciones lineales de 2o orden homogneas.

Antes de intentar resolver la ecuacin (1.1), considere la ecuacin homognea:


y 00 (x) + p (x) y 0 (x) + q (x) y (x) = 0
Proposicin 1 Si y1 (x) e y2 (x) son soluciones de (1.2) y R entonces
i) (y1 + y2 ) (x) = y1 (x) + y2 (x) son soluciones de (1.2)
ii) (y1 ) (x) = y1 (x) es solucin de (.2)
En efecto:
i) (y1 + y2 )00 (x) + p (x) (y1 + y2 )0 (x) + q (x) (y1 + y2 ) (x) = 0 + 0
ii) (y1 )00 (x) + p (x) (y1 )0 (x) + q (x) (y1 ) (x) = 0
Considere
Sh ([a, b]) = {y : [a, b] R | y es solucin de (1.2)}
La proposicin anterior muestra que Sh ([a, b]) es un espacio vectorial.
54

(1.2)

Proposicin 2 Sh ([a, b]) con la suma habitual de funciones y la multiplicacin de un escalar real por una
funcin es un espacio vectorial de dimensin 2.
Esto significa que Sh ([a, b]) tiene una base formada por dos elementos, en particular si {y1 (x) , y2 (x)} es
una base de Sh ([a, b]), entonces para cada y Sh ([a, b]) existen escalares C1 , C2 R, tales que:
y (x) = C1 y1 (x) + C2 y2 (x)
Conclusion: si se desea determinar las soluciones de la ecuacin homognea
y 00 (x) + p (x) y 0 (x) + q (x) y (x) = 0
es suficiente determinar dos soluciones y1 (x) e y2 (x) de esta ecuacin que sean linealmente independientes.
Definicin 2 Si y1 , y2 : [a, b] R son funciones diferenciables, el Wronskiano de y1 , y2 , W [y1 , y2 ], es

y1 (x) y2 (x)
W [y1 (x) , y2 (x)] = det
y10 (x) y20 (x)
W (x) = W [y1 (x) , y2 (x)] = y1 (x) y20 (x) y2 (x) y10 (x)

Proposicin 3 (Frmula de Abel)


Si y1 (x) e y2 (x) son solucin de la ecuacin lineal homognea de 2o orden
y 00 (x) + p (x) y 0 (x) + q (x) y (x) = 0
entonces
W [y1 (x) , y2 (x)] = W (x) = Ce

p(x)dx

(Formula de Abel)

Corolario 2 Sean y1 (x) , y2 (x) soluciones de la ecuacin homognea (2.2), entonces


i) {y1 (x) , y2 (x)} es linealmente independiente si y slo si W [y1 (x) , y2 (x)] 6= 0, x [a, b]
ii) {y1 (x) , y2 (x)} es linealmente dependiente si y slo si W [y1 (x) , y2 (x)] = 0, x [a, b]
Ejemplo 1 Muestre que y (x) = C1 sen (x) + C2 cos (x) es la solucin general de la ecuacin y 00 + y = 0.
i. Primero verificamos que son soluciones de la ecuacin:
y1 (x) = sen (x) , y2 (x) = cos (x)
y10 (x) = cos (x) , y20 (x) = sen (x)
y100 (x) = sen (x) , y200 (x) = cos (x)
{z
}
{z
}
|
|
y100 + y1 = sen (x) + sen (x) = 0, y200 + y2 = cos (x) + cos (x) = 0
ii. Ahora verificamos que {y1 (x) , y2 (x)} es linelamente independiente

sen (x) cos (x)


W [y1 (x) , y2 (x)] = det
= sen2 (x) cos2 (x) = 1 6= 0 x
cos (x) sen (x)

{y1 (x) , y2 (x)} es linealmente independiente.


55

1.1.1

Empleo de una solucin conocida y1 de y 00 (x) + p (x) y 0 (x) + q (x) y (x) = 0 para determinar y2
solucin linealmente independiente.

Suponga y1 es una solucin conocida de y 00 (x) + p (x) y 0 (x) + q (x) y (x) = 0, Observe que si y2 es cualquier
solucin de la ecuacin homognea (1.2), entonces, segn la frmula de Abel aplicada a {y1 (x) , y2 (x)}, y2
satisface la ecuacin lineal de 1er orden:
y1 (x) y20 (x) y2 (x) y10 (x) = e
o bien
y20

p(x)dx

y 0 (x)
e p(x)dx
(x) 1
y2 (x) =
y1 (x)
y1 (x)

esta ecuacin puede ser resuelta, pues es de 1er orden


Ejemplo 2 Determine la solucin general de la ecuacin lineal homognea de 2o orden
x2 y 00 + xy 0 y = 0, y1 (x) = x.
Sean
y1 (x) = x, y10 (x) = 1, y100 (x) = 0
reemplazando en la ecuacin se tiene
x2 0 + x 1 x = 0
Para resolver la ecuacin en primer lugar se debe normalizar la ecuacin:
y 00 +

1 0
1
y 2y = 0
x
x

Sea y2 (x) otra solucin de la ecuacin que es linealmente independiente con y1 , entonces, segn la frmula de
Abel:
W [y1 (x) , y2 (x)] = e
y1 y20 y10 y2
xy20 y2
y20
R

1
y2
x

= e

R
R

p(x)dx
1
x dx

= e ln|x| =
=

1
x

1
x2

determinando el factor integrante: (x) = e x dx = e ln|x| = x1

1
d 1
, integrando
=
y2
dx x
x3
1
1
y2 = x2 + C, despejando y2
x
2
1
y2 (x) =
+ Cx
2x

56

Observe que:
y (x) = C1 y1 (x) + C2 y2 (x)

1
= C1 x + C2
+ Cx
2x

C2 1
y (x) = (C1 + C) x +
.
2 x
Luego, y1 (x) = x e y2 (x) = x1 . Por lo tanto

1
Sh ([a, b]) = C1 x + C2 | C1 , C2 R
x

1.2

Ejercicios

1. Determine cules de los siguientes conjuntos de funciones son linealmente dependientes y cules son linealmente independientes. En cada caso use la defnicin directa y los teoremas que involucran al Wronskiano.
(a) e4x , e4x
(b) 2x3 , 3x3
(c) 1, cos(x)

(d) x2 , x2 + 1 , x2 1

(e) (x + 1)(x 2) , (2x 1)(x + 3) , (x + 2)(x 1)


(f) sen2 (x), cos2 (x) , 2

(g) sen(x) + cos(x) , 3sen(x) 2 cos(x), 4 cos(x)


2. Determine si los siguientes conjuntos de funciones son linealmente independientes en - < x <
(a) {y1 (x) = x, y2 (x) = x2 , y3 (x) = 4x 3x3 }

(b) {y1 (x) = 0, y2 (x) = x, y3 (x) = ex }

(c) y1 (x) = 5, y2 (x) = cos2 (x), y3 (x) = sen2 (x)


(d) {y1 (x) = cos(x), y2 (x) = 1, y3 (x) = cos2 (x) }

3. En los siguientes problemas se da una solucin de la ecuacin propuesta. Determine otra solucin que sea
linealmente independiente de la solucin dada.
(a) x2 y 00 7xy 0 + 16y = 0 , y1 = x4

(b) xy 00 + y 0 = 0 , y1 = ln(x)

(c) 4x2 y 00 + y = 0, y1 = x1/2 ln(x)


(d) y 00 3 tg(x) y 0 = 0 , y1 = 1

(e) (1 2x x2 )y 00 + 2(1 + x)y 0 2y = 0 , y1 = x + 1 .

4. Determinar la solucin general:


57

(a) y 00 + y = 0, y1 (x) = sen (x) ,


(b) y 00 y = 0, y1 (x) = ex ,

(c) x y 00 + 3y 0 = 0, solucin evidente y1 (x) = 1,

(d) 1 x2 y 00 2xy 0 + 2y = 0, y1 (x) = x

Ecuacin lineal homognea de 2o orden con coeficientes constantes.

La ecuacin lineal homognea con coeficientes constantes es la ecuacin:


y 00 (x) + p (x) y 0 (x) + q (x) y (x) = 0
donde p (x) = p, q (x) = q son constantes

(2.1)

Para resolver esta ecuacin, se necesita encontrar por lo menos una solucin.
Como las derivadas de y = emx son mltiplos de y existe alguna posibilidad de determinar m de manera que
y = emx sea solucin de la ecuacin en cuestin.
Sea y1 (x) = emx y suponga que es solucin de la ecuacin (2.1), entonces
m2 emx + pmemx + qemx
2

m + pm + q emx

= 0
= 0

por lo tanto si m2 + pm + q = 0, entonces y1 (x) = emx ser solucin de (2.1). Cul es el valor de m?.
p
p + p2 4q
,
m1 =
p2
p p2 4q
m2 =
2
Luego se tienen tres posibilidades:
I. Races reales y distintas.
II. Races reales e iguales.
III. Races complejas.
I. p2 4q > 0; m1 6= m2 ; m2 + pm + q = (m m1 ) (m m2 ) y1 (x) = em1 x , y2 (x) = em2 x son ambas
soluciones de (2.1), adems se verifica que W [y1 (x) , y2 (x)] 6= 0. Por lo tanto {y1 (x) , y2 (x)} es un
conjunto linealmente independiente.
Conclusin:
{y (x) = C1 em1 x + C2 em2 x | C1 , C2 R} es la solucin general de la ecuacin (2.1)
Ejemplo 3 Resolver
y 00 (x) 5y 0 (x) + 6y (x) = 0

58

Aqu, p = 5, q = 6. Considere el polinomio asociado


m2 5m + 6
cuyas races son

51
25 24
m =
=
2
2
m1 = 2, m2 = 3
m2 5m + 6 = (m 2) (m 3)
5

Solucin de la ecuacin:
Ejemplo 4 Resolver:

y (x) = C1 e2x + C2 e3x | C1 , C2 R


y 00 (x) y 0 (x) 2y (x) = 0

Considere las races del polinomio asociado


m2 m 2 = (m + 1) (m 2)
Solucin general:

y (x) = C1 ex + C2 e2x | C1 , C2 R

II. p2 4q = 0; m1 = m2 = p2 , m2 + pm + q = (m m1 )2 y1 (x) = e 2 x es solucin de (2.1).


Para determinar y2 (x) una solucin linealmente independiente de (2.1) con y1 (x), considere la frmula de
Abel:
R

W [y1 (x) , y2 (x)] = Ce pdx


p
p p
e 2 x y20 (x) + e 2 x y2 (x) = Cepx
2
p
p
y20 (x) + y2 (x) = Ce 2 x
2
el factor de integracin: (x) = e

p
2 dx

= e2x

p
p
d px
= Ce 2 x e 2 x , integrando
e 2 y2 (x)
dx
p
e 2 x y2 (x) = Cx
p

y2 (x) = Cxe 2 x
como interesa encontrar una solucin de la ecuacin entonces, considerando C = 1, se tiene
p

y2 (x) = xe 2 x
Como m1 = p2 se concluye que la solucin general de la ecuacin (2.1) es, en este caso:
{y (x) = C1 em1 x + C2 xem1 x | C1 , C2 R}
59

Ejemplo 5 Resolver
y 00 (x) + 4y 0 (x) + 4y (x) = 0
Se observa que el polinomio asociado es
m2 + 4m + 4 = (m + 2)

luego la raz: m1 = m2 = 2 = p2 , por lo tanto, la solucin general de la ecuacin dada es:

y (x) = C1 e2x + C2 xe2x | C1 , C2 R

4qp2
III. p2 4q < 0; m1 = p2 +
i = a bi, m2 = a + bi, luego
2

y, segn la frmula de Euler:

ye1 (x) = em1 x = e(abi)x = eax ebxi


ye2 (x) = em2 x = e(a+bi)x = eax ebxi
ei = cos () + isen ()

se obtiene:
ye1 (x) = eax cos (bx) ieax sen (bx)
ye2 (x) = eax cos (bx) + ieax sen (bx)

pero estas expresiones involucran nmeros complejos. Considere las combinaciones


y1 (x) =
y2 (x) =

1
(e
y1 (x) + ye2 (x)) = eax cos (bx)
2
i
(e
y1 (x) ye2 (x)) = eax sen (bx) .
2

Se verifica fcilemente que y1 (x) e y2 (x) son soluciones de (2.1) (Ejercicio) (basta observar que si m1 =
a bi, m2 = a + bi, entonces
m2 + pm + q

= (m m1 ) (m m2 )
= m2 (m1 + m2 ) m + m1 m2

= m2 2am + a2 + b2

de modo que la ecuacin (2.1) se expresa por

y 00 (x) 2ay 0 (x) + a2 + b2 y (x) = 0

y al reemplazar y1 (x) e y2 (x) en esta ecuacin se verifica la igualdad en cada caso.)


Adems
W [y1 (x) , y2 (x)] 6= 0 (Ejercicio)
Conclusin: la solucin general de (2.1) es, en este caso,
{y (x) = C1 eax cos (bx) + C2 eax sen (bx) | C1 , C2 R}
60

Ejemplo 6 Resolver la ecuacin


y 00 (x) 4y 0 (x) 8y (x) = 0
Polinomio asociado:
m2 4m + 8
Races:
m=

16 32
= 2 2i
2

Solucin general de la ecuacin:


y (x) = C1 e2x cos (2x) + C2 e2x sen (2x) , C1 , C2 R
Ejemplo 7 Resolver
y 00 (x) + 2y 0 (x) + 5y (x) = 0
Polinomio asociado:
m2 + 2m + 5
Races:
m=

4 20
= 1 2i
2

Solucin general:
y (x) = C1 ex cos (2x) + C2 ex sen (2x) , C1 , C2 R

2.1

Ejercicios

1. Determine la solucin general de las siguientes ecuaciones:


(a) y 00 + 5y 0 + 6y = 0
(b) y 00 + 8y 0 + 16y = 0
(c) z 00 + z 0 z = 0

(d) y 00 + 9y = 0

(e) z 00 6z 0 + 10z = 0

(f) w00 + 4w0 + 6w = 0

2. Resuelva los siguientes P.V.I.


(a) y 00 + 2y 0 8y = 0; y (0) = 3, y 0 (0) = 12

(b) y 00 + 2y 0 + y = 0; y (0) = 1, y 0 (0) = 3

(c) w00 4w0 + 2w = 0; w (0) = 0, w0 (0) = 1

(d) y 00 2y 0 + 2y = 0; y () = e , y 0 () = 0.

61

Ecuacin lineal de 2o orden no homognea con coeficientes constantes

La ecuacin lineal de 2o orden no homognea con coeficientes constantes es la ecuacin


y 00 (x) + py 0 (x) + qy (x) = r (x)

(3.1)

Proposicin 4 Sea {y1 (x) , y2 (x)} una base del espacio solucin de la ecuacin homognea
y 00 (x) + py 0 (x) + qy (x) = 0
e yp (x) una solucin particular de la ecuacin (3.1).
Entonces la solucin general de (3.1) es:
y (x) = yp (x) + C1 y1 (x) + C2 y2 (x) ; C1 , C2 R
en otras palabras, y (x) es solucin de (3.1) si y slo si y (x) = yp (x) + yh (x) , donde yh (x) es solucin de la
ecuacin homognea (2.1).
Justificacin:
) si y (x) = yp (x) + yh (x) es inmediato que y (x) es solucin de (2.5)
) si y (x) es solucin de (3.1), entonces y (x) = yp (x) + (y (x) yp (x))
{z
}
|

solucin de la homognea

Segn esta proposicin, para resolver (3.1) slo se necesita conocer una solucin particular de (3.1).

3.1

Mtodo de Variacin de Parmetros.

Suponga que y1 (x) e y2 (x) son soluciones de la ecuacin homognea


y 00 (x) + p (x) y 0 (x) + q (x) y (x) = 0
(note que los coeficientes no son necesariamente constantes).
Es posible preguntarse si existe alguna solucin yp (x) de la ecuacin
y 00 (x) + p (x) y 0 (x) + q (x) y (x) = r (x)
que sea de la forma:
yp (x) = u1 (x) y1 (x) + u2 (x) y2 (x) .
Supongamos que existe una solucin de esta forma, entonces se tiene que:

y1 (x) u01 (x) + y2 (x) u02 (x) = 0


, x
y10 (x) u01 (x) + y20 (x) u02 (x) = r (x)

62

(3.2)

de donde se necesita encontrar u1 y u2 :

r (x)

y1 (x)
0
y1 (x)

y1 (x)
0
y (x)
1
y1 (x)
0
y1 (x)

u01 (x) =

u02 (x) =

y2 (x)
y20 (x)
y2 (x) r (x)
=

W [y1 (x) , y2 (x)]


y2 (x)
y20 (x)

0
r (x)
y1 (x) r (x)
=
W [y1 (x) , y2 (x)]
y2 (x)
y20 (x)

y, entonces, la solucin particular yp (x) de (3.2) es:


Z
Z
y2 (x) r (x)
y1 (x) r (x)
dx + y2 (x)
dx
yp (x) = y1 (x)
W [y1 (x) , y2 (x)]
W [y1 (x) , y2 (x)]
Observacin: Si se exige que yp (x) sea solucin del P.V.I.:
y 00 (x) + p (x) y 0 (x) + q (x) y (x) = r (x)
y (x0 ) = 0
y 0 (x0 ) = 0
entonces
yp (x) =

x0

y2 (x) y1 (t) y1 (x) y2 (t)


W [y1 (t) , y2 (t)]

r (t) dt

Ejemplo 8 Resolver
y 00 + 3y 0 + 2y =

1
1 + ex

i. Se determina la solucin general de la ecuacin homognea: y 00 + 3y 0 + 2y = 0.


m2 + 3m + 2 = (m + 2) (m + 1) .
Solucin general de la homognea:
yh (x) = C1 e2x + C2 ex
ii. Como y1 (x) = e2x e y2 (x) = ex son soluciones linealmente independientes de la homognea, se determina
una solucin particular yp (x) de la forma:
yp

= u1 (x) y1 (x) + u2 (x) y2 (x)


= u1 (x) e2x + u2 (x) ex

segn el anlisis anterior, se utiliza la condicin


y1 u01 + y2 u02

= 0

y10 u01 + y20 u02

63

1
1 + ex

despejando u01 , u02 :

u01

(x) =

u01 (x) =
v

u02 (x) =

La solucin particular:

ex
ex

1
e
ex (1+e
1+e
x)
x

= 3x =
1
e
ex
e3x
ex
Z
e2x
e2x

,
u
(x)
=
dx
1
1 + ex
1 + ex
Z
Z
e2x
vdv
x
x
e ; dv = e dx
dx =
= (v ln |1 + v|)
x
1+e
1+v
u1 (x) = ex + ln |1 + ex |

e2x
0

1
2e2x

ex
1+ex

=
u2 (x) = ln |1 + ex |
e2x
1 + ex
ex

2e2x ex
1

1+e
e2x

2e2x

yp
yp

= (ex + ln (1 + ex )) e2x + (ln (1 + ex )) ex

= ex ex + 1 ln (1 + ex ) ex

La solucin general de la ecuacin dada: y = yp + yh

y (x) = ex ex + 1 ln (1 + ex ) ex + C1 e2x + C2 ex
o simplemente:

Ejemplo 9 Resolver

y (x) = ex ex + 1 ln (1 + ex ) + C1 e2x + C2 ex
y 00 + y = csc (x) , csc (x) =

1
sen (x)

i. Solucin de la ecuacin homognea asociada:


y 00 + y = 0
polinomio asociado:
m2 + 1
races:
m1 = 0 i, m2 = 0 + i
Solucin de la ecuacin homognea
y1 (x) = cos (x) , y2 (x) = sen (x)
yh = C1 cos (x) + C2 sen (x)

64

ii. Se supone que existe la solucin particular:


yp (x) = u1 (x) y1 (x) + u2 (x) y2 (x)
se impone la condicin:
y1 (x) u01 (x) + y2 (x) u02 (x) = 0
y10 (x) u01 (x) + y20 (x) u02 (x) =

1
sen (x)

entonces:

u01 (x)

u02 (x)

0
1

sen(x)
cos (x)

sen (x)

cos (x)

sen (x)

cos (x)

sen (x)

sen (x)
cos (x)

1
=
u1 (x) = x

1
sen (x)
cos (x)

cos (x)
sen(x)
=
u2 (x) = ln |sen (x)|
sen (x)
sen (x)
cos (x)

yp (x) = x cos (x) + sen (x) ln |sen (x)|


Solucin general de la ecuacin dada: y (x) = yp (x) + yh (x)

y (x) = (C1 x) cos (x) + (C2 + ln |sen (x)|) sen (x)

3.2

Ecuaciones lineales de 2o orden con coeficientes constantes, otro punto de


vista. (Mtodo del Anulador o Coeficientes Indeterminados)

Definicin 3 Dado el siguiente conjunto de funciones:


C ([a, b]) = {y : [a, b] R | y es diferenciable}
Si y C ([a, b]), entonces podemos definir D (y) = y 0 como la aplicacin
D

C ([a, b]) C ([a, b])


y 7 D (y) = y 0
El conjunto (C ([a, b]) , +, ) es un Espacio Vectorial y D es una funcin lineal.
Proposicin 5 Dada la aplicacin D : C ([a, b]) C ([a, b]) , D (y) = y 0 , tiene las siguientes propiedades:
i. La compuesta D D
D D : C ([a, b]) C ([a, b])
(D D) (y) = D (D (y)) = D (y 0 ) = y 00
Como D es lineal, D D tambin es lineal. Notacin: D2 = D D
65

ii. Si m1 es una constante, entonces se puede considerar la funcin


m

C ([a, b]) 1 C ([a, b])


y 7 m1 y
esta funcin tambin es lineal.
iii. Si se suman funciones lineales se obtiene una funcin lineal:
D + m : C ([a, b]) C ([a, b])
(D + m) (y) = D (y) + m y = y 0 + my
Considere la ecuacin lineal de 1er orden
y 0 + 2y = 0
entonces, se puede reescribir esta funcin como
(D + 2) (y) = 0
Observe que el operador D + 2, tiene Kernel, cuya definicin es:
ker (D + 2) = {y C ([a, b]) | D (y) + 2y = y 0 + 2y = 0}
Luego, el determinar las funciones que estn en ese conjunto es resolver la ecuacin diferencial:
1 0
y = 2 y (x) = Ce2x
y
2x

ker (D + 2) =
Ce
|CR
luego, podemos generalizar este hecho particular con la siguiente proposicin:
Proposicin 6 Dada la ecuacin de primer orden
y 0 m1 y = 0, m1 constante
esta ecuacin se puede reescribir como
(D m1 ) (y) = 0,

cuya solucin est dada por

ker (D m1 ) = {Cem1 x | C R}

Luego, si L1 , L2 : C ([a, b]) C ([a, b]) son operadores lineales, entonces se pueden componer
L1 L2 : C ([a, b]) C ([a, b])
y L1 L2 es lineal. As, si L1 = D + m1 ; L2 = D + m2 , se tiene
(L1 L2 ) (y) =
=
=
=
=
=
=
=

L1 (L2 (y))
(D + m1 ) (L2 (y))
D (L2 (y)) + m1 L2 (y)
D ((D + m2 ) (y)) + m1 ((D + m2 ) (y))
D (D (y) + m2 y) + m1 (D (y) + m2 y)
D2 (y) + m2 D (y) + m1 D (y) + m1 m2 y
D2 (y) + (m1 + m2 ) D (y) + m1 m2 y
2

D + (m1 + m2 ) D + m1 m2 (y)
66

que es la forma general de la ecuacin diferencial de 2o orden con coeficientes constantes.


Note que las soluciones de la ecuacin
y 00 + (m1 + m2 ) y 0 + m1 m2 y = 0
constituyen el Kernel del operador lineal

2
D + (m1 + m2 ) D + m1 m2 = L1 L2 , esto es

{y (x) | y 00 (x) + (m1 + m2 ) y 0 (x) + m1 m2 y (x) = 0} = ker D2 + (m1 + m2 ) D + m1 m2


= ker (L1 L2 )
Proposicin 7 Si L2 (y2 ) 0, entonces (L1 L2 ) (y2 ) = 0
Consecuencia:
Si y2 satisface L2 (y2 ) = 0, entonces y2 ker (L1 L2 ) y por lo tanto y2 es solucin de y 00 + (m1 + m2 ) y 0 +
m1 m2 y = 0, como L2 = D + m2 entonces y2 = em2 x es solucin de L2 (y) = 0 y por lo tanto y2 = em2 x es
solucin de y 00 + (m1 + m2 ) y 0 + m1 m2 y = 0. Cmo se puede determinar otra solucin?.
Como L1 L2 = L2 L1 , en efecto:
(L2 L1 ) (y) =
=
=
=
=
=
=

L2 (L1 (y))
D (L1 (y)) + m2 L1 (y)
D (D (y) + m1 y) + m2 (D (y) + m1 y)
D2 (y) + m1 D (y) + m2 D (y) + m2 m1 y
D2 (y) + (m2 + m1 ) D (y) + m2 m1 y
2

D + (m2 + m1 ) D + m2 m1 (y)
(L1 L2 ) (y)

Pero, entonces, si y1 es solucin de L1 (y) = 0, entonces y1 es solucin de (L2 L1 ) (y) = 0, esto es y1 es solucin
de
2

D + (m1 + m2 ) D + m1 m2 (y) = 0

Como L1 = D + m1 , entonces y1 (x) = em1 x ker (L1 ) y por lo tanto: y1 ker (L1 L2 ) = ker (L2 L1 ) =
{y | y 00 + (m1 + m2 ) y 0 + m1 m2 y = 0}, de esta manera se han obtenido las soluciones linealmente independientes
y1 (x) = em1 x , y2 (x) = em2 x

Luego, se ha mostrado otra tcnica para resolver ecuaciones lineales homogneas.


Ejemplo 10 Resolver
y 00 + 9y 0 + 18y = 0
Considere
y se verifica fcilmente que

D + 9D + 18 y = 0

D2 + 9D + 18 = (D + 6) (D + 3)
Composici
on

67

Se resuelven las ecuaciones


(D + 6) y = 0, (D + 3) y = 0
se obtiene
y1 (x) = e6x , y2 = e3x
y la solucin general es
Ejemplo 11 Resolver

y (x) | y (x) = C1 e6x + C2 e3x , C1 , C2 R


2

D + 4D + 4 y = 0

Note que

D2 + 4D + 4 = (D + 2) (D + 2) = (D + 2)

Se resuelve (D + 2) y = 0 , obteniendo y1 (x) = e2x , y segn el mtodo antes estudiado y2 (x) = xe2x . Por lo
tanto la solucin general es

y (x) = C1 e2x + C2 xe2x | C1 , C2 R

Ejemplo 12 Resolver

D2 y + y = 0
En esta ecuacin no es posible "factorizar" D2 + 1 es "factores" lineales de orden 1 con coeficientes
constantes reales. Esta ecuacin es simplemente
y 00 + y = 0
y se resuelve segn el mtodo presentado anteriormente:
y1 (x) = cos (x) , y2 (x) = sen (x)
y la solucin general es
{y (x) = C1 cos (x) + C2 sen (x) | C1 , C2 R}
Problema: Resolver la ecuacin
y 00 + 5y 0 + 6y = ex
i. Solucin de la ecuacin homognea

yh = C1 e2x + C2 e3x | C1 , C2 R

ii. Determinar una solucin particular de la ecuacin.

Desde el nuevo punto de vista se puede escribir la ecuacin como


2

D + 5D + 6 y = ex
o tambin

(D + 2) (D + 3) y = ex
Dos observaciones:
68

a. ex es solucin de (D + 1) y = 0
b. Si yp (x) es solucin de (D + 2) (D + 3) y = ex , entonces

(D + 1) ((D + 2) (D + 3)) (yp ) = (D + 1) ex = 0

y como los coeficientes son constantes, tambin se puede escribir


((D + 2) (D + 3)) (D + 1) y
si (D + 1) y

= 0, ahora bien
= 0 (D + 2) (D + 3) (D + 1) y = 0.

Esto permite realizar la siguiente pregunta: existir una solucin yp de la ecuacin dada que satisfaga
(D + 1) (yp ) = 0?.
Si la respuesta es afirmativa, entonces se puede determinar una solucin particular yp (x) de la forma
yp (x) = Cex
(El operador D + 1, se denomina anulador de ex ).
Para verificar esto, se reemplaza yp propuesto en la ecuacin dada:

2
D + 5D + 6 Cex =
Cex 5Cex + 6Cex =
(C 5C + 6C) ex =
2Cex =
entonces, si C =

1
2

ex
ex
ex
ex

se determina una solucin particular


yp (x) =

1 x
e
2

Entonces la solucin general de la ecuacin es:


y (x) =

1 x
e + C1 e2x + C2 e3x , C1 , C2 R
2

Ejemplo 13 Resolver
y 00 + 2y 0 3y = cos (x)
La ecuacin es
(D + 3) (D 1) y = cos (x)
i. La solucin de la ecuacin homognea es
yh (x) = C1 e3x + C2 ex , C1 , C2 R
ii. Se determina (si es posible) un anulador de cos (x), esto es alguna expresin lineal (con coeficientes constantes) L tal que L (cos (x)) = 0.
69

Recuerde que cos (x) es solucin de la ecuacin

2
D +1 y =0

Como en el ejemplo anterior, se observa que si yp es solucin de la ecuacin


(D + 3) (D 1) y = cos (x)
entonces

D2 + 1 (D + 3) (D 1) yp = D2 + 1 (cos (x)) = 0

Existir una solucin particular yp que sea solucin de D2 + 1 yp = 0?.

Si esto ocurre, entonces yp (x) = A1 cos (x) + A2 sen (x) .

Para determinar los coeficientes A1 y A2 se reemplaza yp en la ecuacin:


(D + 3) (D 1) (A1 cos (x) + A2 sen (x))
(D + 3) (A1 sen (x) + A2 cos (x) A1 cos (x) A2 sen (x))
(A2 A1 ) sen (x) (A1 + A2 ) cos (x) + 3 (A2 A1 ) cos (x) 3 (A1 + A2 ) sen (x)
(4A2 2A1 ) sen (x) + (2A2 4A1 ) cos (x)
esta ltima igualdad implica el siguiente sistema de ecuaciones
2A2 4A1
4A2 2A1

= 1
= 0

cuyas soluciones son:


1
1
A1 = , A2 =
5
10
por lo tanto
1
1
yp = cos (x) + sen (x)
5
10
La solucin general de la ecuacin dada es:
y (x) = yp + yh
1
1
y (x) =
sen (x) cos (x) + C1 e3x + C2 ex , C1 , C2 R.
10
5
Ejemplo 14 Resolver la ecuacin
3y 00 6y 0 + 6y = ex + ex
i. Solucin homognea
3y 00 6y 0 + 6y
2

3 D 2D + D y

= 0
= 0

como m2 2m + 2 = (m (1 + i)) (m (1 i)) se obtiene


yh = C1 ex cos (x) + C2 ex sen (x)
70

=
=
=
=

cos (x)
cos (x)
cos (x)
cos (x)

ii. Anulador de ex + ex .
Como (D 1) ex = 0 y (D + 1) ex = 0, se observa que (D 1) (D + 1) = D2 1 anula a ex + ex . El
razonamiento es idntico al de los ejemplos anteriores:
Si yp es solucin de la ecuacin, entonces

D2 1 ex + ex

= D2 1 3D2 6D + 2 yp

= 3D2 6D + 2 D2 1 yp

Existe una solucin particular yp que tambin es solucin de D2 1 yp = 0?


0 =

Si existe, entonces

yp = Aex + Bex
reemplazamos en la ecuacin:

3 Aex Bex 6 Aex Bex + 2 Aex + Bex


3Aex + 3Bex 6Aex + 6Bex + 2Aex + 2Bex
(3A 6A + 2A) ex + (3B + 6B + 2B)
Aex + 11Bex

=
=
=
=

ex + ex
ex + ex
ex + ex
ex + ex

esto implica
A = 1, 11B = 1
1
A = 1, B =
11
Luego, la solucin particular es
yp = ex +

1 x
e
11

Por lo tanto la solucin general es


1 x
e ex + C1 ex cos (x) + C2 ex sen (x)
11
1
y (x) = (C1 cos (x) + C2 sen (x) 1) ex + ex , C1 , C2 R.
11

y (x) =

Ejemplo 15 Resolver
y 00 + y = xex
i. Solucin homognea
yh = C1 cos (x) + C2 sen (x)
ii. Anulador de xex : (D 1)2 (xex ) = 0.

71

Es necesario encontrar una solucin particular yp de la ecuacin que sea solucin de (D 1) y = 0. Se observa
que yp (x) = Aex + Bxex , reemplazando en la ecuacin se tiene
(Aex + Bex + Bxex )0 + Aex + Bxex
Aex + Bex + Bex + Bxex + Aex + Bxex
2 (A + B) ex + 2Bxex

= xex
= xex
= xex

luego
A+B
2B
entonces

= 0
= 1

B=

1
1
, A=
2
2

yp =

1
(xex ex )
2

Por lo tanto

Por lo tanto la solucin general es


y (x) =

3.3

1
(x 1) ex + C1 cos (x) + C2 sen (x) , C1 , C2 R.
2

Ejercicios

1. Mediante el mtodo de variacin de parmetros resuelva las siguientes ecuaciones diferenciales.


(a) y 00 + y = sec(x)
(b) y 00 + y = sec(x) tg(x)
(c) y 00 4y = x ex , sujeta a las condiciones y(0) = 1, y 0 (0) = 0

(d) 2y 00 + y 0 y = x + 1 , sujeta a las ondiciones y(0) = 1, y 0 (0) = 0


(e) y 00 4y 0 + 4y = (12 x2 6x) e2x

2. Resuelva las ecuaciones siguientes:


(a) (D2 + 1)y = e2x ,
(b) (xD + 1)(D 2)y = sen(x),
(c) (D 1)(D + 1)y = ex ,

(d) (D + 4)(xD 1)y = x.


3. Mediante el mtodo de los coeficientes indeterminados resuelva las siguientes ecuaciones.
(a) y 9y = 54

(b) y 00 + 4y 0 + 4y = 2x + 6
(c) y 00 + 3y 0 = 4x 5
72

(d) y 00 + 4y = 4 cos(x) + 3 sen(x) 8


(e) y 00 + 4y = cos2 (x)

(f) y 00 + y 0 + 14 y = ex (sen(3x) cos(3x))

(g) y 00 + 2y 0 + y = x2 ex

4. Muestre que la sustitucin x = et , transforma la Ecuacin de Cauchy - Euler an xn y (n) +an1 xn1 y (n1) +
... + a1 xy (1) + a0 y = g(x) en una ecuacin lineal con coeficientes constantes (verifique esta afirmacin por
lo menos para n = 2 y 3 ,. Para esto considere z = y(x) = y(et ) , y derive z respecto a t para obtener
dy
dy2
dz
dz
t dz 2
2t
+ dx
, etc. ...
dt = dx e , dt2 = dx2 e
5. Resuelva las ecuaciones x2 y 00 + xy 0 + y = ln(x) , x3 y 000 + 3x2 y 00 + xy 0 = 0

Ecuaciones Lineales con coeficientes constantes de orden mayor


que dos.

Definicin 4 Las ecuaciones lineales con coeficientes constantes de orden mayor que dos son ecuaciones de la
forma:
y (n) (x) + an1 y (n1) (x) + + a1 y 0 (x) + a0 y (x) = r (x) ,
(4.1)
o en otras palabras

n
D + an1 Dn1 + + a1 D + a0 (y (x)) = r (x)

donde a0 , a1 , . . . , an1 son constantes.


La ecuacin homognea asociada es:

n
D + an1 Dn1 + + a1 D + a0 (y (x)) = 0

Teorema 3 El conjunto de todas las soluciones de la ecuacin lineal homognea de orden n

n
D + an1 Dn1 + + a1 D + a0 (y (x)) = 0
es un espacio vectorial de dimensin n

Observacin 1 El conjunto de todas las soluciones no es otra cosa que

ker Dn + an1 Dn1 + + a1 D + a0


y por lo tanto es un espacio vectorial.

Definicin 5 Sean y1 (x) , y2 (x) , . . . , yn (x) las n soluciones de (4.1), entonces el Wronskiano se define

y1 (x)
y2 (x)
y3 (x)
y4 (x)
...
yn (x)
0
0
0
y10 (x)
y
(x)
y
(x)
y
(x)
.
.
.
yn0 (x)
2
3
4

00
00
00
y100 (x)
y2 (x)
y3 (x)
y4 (x)
...
yn00 (x)

000
000
000
W [y1 (x) , y2 (x) , . . . , yn (x)] = det y1000 (x)
y2 (x)
y3 (x)
y4 (x)
...
yn000 (x)

..
..
..
..
..

..

.
.
.
.
.
.
(n1)
(n1)
(n1)
(n1)
(n1)
(x) y2
(x) y3
(x) y4
(x) . . . yn
(x)
y1
73

como:

Teorema 4 Sea L = Dn + an1 Dn1 + a1 D + a0 y suponga que L = Ln1 1 Lnk k , donde cada Li es
un polinomio de la forma D mi D2 2aD + a2 + b2 donde mi , a bi son ceros del polinomio asociado
mn + an1 mn1 + + a1 x + a0 .

Una base del espacio solucin de la ecuacin Dn + an1 Dn1 + a1 D + a0 y = 0, est determinada por
la reunin de las bases de los espacios solucin de las ecuaciones Ln1 1 y = 0, Ln2 2 y = 0, . . . , Lnk k y = 0. Donde:

i. El conjunto em1 x , xem1 x , x2 em1 x , . . . , xk1 em1 x es una base del espacio solucin de la ecuacin homognea
k

(D m1 ) y = 0

y por lo tanto la solucin general de esta ecuacin es


y (x) = C1 em1 x + C2 xem1 x + C3 x2 em1 x + + Ck xk1 em1 x =
ii. El conjunto

k
X
i=1

Ci xi1 em1 x , Ci R, i {1, 2, . . . , k}

ax
e cos (bx) , xeax cos (bx) , x2 eax cos (bx) , . . . , xk1 eax cos (bx),
eax sen (bx) , xeax sen (bx) , x2 eax sen (bx) , . . . , xk1 eax sen (bx)

es una base para el espacio solucin de la ecuacin homognea

k
D 2aD + a2 + b2
y=0

Recuerde del lgebra que todo polinomio se factoriza como producto de potencias de polinomios de grado 1
(determinados por las races reales) y/ grado 2 (determinados por las races complejas).
Ejemplo 16 Resolver la ecuacin
y 000 + 4y 00 + y 0 6y = 0
Solucin: La estrategia es factorizar
D3 + 4D2 + D 6
Considere las races del polinomio asociado
m3 + 4m2 + m 6
se puede observbar que m = 1 es una raz del polinomio
: m 1 = m2 + 5m + 6
m3 +4m2 + m 6
3
2
m m
2
0 + 5m
m 6
+
5m2 5m
0 + 6m 6
por lo tanto

m3 + 4m2 + m 6 = (m 1) m2 + 5m + 6
= (m 1) (m + 2) (m + 3)
74

Entonces se puede factorizar


D3 + 4D2 + D 6 = (D 1) (D + 2) (D + 3)
y entonces la ecuacin puede escribirse de cualquiera de las tres formas siguientes:
(D 1) (D + 2) (D + 3) y
(D + 2) (D + 3) (D 1) y
(D + 3) (D 1) (D + 2) y

= 0
= 0
= 0

y se determinan las soluciones de la ecuacin como soluciones de


(D + 3) y
(D 1) y
(D + 2) y

= 0 y1 (x) = e3x
= 0 y2 (x) = ex
= 0 y3 (x) = e2x

se verifica fcilmente que y1 (x) , y2 (x) , y3 (x) son linealmente independientes.


Conclusin: la solucin de la ecuacin
y 000 + 4y 00 + y 0 6y = 0
es
y (x) = C1 e3x + C2 ex + C3 e2x , C1 , C2 , C3 R
Ejemplo 17 Resolver la ecuacin
y (4) + 4y (3) + 10y (2) + 12y (1) + 5y = 0
Factorizar el polinomio asociado
m4 + 4m3 + 10m2 + 12m + 5
se observa que 1 es un cero de este polinomio

4
3
2
: m + 1 = m3 + 3m2 + 7m + 5
m 4+ 4m3 + 10m + 12m + 5
m +m
3
2
3m
+ 10m + 12m + 5
3m3 + 3m2
2
7m
12m +
5
+
7m2 + 7m
5m + 5

y por lo tanto

m4 + 4m3 + 10m2 + 12m + 5 = (m + 1) m3 + 3m2 + 7m + 5

note que 1 es un cero de m3 + 3m2 + 7m + 5,

3
2
: m + 1 = m2 + 2m + 5
m3 + 3m
+ 7m + 5
2
m +m
2
5
2m 2+ 7m +
2m + 2m
5m + 5
(5m + 5)
0

75

se obtiene finalmente

m4 + 4m3 + 10m2 + 12m + 5 = (m + 1)2 m2 + 2m + 5

y entonces la ecuacin se escribe:

(D + 1)2 D2 + 2D + 5 y
2

D + 2D + 5 (D + 1)2 y

= 0, o tambin
= 0

Se determinan las soluciones como soluciones de las ecuaciones:

2
D + 2D + 5 y
(D + 1)2

4 20
= 1 2i
2
y1 (x) = ex cos (2x) , y2 (x) = ex sen (x)
2

0 m2 + 2m + 5 = 0 m =

0 y3 (x) = ex , y4 (x) = xex

Entonces y1 (x) , y2 (x) , y3 (x) , y4 (x) son soluciones de la ecuacin (2.10).


Se verifica fcilmente que W [y1 (x) , y2 (x) , y3 (x) , y4 (x)] 6= 0, aqu

y1 y2 y3 y4
y10
y20
y30
y40

W = det
00
00
00
y1 y2 y3 y400
y1000 y2000 y3000 y4000

Como el espacio solucin de la ecuacin (2.10) tiene dimensin 4, entonces la solucin general de (2.10) es
y (x) = C1 ex cos (x) + C2 ex sen (x) + C3 ex + C4 xex , C1 , C2 , C3 , C4 R.

Ejemplo 18 Resolver (D + 2)3 y = 0.


2x

e
, xe2x , x2 e2x es una base del espacio solucin la solucin general es
y (x) = C1 e2x + C2 xe2x + C3 x2 e2x , C1 , C2 , C3 R

Ejemplo 19 Resolver D5 y = 0
Se puede escribir (D 0)5 y = 0 y entonces una base para el espacio solucin es
0x

e = 1, xe0x = x, x2 e0x = x2 , x3 e0x = x3 , x4 e0x = x4

o, equivalentemente

1, x, x2 , x3 , x4

y la solucin general es

y (x) = C1 + C2 x + C3 x2 + C4 x3 + C5 x4 , C1 , C2 , C3 , C4 , C5 R.
Ejemplo 20 Resolver

(D 2) (D + 1) (D 4D + 8) y = 0
76

La solucin es
y (x) = C1 e2x + C2 xe2x + C3 x2 e2x + C4 ex + C5 xex + C6 e2x cos (2x) + C7 xe2x cos (2x) +
= + C8 x2 e2x cos (2x) + C9 e2x sen (2x) + C10 xe2x sen (2x) + C11 x2 e2x sen (2x) , C1 , . . . , C11 R

Ejemplo 21 Resolver D6 2D5 + 3D4 4D3 + 3D2 2D + 1 y = 0

Observe que m1 = 1, a bi = 0 i son races del polinomio asociado: m6 2m5 + 3m4 4m3 + 3m2 2m + 1.
Se puede factorizar el polinomio asociado:
6
4
3
2
2m5 + 3m
m
4m + 3m 2m + 1
6
4
+m
m
5

2m
+ 2m4 4m3 + 3m2 2m + 1

5
2m3
2m
4
3
2
2m4 2m + 3m
2m + 1
2
+ 2m
2m
3

2m
+ 3m2 2m + 1

3
2m
2m
2
m + 1
m2 + 1
0

: m2 + 1 = m4 2m3 + 2m2 2m + 1

se obtiene

m6 2m5 + 3m4 4m3 + 3m2 2m + 1 = m2 + 1 m4 2m3 + 2m2 2m + 1


4
3
2
2m
+ 2m 2m + 1
m
4
3
m m
3
2
m + 2m 2m + 1
m3 + m2
2
+1
m2 2m
m m
m+1
(m + 1)
0

: m 1 = m3 m2 + m 1

luego
2

m + 1 (m 1) m3 m2 + m 1

= m2 + 1 (m 1) m2 (m 1) + (m 1)
2

= m2 + 1 (m 1)2

m6 2m5 + 3m4 4m3 + 3m2 2m + 1 =

La ecuacin se expresa como L21 L22 y = 0 donde L1 = D 1, L2 = D2 + 1


Se resuelve por separado:
L21 y
L22 y

= 0; (D 1)2 y = 0 {y1 (x) = ex , y2 (x) = xex } es base del espacio solucin.

2
= 0; D2 + 1 y = 0 {y3 (x) = cos (x) , y4 (x) = x cos (x) , y5 (x) = sen (x) , y6 (x) = xsen (x)}
77

Segn el teorema anterior, el conjunto


{y1 (x) = ex , y2 (x) = xex , y3 (x) = cos (x) , y4 (x) = x cos (x) , y5 (x) = sen (x) , y6 (x) = xsen (x)}
es una base del espacio solucin de la ecuacin dada. La solucin general de la ecuacin es:
y (x) = C1 ex + C2 xex + C3 cos (x) + C4 x cos (x) + C5 sen (x) + C6 xsen (x)
o
y (x) = (C1 + C2 x) ex + (C3 + C4 x) cos (x) + (C5 + C6 x) sen (x)
Considere nuevamente L = Dn + an1 Dn1 + a1 D + a0 , al igual que en el caso de ecuaciones de orden 2, la
solucin general de la ecuacin Ly = r (x) se obtiene como y (x) = yp (x) + yh (x) , donde yh (x) es la solucin
general de la ecuacin homognea Ly = 0 e yp (x) es alguna solucin de la ecuacin Ly = r (x) .
Ejemplo 22 Resolver la ecuacin
2

D 2D + 1 y = ex , L = D 1

Se puede escribir

(D 1) y = ex
i. La solucin de la homognea
yh (x) = C1 ex + C2 xex
ii. Para encontrar yp (x), se emplea el mtodo de los coeficientes indeterminados:
Si yp (x) es solucin de Ly = ex , entonces yp (x) es solucin de (D 1)3 y = 0, entonces
yp (x) =

C1 ex + C2 xex + C3 x2 ex
|
{z
}

solucin de Ly=0

reemplazando

yp (x) = C3 x2 ex

yp = Ax2 ex

yp0 (x) = A2xex + Ax2 ex = A 2x + x2 ex

yp00 (x) = A (2 + 2x) ex + A 2x + x2 ex = A 2 + 4x + x2 ex

A 2 + 4x + x2 ex 2A 2x + x2 ex + Ax2 ex

A 2 + 4x + x2 4x 2x2 + x2 ex

yp (x) = 12 x2 ex es una solucin particular.

78

= ex
= ex 2C3 = 1 C3 =

1
2

La solucin general es:


1 2 x
x e + C1 ex + C2 xex
2

1
=
C1 + C2 x + x2 ex , C1 , C2 R.
2

y (x) =

Ejemplo 23 Resolver la ecuacin:

6
D 2D5 + 3D4 4D3 + 3D2 2D + 1 y = ex
Segn se ha visto en el ejemplo 76, la ecuacin puede expresarse como
2

(D 1)2 D2 + 1 y = ex

i. La solucin de la ecuacin homognea se ha determinado en el ejemplo 76.


ii. Para determinar una solucin particular yp (x) se puede emplear el mtodo de los coeficientes indeterminados:
Como ex se anula en D 1, entonces cualquier solucin yp de Ly = ex es solucin de
2
2
(D 1) (D 1) D2 + 1 y = 0
2

Las soluciones de D2 + 1 y = 0 se descartan, pues cualquier solucin de esta ecuacin es solucin de


Ly = 0.
3

Las soluciones de (D 1) y = 0 son yp = C1 ex + C2 xex + C3 x2 ex


|
{z
}
soluciones de Ly=0

yp (x) = C3 x e .
2 x

yp = Ax2 ex

yp0 (x) = A2xex + Ax2 ex = A 2x + x2 ex

yp00 (x) = A (2 + 2x) ex + A 2x + x2 ex = A 2 + 4x + x2 ex

yp000 (x) = A (4 + 2x) ex + A 2 + 4x + x2 ex = A 6 + 6x + x2 ex

yp(4) (x) = A (6 + 2x) ex + A 6 + 6x + x2 ex = A 12 + 8x + x2 ex

yp(5) (x) = A (8 + 2x) ex + A 12 + 8x + x2 ex = A 20 + 10x + x2 ex

yp(6) (x) = A (10 + 2x) ex + A 20 + 10x + x2 ex = A 30 + 12x + x2 ex

reemplazando:

A 30 + 12x + x2 ex 2A 20 + 10x + x2 ex + 3A 12 + 8x + x2 ex 4A 6 + 6x + x2 ex + ...

... + 3A 2 + 4x + x2 ex 2A 2x + x2 ex + Ax2 ex

A 30 + 12x + x2 2 20 + 10x + x2 + 3 12 + 8x + x2 4 6 + 6x + x2 + ...

... + 3 2 + 4x + x2 2 2x + x2 + x2 ex

A 30 + 12x + x2 40 20x 2x2 + 36 + 24x + 3x2 24 24x ...

... 4x2 + 6 + 12x + 3x2 4x 2x2 + x2 ex


A8ex
79

=
= ex
=
= ex
=
= ex
= ex

entonces: 8A = 1 A =

1
8

Por lo tanto yp = 18 x2 ex es la solucin particular de la ecuacin.


La solucin general es:
1 2 x
x e + C1 ex + C2 xex + C3 cos (x) + C4 x cos (x) + C5 sen (x) + C6 xsen (x)
8

x2
=
C1 + C2 x +
ex + C3 cos (x) + C4 x cos (x) + C5 sen (x) + C6 xsen (x) , C1 , . . . , C6 R.
8

y (x) =

Ejemplo 24 Resolver
y (5) y (4) + 2y 000 2y 00 + y 0 y = cos (x)
Se puede escribir
4

D D + 2D2 + 1 D4 + 2D2 + 1 y
2

(D 1) D2 + 1 y

= cos (x)
= cos (x)

i. La solucin de la ecuacin homognea es:

y (x) = C1 ex + C2 cos (x) + C3 x cos (x) + C4 sen (x) + C5 xsen (x) , C1 , ..., C5 R.
ii. Como D2 + 1 aniquila cos (x) entonces toda solucin (particular) yp (x) de la ecuacin dada debe ser solucin
de la ecuacin homognea
3

(D 1) D2 + 1 y = 0
Se concluye que existe una solucin particular de la forma

yp (x) = Ax2 cos (x) + Bx2 sen (x)


= x2 (A cos (x) + Bsen (x))
Luego:
yp (x) = x2 (A cos (x) + Bsen (x))
yp0 (x) = 2x (A cos (x) + Bsen (x)) + x2 (Asen (x) + B cos (x))
yp00 (x) = 2 (A cos (x) + Bsen (x)) + 4x (Asen (x) + B cos (x)) x2 (A cos (x) + Bsen (x))

yp000 (x) = 6 (Asen (x) + B cos (x)) 6x (A cos (x) + Bsen (x)) x2 (Asen (x) + B cos (x))

y (4) (x) = 12 (A cos (x) + Bsen (x)) 8x (Asen (x) + B cos (x)) + x2 (A cos (x) + Bsen (x))
y (5) (x) = 20 (Asen (x) + B cos (x)) + 10x (A cos (x) + Bsen (x)) + x2 (Asen (x) + B cos (x))
reemplazando en la ecuacin y simplificando se tiene:
cos (x) (8A 8B) + sen (x) (8A + 8B) = cos (x)
luego:
8A 8B
8A + 8B
80

= 1
= 0

y
A=

1
1
,B =
16
16

por lo tanto:
yp =

1 2
x (cos (x) sen (x))
16

y la solucin general es:


y=

4.1

1 2
x (cos (x) sen (x)) + C1 ex + C2 cos (x) + C3 x cos (x) + C4 sen (x) + C5 xsen (x) , C1 , ..., C5 R
16

Ejercicios:

1. Determine la solucin de las siguientes ecuaciones:


3

(a) D2 8D + 25 y = 0

4
(b) D4 + 4D3 + 6D2 + 4D + 1 y = 0,observe que la ecuacin se puede escribir como (D + 1) y = 0
(c) y 000 y = 0,

(d) y 000 + 5y 00 = 0,
(e) y 000 y 00 4y = 0

(f) y 000 6y 00 + 12y 0 8y = 0

(g)

d4 y
dx4

(h)

d5 y
dx5

(i)

d y
dx5

d3 y
dx3

dy
16 dx

d2 y
dx2

= 0,

=0

d y
d y
2 dx
4 + 17 dx3 = 0

2. Determine la solucin de los siguientes problemas de valor inicial:


(a) y 000 + 12y 00 + 36y 0 = 0, y(0) = 0, y 0 (0) = 1, y 00 (0) = 7

(b) y 000 + 2y 00 5y 0 6y = 0, y(0) = 0 = y 0 (0) , y 00 (0) = 1


(c)

d4 y
dx4

=0,

y(0) = 2, y 0 (0) = 3, y 00 (0) = 5 .

3. Para cada conjunto de funciones dado a continuacin, determine una ecuacin diferencial que contenga
dicho conjunto en su espacio solucin.
(a) {ex , cos(x) , sen(x)}

(b) {cosh(x), cos(x)}


(c) {x2 , x sen(x)}

(d) {1, sen(x) , x senh(x)}


(e) {x4 , cos(x) sen(x) }
(f) {senh(3x) , 5}

(g) {x, x2 cosh(3x)}


81

(h)

cos (x), sen2 (x)

4. Mediante el mtodo de los coeficientes indeterminados resuelva las siguientes ecuaciones.


(a) y 9y = 54

(b) y 00 + 4y 0 + 4y = 2x + 6
(c) y 00 + 3y 0 = 4x 5

(d) y 00 + 4y = 4 cos(x) + 3 sen(x) 8


(e) y 00 + 4y = cos2 (x)

(f) y 00 + y 0 + 14 y = ex (sen(3x) cos(3x))

(g) y 00 + 2y 0 + y = x2 ex

(h) y (4) 4y (2) = 5x2 e2x


(i) 16y (4) y = ex/2

(j) y (4) 5y (2) + 4y = 2 cosh(x) 6

El mtodo de Variacin de Parmetros.

Como en el caso de orden 2, si y1 (x) , ..., yn (x) son soluciones linealmente independientes de la ecuacin

n
D + an1 Dn1 + ... + a1 D + a0 y = 0
entonces existe una solucin particular de la ecuacin

n
D + an1 Dn1 + ... + a1 D + a0 y = r (x)

de la forma

yp (x) = u1 (x) y1 (x) + + un yn (x)

los coeficientes u1 (x) , ..., un (x) satisfacen:

y1 (x) u01 (x) + yn (x) u0n (x) = 0


y10 (x) u01 (x) + yn0 (x) u0n (x) = 0
..
.
(n2)

(n2)

(x) u01 (x) + yn


(x) u0n (x) = 0
y1
(n1) 0
(n1) 0
y1 (x)
u1 (x) + yn (x)
un (x) = r (x)
Ejemplo 25 Resolver

y 000 y 0 = sen (x)


i. Solucin homognea:

3
D D y
D (D + 1) (D 1) y

= 0
= 0

luego:
yh (x) = C1 1 + C2 ex + C3 ex , C1 , C2 , C3 R
82

ii. La solucin particular es:


yp (x) = u1 (x) 1 + u2 (x) ex + u3 (x) ex

Con la condicin se tiene el siguiente sistema de ecuaciones:

1 u01 (x) + ex u02 (x) + ex u03 (x) = 0


0 u01 (x) ex u02 (x) + ex u03 (x) = 0
0 u01 (x) + ex u02 (x) + ex u03 (x) = sen (x)

Para determinar u01 (x) , u02 (x) , u03 (x) :

1 ex ex
x x
= det 0 ex ex = 2
W 1, e , e
0 ex ex

0
ex ex
0
ex ex
det
sen (x) ex ex
1
u01 (x) =
= sen (x) 2 = sen (x) u01 (x) = sen (x)
2
2
u1 (x) = cos (x)

1
0
ex
0
ex
det 0
1
0 sen (x) ex
1
u02 (x) =
= ex sen (x) = ex sen (x)
2
2
2
Z
1
x
u2 (x) =
e sen (x) dx
2
Z
Z
Z
x
x
x
x
x
e sen (x) dx = e cos (x) + e cos (x) dx = e cos (x) + e sen (x) ex sen (x) dx
Z
1

ex sen (x) = ex (sen (x) cos (x))


2
1 x
u2 (x) =
e (sen (x) cos (x))
4

Z
0
1 ex
1
1
1 x
0
x

0 e
0
u3 (x) = det
= e sen (x) u3 (x) =
ex sen (x) dx
2
2
2
0 ex sen (x)
Z
Z
Z
x
x
x
x
x
e sen (x) dx = e cos (x) e cos (x) dx = e cos (x) e sen (x) ex sen (x) dx
Z
1

ex sen (x) = ex (sen (x) + cos (x))


2
1
u3 (x) = ex (cos (x) + sen (x))
4
por lo tanto la solucin particular es:
yp (x) = cos (x) +
yp (x) =

1
1
(sen (x) cos (x)) (cos (x) + sen (x))
4
4

1
cos (x)
2
83

La solucin general de la ecuacin dada es:


y (x) =

1
cos (x) + C1 + C2 ex + C3 ex , C1 , C2 , C3 R
2

Ejemplo 26 Resolver
y 000 2y 00 = 4 (x + 1)
i. Solucin homognea:
D2 (D 2) y = 0
yh (x) = C1 + C2 x + C3 e2x , C1 , C2 , C3 R.
ii. La ecuacin no homognea:
yp (x) = 1 u1 (x) + xu2 (x) + e2x u3 (x)
Entonces, el sistema de ecuaciones es:
1 u01 (x) + xu02 (x) + e2x u03 (x) = 0
0 u01 (x) + 0 ex u02 (x) + 2e2x u03 (x) = 0
0 u01 (x) + 0 ex u02 (x) + 4e2x u03 (x) = 4 (x + 1)
Luego:

1 x e2x

2x
W 1, x, e
= 0 1 2e2x
0 0 4e2x

= 4e2x

0
x e2x

0
1 2e2x = 2x 4 (x + 1) (2x 1) e2x = (x + 1) (2x 1)
u01 (x) =

4 (x + 1) 0 4e2x 4e
2
1
u01 (x) = 2x2 + x 1 u1 (x) = x3 + x2 x
3
2

2x
1
0
e

1
1
0
0
2e2x = 2x 4 (x + 1) 2e2x = 2 (x + 1)
u02 (x) =

2x
4e
4e
0 4 (x + 1) 4e2x
1
4e2x

u2 (x) = (x + 1)

1 x
0

1
0
0 1
0
u3 (x) =
4e2x
0 0 4 (x + 1)
Z
u3 (x) =
e2x (x + 1) dx

2x

1 2x
= e
(4 (x + 1)) = e2x (x + 1)
4

1
1
(x + 1) dx = e2x (x + 1) e2x u3 (x) = e2x
2
4

84

1
3
x+
2
4

Por lo tanto, la solucin particular es:


2 3 1 2
1
3
x + x x x (x + 1)2 x
3
2
2
4
2 3 1 2
1
3
yp (x) =
x + x x x3 2x2 x x
3
2
2
4
3
1
3
5
yp (x) = x3 x2 x
3
2
2
4
yp (x) =

La solucin general es:


1
3
5
3
y (x) = x3 x2 x + C1 + C2 x + C3 e2x , C1 , C2 , C3 R.
3
2
2
4

5.1

Ejercicios

1. Mediante el mtodo de variacin de parmetros resuelva las siguientes ecuaciones diferenciales:


(a) y 00 + y = sec(x)
(b) y 00 + y = sec(x) tan(x)
(c) y 00 4y 0 + 4y = (12 x2 6x) e2x

(d) y 000 2y 00 y 0 + 2y = e3x

(e) 2y (3) 6y (2) = x2

(f) y (4) 9y 00 = x2 + 1 sen (3x)

3
(g) (D 1) D2 4 y = xex + e2x + e2x

(h) y 00 4y = x ex , sujeta a las condiciones y(0) = 1, y 0 (0) = 0

(i) 2y 00 + y 0 y = x + 1 , sujeta a las ondiciones y(0) = 1, y 0 (0) = 0

6
6.1

Aplicaciones de las ecuaciones de 2o orden


Resortes

Recordemos la Ley de Hooke:


"La fuerza realizada por un resorte R de longitud l, deformada de manera rectilnea hasta una longitud l1
es F = k (l l1 ), donde k es una constante positiva que depende del resorte R ".

85

Figura 16: Resorte comprimido

Figura 17: Resorte extendido

l l1 < 0
l l1 > 0
F = k (l l1 ), k > 0,
F = k (l l1 )

Considere un resorte R de longitud l, vertical y fijo por su extremo superior, al cual se le ha suspendido una
masa m en si extremo inferior. Cmo determinar el movimiento de la masa m en el extremo del resorte?.

86

Figura 18: Sistema Masa-Resorte vertical


Consideremos que el sistema de coordenadas est dispuesto como en la figura. Segn la Ley de Hooke:
ks + mg = 0

(6.1)

Si en el instante t la masa se encuentra en la posicin y (t), entonces la fuerza resultante sobre la masa m es:
FS (y (t)) = k (s + y (t)) + mg = ky (t)

(2.12)

y segn la ley de Newton: Fs (y (t)) = my 00 (t) :


my 00 (t) = ky (t)

87

o bien
k
y 00 (t) + y (t) = 0
m

k
2
y (t) = 0
D +
m
y entonces el movimiento de la masa m que cuelga del extremo del resorte queda descrito por la funcin de
posicin:
r !
r !
k
k
y (t) = C1 cos
t + C2 sen
t , C1 , C2 R
m
m
Supongamos ahora que existan fuerzas que se oponen al movimiento de la masa m, por ejemplo el roce con
el aire o el fluido donde m puede estar inmerso (sistema de amortiguacin). Experimentalmente se observa que
estas fuerzas, en el caso de no ser demasiado grandes, son proporcionales a la velocidad de m.
Como esta fuerza siempre se opone al movimiento, entoncesFR (y (t)) = y 0 (t) , > 0

Figura 19: Sistema Masa-Resorte horizontal


En estas condiciones
my 00 (t) = Fy(t) = Fs (y (t)) + Fr (y (t))
= ky (t) y 0 (t)
de donde
y 00 (t) +

0
k
y (t) + y (t) = 0
m
m
88

o bien

k
2
D + D+
y = 0, > 0, k > 0
m
m

Finalmente, suponga que el sistema masa-resorte sea afectado por una fuerza adicional que en el instante t est
determinada por f (t), entonces, la funcin y (t) que describe la posicin de la masa en el instante t satisface:

k
1
2
D + D+
y = f (t)
m
m
m
Ejemplo 27 Una masa de 100 [gr] se adhiere a un resorte de acero de longitud 50 [cm]. El resorte se extiende
h i
cm
5 [cm] por efecto de esta masa. Si en algn instante se aplica a la masa una velocidad hacia abajo de 10 seg
.
Determine la funcin de posicin que describe el movimiento de la masa.
Solucin:
Suponga que no existe roce ni fuerzas externas que acten sobre la masa.

Figura 20: Sistema Masa-Resorte


Sea k la constante del resorte:
k 5 + 100 g
k
si g = 9, 8

m
seg 2

= 980

cm
seg 2

= 0

cm
cm seg
2
100g
=
= 20g
5
gr

k = 20 980 = 19600 = 1.9 104

89

Sea y (t) la posicin de la masa medida desde la posicin de equilibrio (sentido positivo hacia abajo), entonces
my 00 (t)
k
y 00 (t) + y (t)
m
19600
y 00 (t) +
y (t)
100
00
y (t) + 196y (t)

2
D2 + (14) y
Como y (0) = 0 e y 0 (0) = 10

cm
seg

ky (t)

0; a bi = 0 14i

y (t) = C1 cos (14t) + C2 sen (14t)

C1 cos (0) + C2 sen (0) = 0

14C1 sen (0) + 14C2 cos (0) = 10

C1 1 + C2 0 = 0

14C1 0 + 14C2 1 = 10

C1 = 0 y C2 =
y (t)

10
14

10
sen (14t) describe el movimiento de la masa en el extremo del resorte.
14

Ejemplo 28 Una masa de 200 [gr] extiende 5 [cm] un resorte hespiral. Sii el resorte est conectado a un amorseg
tiguador de aceite con un coeficiente de amortiguamiento de 400 dinas
, determinar el movimiento provocado
cm
si la masa es empujada hacia abajo 2 [cm] adicionales y luego soltada.
Sea y (t) la posicin de m en el instante t.
Suponga que la fuerza amortiguadora es proporcional a la velocidad m, entonces
= y 0 (t) , = 400
my 00 (t) = Fs (y (t)) Fr (y (t))
00
my (t) = ky (t) y 0 (t)

k
y 00 (t) + y 0 (t) + y (t) = 0
m
m
Fr (y (t))

como
5k + mg = 0 k =

200 980
mg
=
= 40 980 = 39200
5
5

90

La ecuacin que gobierna el movimiento de m es


400 0
39200
y (t) +
y (t) = 0
200
200
00
0
y (t) + 2y (t) + 196y (t) = 0
m2 + 2m + 196 = 0

2 4 784
m =
2

i
= 1
780 = 1 195i
2

y 00 (t) +

y entonces

195t + C2 sen
195t et
y (t) = C1 cos

Ahora bien: y (0) = 2, y 0 (0) = 0

195et C1 sen
195t + C2 cos
195t C1 cos
195t + C2 sen
195t et
y 0 (t) =

=
195C2 C1 cos
195t C2 + 195C1 sen
195t et
Por lo tanto

esto es

2 = y (0) = e0 (C1 cos (0) + C2 sen (0))

0 = y 0 (0) =
195C2 C1 cos (0) C2 + 195C1 sen (0) e0
C1

195C2 C1
C1

finalmente
t

y (t) = 2e

= 2
= 0
2
= 2, C2 =
195

!
sen 195t

195t +
cos
195

Ejemplo 29 Si un sistema "masa-resorte" con constante de resorte 20

i
h
dinas
dinas seg
,
de
amortiguamient
20
cm
cm

y con masa 10 [gr] est en movimiento afectado por una fuerza f (t) = 10et cos (t). Determine la funcin que
representa el movimiento de la masa si en el
t = 0 la masa se encuentra en la posicin de equilibrio
h instante
i
cm
con movimiento ascendente de magnitud 10 seg
.
Solucin:
Se tiene que para y (t) la posicin de la masa
y 00 (t) +

0
k
10 t
y (t) + y (t) =
e cos (t)
m
m
m
91

segn los datos del problema: k = 20, = 20, m = 10, reemplazando


y 00 + 2y 0 (t) + 2y (t) = et cos (t)
m2 + 2x + 2 = 0

2 4 8
m =
= 1 i
2
La solucin homognea:
yh (t) = C1 et cos (t) + C2 et sen (t)
Supongamos una solucin particular:
yp (t) = u1 (t) et cos (t) + u2 (t) et sen (t)

luego

et cos (t) u01 + et sen (t) u02 = 0

t
t
0
t
t
0
t
(e cos (t) e sen (t)) u1 + (e cos (t) e sen (t)) u2 = e cos (t)

cos (t) u01 + sen (t) u02 = 0

(cos (t) + sen (t)) u01 + (cos (t) sen (t)) u02 = cos (t)

0
sen (t)

cos (t) cos (t) sen (t)


cos (t) sen (t)
0
=
u1 (t) =
2

cos (t) sen (t) cos (t) + cos (t) sen (t) + sen2 (t)
cos (t)
sen (t)

(cos (t) + sen (t)) cos (t) sen (t)


1
1
u01 (t) = cos (t) sen (t) = sen (2t) u1 (t) = cos (2t)
2
4

u02 (t) =

u2 (t) =
Por lo tanto
yp (t) =

cos (t)
0

(cos (t) + sen (t)) cos (t)


= cos2 (t)

cos (t)
sen (t)

(cos (t) + sen (t)) cos (t) sen (t)


Z
1 + cos (2t)
t
1
2
cos (t) dt =
dt u2 (t) = sen (2t)
2
2 4

1
t
1
t
cos (2t) e cos (t) +
sen (2t) et sen (t) .
4
2 4

La solucin general de la ecuacin es:

t
1
1
cos (2t) + C1 et cos (t) +
sen (2t) + C2 et sen (t)
y (t) =
4
2 4
y como y (0) = 0, y 0 (0) = 10 :

t
1
3
3
1
t
1
0
t
y (t) =
cos (2t) C1 sen (2t) + C2 e cos (t)+
C1 cos (2t) + sen (2t) C2 et sen (t)
2 4
4
2
4
2 4
92

entonces
1
+ C1 = 0
4
1
y 0 (0) = C1 + C2 = 10
4
y (0) =

Luego: C1 = 14 , C2 = 10 y entonces
1
y (t) = (cos (2t) 1) et cos (t) +
4
6.1.1

t
1
sen (2t) 10 et sen (t)
2 4

Circuitos electricos

Otra aplicacin de las ecuaciones diferenciales son los circuitos elctricos sencillos que estn formados por fuentes
de voltaje (por ejemplo, bateras o generadores), resistores, inductores y capacitores. La figura a continuacin
es un diagrama del llamado circuito RLC en serie:

Figura 21: Circuito Elctrico RLC


Los principios fsicos que gobiernan los circuitos elctricos fueron establecidos por G. R. Kircho en 1859.
Los principios son los siguientes:
1.- Ley de la corriente de Kircho : La suma algebraica de las corrientes que fluyen en cualquier punto de
unin deben ser cero.
2. Ley del voltaje de Kircho : La suma algebraica de los cambios instantneos del potencial (cadas de
voltaje) en torno a cualquier lazo cerrado debe ser cero.
La ley de la corriente de Kircho implica que la misma corriente I pasa por cada elemento del circuito en
serie de la figura 18. Para aplicar la ley del voltaje de Kircho, se debe conocer la cada de voltaje a travs de
cada elemento del circuito, cuyas frmulas aparecen a continuacin:
93

a. De acuerdo con la ley de Ohm, la cada de voltaje ER a travs de un resistor es proporcional a la corriente
I que para por el resistor:
ER = RI
La constante de proporcionalidad R se conoce tambin como resistencia.
b. Se puede mostrar mediante las leyes de Faraday y Lenz que la cada de voltaje EL a travs de un inductor
es proporcional a la razn de cambio instantnea de la corriente I :
EL = L

dI
dt

La constante de proprcionalidad L se conoce como inductancia.


c. La cada de voltaje EC a travs de un capacitor es proporcional a la carga elctrica q que aparece en las
placas del capacitor:
1
EC = q
C
La constante C se conoce como capacitancia.
Suponga que una fuente de voltaje suma voltaje oenerga potencial al circuito. Si E (t) indica el voltaje que
se proporciona al circuito en el instante t, entonces la ley de voltaje de Kircho implica
EL + ER + EC = E (t) .

(6.2)

Al sustituir en (2.13) las expresiones para EL , ER y EC se tiene:


L

dI
1
+ RI + q = E (t)
dt
C

(6.3)

La corriente es justamente la razn de cambio instantnea en la carga; es decir,


I=

dq
dt

Por lo tanto, se puede expresar (2.14) slo en trminos que q como sigue:
L

dq
d2 q
1
+ R + q = E (t)
2
dt
dt
C

(6.4)

En la mayor parte de las aplicaciones interesar determinar la corriente I (t). Sei se deriva (2.15) con respecto
a T y se sustituye I en lugar de dq
dt , se obtiene:
L

dI
d2 I
1
dE
+R + I =
dt2
dt
C
dt

(6.5)

Para un circuito RLC en serie, por lo general se tiene la carga incial en el capacitor q (0) y la corriente inicial
I (0) = q 0 (0). Estas constantes proporcionan las condiciones iniciales para la ecuacin (2.15). Sin embargo,
para la ecuacin (2.16) tambin se necesita conocer la razn de cambio inicial en la corriente, I 0 (0), lo que
puede obtenerse de la ecuacin (2.14) sustituyndo los valores iniciales q (0) , I (0) y E (0) .

94

Ejemplo 30 Un circuito RLC en serie tiene una fuente de voltaje dada por E (t) = sen (100t) [V ], V = volts,
un resistor de 0, 02 [], = ohm, un inductor de 0, 001 [H], H = henry, y un capacitor de 2 [F ], F = f arad.
(Se escogen estos valores por conveniencia; los valores tpicos para el capacitor son mucho menores.) Si la
corriente y la carga iniciales en el capacitor son iguales a cero, determinar la corriente en el circuito para t > 0.
Solucin: En este caso, L = 0, 001, R = 0, 02, C = 2 y E (t) = sen (100t). Al sustituir esta en la ecuacin
(6.5) para la corriente se tiene:
(0, 001)

d2 I
dI
+ (0, 02)
+ (0, 5) I = 100 cos (100t) ,
dt2
dt

(6.6)

o en forma equivalente:
dI
d2 I
+ 20 + 500I = 100.000 cos (100t)
dt2
dt
La ecuacin homognea asociada con (6.7) tiene la ecuacin auxiliar

(6.7)

m2 + 20m + 500 = 0

20 400 2000
= 10 20i
m =
2
Por lo tanto, la solucin de la ecuacin homognea es
Ih (t) = C1 e10t cos (20t) + C2 e10t sen (20t) , C1 , C2 R.
Para determinar una solucin particular de (6.7), se puede utilizar el mtodo de los coeficientes indeterminados.
Suponga que:
Ip
Ip0
Ip00

= A cos (100t) + Bsen (100t)


= 100Asen (100t) + 100B cos (100t)
= 10.000A cos (100t) 10.000Bsen (100t)

reemplazando en (6.6):
1
2
(10.000A cos (100t) 10.000Bsen (100t)) +
(100Asen (100t) + 100B cos (100t)) + ... =
1.000
100
5
... +
(A cos (100t) + Bsen (100t)) = 100 cos (100t)
10

5
5
10A + 2B + A cos (100t) + 10B 2A + B sen (100t) = 100 cos (100t)
10
10
luego:
5
A
10
5
10B 2A + B
10
19
2B A
2
19
2A B
2

19
19
B
2B
2
4
10A + 2B +

5 100
95
A = 100 2B A = 100
10
10
5 100
95
= 0 2A +
B = 0 2A B = 0
10
10

= 100 2B +

= 100
19
19
BA= B
2
4
361
16 + 361
377
800
15200
= 100 2B +
B = 100
B = 100
B = 100 B =
A=
8
8
8
377
1508

= 0 2A =

95

por lo tanto la solucin particular es


Ip (t) =

15200
800
cos (100t) +
sen (100t)
1508
377

Y la solucin general es
I (t) = C1 e10t cos (20t) + C2 e10t sen (20t)

3800
800
cos (100t) +
sen (100t) , C1 , C2 R.
377
377

Para determinar los constantes C1 y C2 , se necesitan los valores de I (0) e I 0 (0). Se sabe que I (0) = q (0) = 0.
Para determinar I 0 (0), se sustituyen los valores de L, R y C en la ecuacin (2.14) y se igualan los dos lados en
t = 0, se obtiene:
(0, 001) I 0 (0) + (0, 02) I (0) + (0.5) q (0) = sen (0)
como I (0) = q (0) = 0, se observa que I 0 (0) = 0. Entonces:
3800
=0
377
I 0 (t) = 10C1 e10t cos (20t) 20C1 e10t sen (20t) 10C2 e10t sen (20t) + 20C2 e10t cos (20t) + ...
380000
80000
= ... +
sen (100t) +
cos (100t)
377
377
80000
I 0 (0) = 10C1 + 20C2 +
=0
377
I (0) = C1

entonces
3800
377
80000
10C1 + 20C2 +
377
C1

20C2

= 0 C1 =

3800
377

= 0 20C2 = 10C1
=

80000
377

38000 80000
42000
2100

= 20C2 =
C2 =
377
377
377
377

Por lo tanto, la corriente en el circuito RLC en serie es:


I (t) =

6.2

2100 10t
3800
800
3800 10t
cos (20t)
sen (20t)
e
e
cos (100t) +
sen (100t)
377
377
377
377

Ejercicios

1. Una fuerza de 400 N estira un resorte 2 m . Una masa de 50 kg se sujeta al extremo del resorte y se la
suelta desde la posicin de equilibrio con una velocidad dirigida hacia arriba de 10 m/s . Halle la ecuacin
cuya solucin representa el movimiento de la masa en el extremo del resorte. Resuelva esta ecuacin.
2. Otro resorte cuya constante es de 20 N/m , se suspende del mismo soporte rgido paralelamente al sistema
resorte-masadel problema anterior. Una masa de 20kg se sujeta al segundo resorte y ambas masas se
sueltan desde la posicin de equilibriocon una velocidad dirigida hacia arriba de 10 m/s .
a.- Cul de las masas tiene mayor amplitud de movimiento?
b.- Cul de los dos se mueve ms rpido en t = /4 segundos?, En /2 segundos?
96

c.- En qu instante ambas masas estn en la misma posicin ?, Dnde se ubican en dichos instantes?,
En qu direcciones se estn moviendo?
3. Un carro que pesa 128 lb est sujeto a un muro por un resorte de constante 64 lb/pie. Se aparta el carro
6 pulgadas del muro y se suelta sin velocidad inicial. Simultneamente, se le aplica una fuerza peridica
externa f (t) = 32 sen(4t). Suponiendo que no hay resistencia del aire , hallar la posicin x = x(t) del
carro en el instante t. Qu puede decir de | x(t) | para valores arbitrariamente grandes de t ?, Qu le
ocurrir al resorte despues de algn tiempo?
lb
4. Un cuerpo que pesa 4 lb se sujeta a un resorte cuya constante es 2 pie
. El medio ofrece una resistencia
al movimiento numricamente igual a la velocidad instantnea. Si el cuerpo se suelta desde un punto que
est 1 pie sobre la posicin de equilibrio, determine el instante en que el cuerpo pasa por la posicin de
equilibrio. Encuentre el instante en el cual el citado cuerpo alcanza su desplazamiento extremo desde la
posicin de equilibrio. Cul es la posicin del cuerpo en dicho instante?

5. En un circuito RLC en serie tiene una fuente de voltaje dada por E (t) = sen (100t) [volts], un resistor
de 0.02 [ohm], un inductor de 0.001 [henrys] y un capacitor de 2 [f arads].(Se escogen estos valores por
conveniencia; los valores tpicos para el capacitor son mucho menores). Si la corriente y la carga iniciales
en el capacitor son iguales a cero, determinar la corriente en el circuito para t > 0.
N
6. Una masa de 1 kg se sujeta a un resorte cuya constante es de 16 m
y el sistema completo se sumerge
en un lquido que comunica una fuerza de amortiguacin numricamente igual a 10 veces la velocidad
instantnea. Determine y resuelva las ecuaciones del movimiento si

(a) el peso se suelta, a partir del reposo, desde un punto que est 1 m abajo de la posicin de equilibrio.
(b) el peso se suelta desde un punto que est 1 m abajo de la posicin de equilibrio con una velocidad
dirigida hacia arriba de 12 m
s .
lb
. El sistema completo
7. Un cuerpo que pesa 4 lb est suspendido de un resorte cuya constante es 3 pie
se sumerge en un fluido que opone una fuerza de amortiguacin numricamente igual a la velocidad
instantnea. A partir de t = 0 se aplica sobre el sistema una fuerza exterior f (t) = et . Determine la
ecuacin del movimiento si el peso se suelta a partir del reposo, desde un punto que est 2 pies abajo de
la posicin de equilibrio.

8. Consideremos un sistema masa resorte con m = 12 , k = 17 y = 3 en unidades mks. Sea x (t) el


desplazamiento de la masa m con respecto de su posicin de equilibrio. Si la masa se pone en movimiento
con x (0) = 3 y x0 (0) = 1, determine las oscilaciones libres amortiguadas resultantes.
9. Considere el sistema masa resorte del problema anterior, pero con las condiciones iniciales x (0) = x0 (0) = 0
y con la fuerza externa impuesta f (t) = 15sen (2t). Determinar la posicin del resorte en cualquier
instante.
10. Una masa que pesa 32 [lb] (masa m = 1 [slug]) se une al extremo libre de una largo y ligero resorte que
es alargado 1 pie por una fuerza de 4 [lb]. La masa est incialmente en reposo en su posicin de equilibrio.
A partir del instante t = 0 [seg], una fuerza externa F (t) = cos (2t) se aplica a la masa. Determinar la
funcin posicin resultante x (t) para esta masa.

97

11. Considere una masa de 1 [Kg] unida a un resorte con constante k = 4, al cual se le aplica una fuerza
externa f (t) = sen (3t) por 10 minutos y despus se elimina. Si el resorte parte del reposo en su posicin
de equilibrio, determine el desplazamiento del resorte en cualquier instante.

Parte III: La transformada de Laplace


1

Definiciones y Teoremas Bsicos

Definicin 1 La transformada de Laplace de f : [0, ) R es la funcin L (f ) (s) : (, ) R, definida por


Z
est f (t) dt
L (f ) (s) =
0

Ejemplo 1 Determine la transformada de Laplace de cos (at)


Solucin:

L (cos (at)) (s) =


=
=
=
=

lim

lim

lim

s2

est cos (at) dt


Z

est cos (at) dt

"0

b #

est
(asen (at) s cos (at))
2
2
s +a
0

s cos (0t)
esb
(asen (ab) s cos (ab)) + 2
2
2
s +a
s + a2

s
, s>0
+ aa

Conclusin:
L (cos (at)) (s) =

s
, s>0
s2 + a2

Definicin 2 f : (o, ) R es de orden exponencial si existen constantes C > 0 y > 0 tales que
|f (t)| Cet , t > 0.
Proposicin 1 Si f es de orden exponencial y contnua por tramos, entonces L (f ) est definida s > .
Proposicin 2 La transformada de Laplace es lineal siempre que est definida:
L (f + g) (s) = L (f ) (s) + L (g) (s)
L (f ) (s) = L (f ) (s)

98

Teorema 1 Si f es contnua de orden exponencial en (0, ) y df


dt es contnua por tramos en [0, ), entonces



df
L
(s) = sL (f ) (s) f 0+ , donde f 0+ = lim+ f (x)
dt
x0
Teorema 2 Si f,
tramos, entonces

df
dt ,

n1

..., ddtn1f son contnuas y de orden exponencial para todo t > 0 y si


dn f
dtn

n1

(s) = s L (f ) (s) s

Ejemplo 2 Determine la transformada de eat .

dn f
dtn

es contnua por

+
dn1 f
n2 df
f 0 s
... n1
dt 0+
dt
0+

Solucin:
d at
e
dt

Ejemplo 3 Calcular L (tn )

d at
(s)
e
dt


+
= sL eat (s) ea0 = sL eat (s) 1



1
aL eat sL eat = 1 L eat =
sa
=

aeat L aeat (s) = L

Solucin: Primero determinaremos para t y t2 :


d
Como dt
(t) = 1

d
1
1
= L (1) (s) = L
(t) (s) = sL (t) (s) 0+ L (t) (s) = 2
s
dt
s
2
d
Como dt
t = 2t




d 2
1
2
2L (t) (s) = L
(s) = sL t2 (s) 0+ sL t2 (s) = 2 2 L t2 (s) = 3
t
dt
s
s

entonces, para tn :

n (n 1)!

n
n!
d n
= n+1
(t ) = ntn1 nL tn1 (s) = sL (tn ) 0 L (tn ) = L tn1 =
dt
s
s
sn
s

Otra manera:

(xn )(n) = n! L (n!) (s) = sn L (xn ) (s) 0 L (xn ) =


Ejemplo 4 Calcule L (sen (at)) (s)

99

L (n!) (s)
1
n! 1
n!
= n L (n!) = n = n+1
n
s
s
s
s
s


(cos (at))

1
d
L (sen (at)) (s) = L
(cos (at)) (s)
a
dt

1
= sL (cos (at)) (s) cos a0+
a

1
1
s
s2
1

1
= 2
+
s 2
=
2
a
s +a
a s + a2
a

1
1 s2 + a2 s2
s2
=
=
1 2
2
a
s +a
a
s2 + a2
a
=
, s>a
s2 + a2

Solucin: Como sen (at) = a1

dt

Definicin 3 Se dice que f (t) es la transformada inversa de Laplace de (s) si L (f ) (s) = (s) y se denota
L1 ( (s)) (t) = f (t)
Ejemplo 5 L1

=1

1
Ejemplo 6 L1 s2
= e2t
s

3! 1 1 3! 1 3
= L1 2s
= 2L
4
s4 = 2 t

Ejemplo 8 L1 s2s+9 = cos (3t)


Ejemplo 7 L1

3
s4

Teorema 3 de Lerch:
Sean f y g funciones contnuas por tramos de orden exponencial y supongamos que existe un nmero real
s0 tal que L (f ) (s) = L (g) (s) , s > s0 , entonces f (t) = g (t) t > 0, excepto posiblemente en puntos de
discontinuidad.
Ejemplo 9 Resolver el siguiente P.V.I.:
2

D 1 y = 1,
y (0) = 0,
y 0 (0) = 1.

100

Aplicando la transformada de Laplace:


y 00 y = 1 L
L (y 00 ) (s) L (y) (s) = L (1) (s)


1
s2 L (y) (s) sy 0+ y 0 0+ L (y) (s) =
s

2
1
s 1 L (y) s 0 =
s

1
1
L (y) (s) =
1
+
s2 1
s
s+1
L (y) (s) =
s (s2 1)
1
1+ss
1
1
L (y) (s) =
=
=

s (s 1)
s (s 1)
s1 s

1
Si y es solucin del P.V.I. D2 1 y = 1, y (0) = 0, y 0 (0) = 1 y = L1 s(s1)
1
1+ss
1
1
=
=

s (s 1)
s (s 1)
s1 s

L (y) (s) = L et L (1) = L et 1

L (y) (s) = L et 1 y = et 1

L (y) (s) =

Teorema
en [0, ) entonces
R 4 Si fes de orden exponencial
Rt

t
(i) L 0 f (u) du = 1s L (f ) (s) y L1 1s L (f ) (s) = 0 L1 ( (s)) du, donde (s) = L (f ) (s)
R R R

Rt
t t
t
(ii) L 0 0 1 0 2 ... 0 n1 f (u) dudtn1 ...dt2 dt1 = s1n L (f ) (s)

Ejemplo 10 Determine L1
Como

1.1

1
s(s1)

1
s

1
s1

1
s(s1)

= 1s L (et ) (s) L1

1
s(s1)

= L1

Ejercicios

1. Pruebe que L (sen (at)) (s) =

a
s2 +a2 ,

s L (e

Rt
) (s) = 0 eu du = eu |t0 = et 1.

s>0

2. Determine la transformada de f (t) = 1, la funcin constante igual a uno.


3. Determine la transformada de f (t) = eat , a R es una constante.
4. Determine la transformada de Laplace de:

1 , 0 < x < 1
(a) f (x) =
1,
1x

4, 0 x < 2
(b) f (x) =
0,
2x
101

(c) f (x) =
(d) f (x) =

x,
1,

0<x<1
1x

2x + 1 ,
0,

0<x<1
1x

(e) f (x) = (x + 1)3

5. La funcin gamma, , se define mediante la siguiente integral:


() =

t1 et dt , > 0

(+1)
s+1

Muestre que L(x ) =

, > 1.

6. Use el ejercicio anterior para determinarlas siguientes igualdades:


p
(a) L(x1/2 ) = s
(b) L(x1/2 ) =
(c) L(x3/2 ) =

2s3/2

3
4s5/2

7. Calcule las correspondientes transformadas de Laplace para verificar las igualdades que se afirman a
continuacin.
(a) L(senh(ax))(s) =

a
s2 a2
s
s2 a2

, p >| a |

, p >| a |

s
(c) L(sen2 (ax))(s) = 12 1s s2 +4a
2

s
(d) L(cos2 (ax))(s) = 12 1s + s2 +4a
2
(b) L(cosh(ax))(s) =

8. Calcule L(cos(x) cos(2x))(s), (Considere cos(a b))


9. Verifique las siguientes igualdades:
2
s2 +16
s
2(s2 +4)

(a) L(sen(2x) cos(2x))(s) =

(b) L(cos2 (x))(s) =

1
2s

(c) L( 4x2 5sen(3x))(s) = 4 s23 5 s23+9

(d) L(et senh(x))(s) =

1
2(s2)

1
2s

10. Demuestre que la funcin f (x) = x12 no tiene transformada de Laplace. (Observe que L(f (x))(s) =
R1 st
R
R
e f (t) dt + est f (t) dt , y aplique la de finicin de integral impropia para mostrar que est f (t) dt
0

no existe.)

11. Muestre que f (x) = x1/3 es de orden exponencial pero su derivada no lo es.
102

12. Muestre que la funcin f (x) = 2xex cos(ex ) no es de orden exponencial y tiene transformada de Laplace.
2
2
En efecto L(f (x))(s) = sL(sen(ex ))(s) y sen(ex ) es de orden exponencial.
13. Verifique las siguientes igualdades:
3

3 2
1 3
(a) L1 ( (s+1)
s4 ) = 1 + 3x + 2 x + 6 x

(b) L1 ( s12

48
s5 )

= x 2x4

(c) L1 ( ss+1
2x) +
2 +2 ) = cos(

2x)

1 sen(
2

1
) = 14 senh(4x)
(d) L1 ( s2 16

(e) L1 ( 2s6
s2 +9 ) = 2 cos(3x) 2sen(3x)
1
(f) L1 ( 4s4s
2 +1 ) = cos( 2 x)

(g) L1 ( 2s6
s2 +9 ) = 2 cos(3x) 2sen(3x)

(h) L1 ( s210s
25 ) = 10 cosh(5x)

s+1
1
) = 20
+
(i) L1 ( (s2 4s)(s+5)

(j) L1 ( 4s21+1 ) = 12 sen( 12 x)

5 4t
36 e

4 5t
45 e

(k) L1 ( s2 (s12 +4) ) = 14 x 18 sen(2x)

s1
(l) L1 ( s2 (s
2 +1) ) = 1 x cos(x) + sen(x)

s
)=
(m) L1 ( (s2 +4)(s+2)

1
4

cos(2x) + 14 sen(2x) 14 e2x

6s+3
1
(n) L1 ( (s2 +4)(s
2 +1) ) = 2 cos(x) + sen(x) 2 cos(2x) 2 sen(2x)

Teoremas de Traslacin.

Teorema 5 (1er teorema de traslacion)


Suponga que L (f ) (s) = (s) es conocida, entonces

L eat f (t) (s) = (s a) = L (f (t)) (s a)

o tambin

L1 (L (f ) (s a)) = eat f (t)


Ejemplo 11 Calcular la transformada de Laplace de e2t cos (3t)

L e2t cos (3t) (s) = L (cos (3t)) (s 2)

s
=
s2 + 32
s=s2

s2
2

(s 2) +
103

32

s2
2

(s 2) + 9

Ejemplo 12 Calcular la transformada inversa de

L1

s
s2 4s + 13

= L1
= L1
= L1

s
s2 4s+13 .

s
2

(s 2) + 9

s2

s2

(s 2) + 9
2

(s 2) + 9

= L1

(s 2) + 9
!

(s 2)2 + 9

+ L1

s2+2

2
2

(s 2) + 9

!
2 1
3
1
= L (L (cos (3t)) (s 2)) + L
3
(s 2)2 + 9
2
= e2t cos (3t) + L1 (L (sen (3t)) (s 2))
3
2
= e2t cos (3t) + e2t sen (3t)
3
Definicin 4 Dada c > 0, la funcin escaln unitario o de Heaviside, se define como

0 si 0 t < c
c (t) =
1
si c t
cuya grfica es:

Figura 22: Funcin escaln unitario c (t)


Note que: a tiene la siguiente grfica:

104

Figura 23: Funcin escaln unitario a (t)


y b tiene la grfica:

Figura 24: Funcin escaln unitario b (t)


y es inmediato que a b = 1[a,b) de manera que:

si 0 t < a
0
h (t) si a t < b = h (t) (a (t) b (t))
f (t) =

0
si b t

Ejemplo 13 Considere la funcin:

105

f (t) =
Entonces f (t) = (t + 1) (2 (t) 5 (t))

0
0x<2
t+1 2t<5

0
5t

Teorema 6 2o Teorema de traslacin.


Si f (t) = a (t) g (t a) entonces L (f (t)) (s) = eas L (g (t)) (s) o en trminos de la transformada inversa:
L1 (eas L (g (t)) (s)) = a (t) g (t a) .

Ejemplo 14 Calcular L 2 (t) (t + 3)2 .


2

Primero se determina la funcin g (t) de la siguiente forma: g (t 2) = (t + 3) de manera que g (t) =


(t + 2 + 3)2 = (t + 5)2 y entonces
2 (t) (t + 3)2 = 2 (t) g (t 2) , g (t) = (t + 5)

luego:

L 2 (t) (t + 3)2
= e2s L (t + 5)2 (s) = e2s L t2 + 10t + 25 (s)

10 25
2
2s
= e
+ 2 +
s3
s
s

si 0 t < a
0
h (t) si a t < b ?. Se puede escribir f (t) = h (t) (a (t) b (t)) =
Cmo evaluar L (f ) si f (t) =

0
si b t
a h (t) b h (t) , entonces
L (f (t)) (s) = L (a (t) h (t)) (s) L (b (t) h (t)) (s)
Note que si: g (t) = h (t + a) g (t a) = h (t a + a) = h (t), luego:
L (f (t)) (s) = L (a (t) h (t a + a)) (s) L (b (t) h (t b + b)) (s)
= L (a (t) g (t a)) (s) L (b (t) g (t b)) (s)
o, equivalentemente:
L (f (t)) (s) = eas L (h (t + a)) (s) ebs L (h (t + b)) (s)
Ejemplo 15 Calcular L (f (t)) (s) si

f (t) =

0
0t<1
t+3 1t<4

0
4t

Como f (t) = (t + 3) (1 (t) 4 (t)), entonces:

f (t) = (t + 3) (1 (t) 4 (t)) = 1 (t) (t + 3) 4 (t) (t + 3)


= 1 (t) ((t + 4) 1) 4 (t) ((t + 7) 4)
106

entonces
L (f (t)) (s) = L (1 (t) ((t + 4) 1)) (s) L (4 (t) ((t + 7) 4)) (s)
= es L (t + 3 + 1) (s) e4s L (t + 3 + 4) (s)
= es L (t + 4) (s) e4s L (t + 7) (s)

4
7
1
1
4s
= es
+
+

e
s2
s
s2
s
Ejemplo 16 Calcular L (f ) si

0
0t<1
t+3 1t<4
f (t) =
4t<5
t2

0
5t

Note que

0
0t<1
0
t+3 1t<4 +
t2
f (t) =

0
4t
0

entonces:

0t<4
4t<5
5t

f (t) = (t + 3) (1 (t) 4 (t)) + t2 (4 (t) 5 (t))

= 1 (t) (t + 3) + 4 (t) t2 t 3 5 (t) t2

L (f (t)) (s) = L (1 (t) (t + 3)) (s) + L 4 (t) t2 t 3 (s) L 5 (t) t2 (s)

= es L ((t + 1) + 3) (s) + e4s L (t + 4)2 (t + 4) 3 (s) e5s L (t + 5)2 (s)

= es L (t + 4) (s) + e4s L t2 + 7t + 9 (s) e5s L t2 + 10t + 25 (s)

4
7
9
10 25
1
2
1
4s
5s
= es
+
+
+
+
+
+
e

e
s2
s
s3 s2
s
s3
s2
s
Ejemplo 17 Determine la transformada inversa de
e3s
s2 + 6s + 10
Como
1

e3s
s2 + 6s + 10

107

= 3 (t) g (t 3)

entonces
1

e3s
s2 + 6s + 10

1
= 3 (t) L
s2 + 6s + 10

!
1
1
= 3 (t) L
2
(s + 3) + 1
1

= 3 (t) L1 (L (sen (t)) (s + 3))




= 3 (t) L1 L e3t sen (t) (s) (t3)
= 3 (t) e3(t3) sen (t 3)

2.1

Ejercicios

1. Verifique las siguientes igualdades:


(a) L(xe10x ) =
(b) L(xe6x ) =

1
(s10)2
1
(s+6)2

(c) L(x3 e2x ) =

3!
(s+2)4

(d) L(x10 e7x ) =

10!
(s+7)11

(e) L(e5x sen(3x) =

3
(s1)2 +9

(f) L(e2x cos(4x)) =


(g) L(ex cosh(x)) =

(h) L( 16 x3 ex ) =

s+2
(s+2)2 +16
s+1
(s+1)2 1

1
(s1)4

(i) L(e2x (x 1)2 ) =

2
(s2)3

2
(s2)2

(j) L(ex cos2 3x) ) =

1
2(s1)

(s1)
2(s1)2 +72

1
s2

2. Verifique las siguientes igualdades:


(a) L1 (

1
1 x
sen(2x)
s2 +2s+5 ) = 2 e
5s
2x
+ 10xe2x
(s2)2 ) = 5e

(c) L1 (

2s+5
s2 +6s+34 )

(b) L1 (
(d) L1 (

(s+1)2
(s+2)4 )

= 12 e3x cos(5x) 15 e3x sen(5x)

= xe2x x2 e2x + 16 x3 e2x ,

3. Verifique las igualdades a continuacin:


(a) L(e2x U2 (x)) =

e2s
s+1

(b) L((3x + 1)U3 (x)) =


(c) L((3x + 1)U3 (x)) =

3e3s
s2

10e3s
s

3e3s
s2

10e3s
s

108

(d) L(sen(x)U/2 (x)) =


(e) L(xex5 U5 (x)) =

ses
s2 +1

e5s
(s1)2

5e5s
s1

s/2

(f) L1 ( ses2 +4 ) = cos(2(x 2 ))U/2 (x)


2s

(g) L1 ( s2e(s1) ) = (1 x + ex2 )U2 (x)


4. Verifique las igualdades a continuacin:
a.- L(1 U4 (x) + U5 (x) ) =

1
s

e4s
s

b.-L(sen(x)Ur/2 (x) ) =

se3/s
s2 +1

c.-L(sen(x)(1 U2 (x)) ) =

1
s2 +1

e5s
s

e2s
s2 +1

5. Verifique las igualdades y trace la grfica de cada transformada inversa

x, 0x<1
1
es
1
(a) L ( s2 s2 ) =
1
, 1x

, 0x<1
2
s
s
3x + 5 , 1 x < 2
(b) L 1 ( s2 3 se2 + 5 se2 ) =

2x 5 ,
2x
(c) L

1 3x
(ln( s3
ex )
s+1 ))) = x (e
2

1
(d) L1 (ln( ss2+1
+4 ))) = x (2 cos(x) 2 cos(2x))

6. Determine L ((x2 3x)U2 (x)) = e2s ( s23 +

1
s2

2s ) , (Exprese x2 3x como potencias de x 2).

7. Cmo expresaria las siguientes funciones para aplicar directamente el segundo teorema de traslacin?
(a) L((2x + 1)U1 (x))

(b) L (et U5 (x))

(c) L (cos(x)U (x))

(d) L (sen(x)U2 (x))

Funciones Peridicas.

Definicin 5 Una funcin peridica es una funcin f (t) tal que


f (t + p) = f (t) t [0, p]
y adems p es el menor nmero positivo con esta propiedad.
Note que f (t) = f (t + p) = f (t + 2p) = f (t + 3p) = ... = f (t + kp) , k N.
Ejemplo 18 f (x) = cos (x) : f (x + 2) = cos (x + 2) = cos (x) = f (x) y p = 2 es en menor nmero
positivo con esa propiedad.
109

Ejemplo 19 Considere el siguiente grfico:

Figura 25: Funcin Peridica

entonces
f (x) =

1
si x [2n, 2n + 1)
0 si x [2n + 1, 2n + 2)

Teorema 7 Si f es de orden exponencial y peridica de perodo p, entonces


Z p
1
L (f (t)) (s) =
est f (t) dt
1 esp 0
Ejemplo 20 Determine la transformada de f si la grfica de f es:

Figura 26: Funcin Peridica

110

De la grfica se deduce que f es perodica de perodo 2 y entonces:


Z 2
1
est f (t) dt
L (f ) (s) =
1 e2s 0

Z 1
Z 2
1
st
st
1

e
dt
+
0

e
dt
=
1 e2s
0
1
Z 1

1
=
est dt
1 e2s
0
st 1

1
e
1 s
1
=

=
e 1

2s
2s
1e
s
1e
s
0

1
1 1 es
=

2s
s 1e
s (1 + es )

Ejemplo 21 Calcular L (f ) si f (t) = n + 1 si n t < n + 1.


Anotando f (t) = t + g (t) se puede ver que g (t) = f (t) t = n + 1 t si n t < n + 1, note que
n t < n + 1 t = n + u, 0 u < 1 g (t) = n + 1 t = n + 1 (n + u)
Por lo tanto g (t) = 1 u si n t < n + 1, t = n + u.
Por otra parte, como 0 u < 1 y t = n + u, en particular, para t [0, 1), g (t) = g (0 + u) = g (u) y para
n t < n + 1, t = n + u, g (t) = g (n + u) = 1 u y g (0 + u) = 1 u g (n + u) = g (0 + u) = g (u) g (u) =
g (u + 1) por lo tanto g (u) tiene perodo 1.
1
L (t + g (t)) = 2 + L (g (t)) (s)
s
Z 1
1
1
1
1
L (g (t)) (s) =
est (1 t) dt = 2 es + es 2
s
1e
s
s
s

1
1 s
es
1 s
1
=
e + 2 e 1 =
2
s
s
1e
s
s
s (1 e ) s
1
es
1
L (f (t)) (s) = 2 +

s
s (1 es ) s2

L (f (t)) (s)

Teorema 8 Si L (f (t)) (s) = (s) entonces


L (tn f (t)) (s) = (1)

dn
(s)
dsn

Ejemplo 22 Calcular la transformada de tn .

L (tn ) = L (tn 1) = (1)


= (1)n
n!

dn
(1)
dsn

dn 1
s
= (1)n (1)n n!s(n+1)
dsn

sn+1
111

Ejemplo 23 Determine la transformada inversa de

s
(s2 +1)2

Sea
1

g (t) = L

s
(s2 + 1)2

entonces:

1 d
1
L (g (t)) (s) =
2 = 2
ds s2 + 1
(s2 + 1)

1 d
=
(L (sen (t)) (s))
2 ds
s

Luego

1 d
(L (sen (t)) (s))

2 ds
d
(L (sen (t)) (s))
ds
d
(L (sen (t)) (s))
ds
tsen (t)
t
sen (t)
2

L (g (t)) (s) =
2L (g (t)) (s) =
L (2g (t)) (s) =
2g (t) =
g (t) =

Ejemplo 24 Determine la transformada inversa de

L
Como

d
ds

1
s2 + 1

1
.
(s2 +1)2

1
(s2 + 1)2

= g (t)

1
=
2
2 = 2s
2
2
(s + 1)
(s + 1)
2s

d
ds

1
s2 + 1

entonces
1

1
(s2 + 1)2

=
=
=
=

1
d
1
L

2s
ds s2 + 1

1 1 1
L
(L (tsen (t)))
2
s
Z t
1
1
vsen (v) dv = (v cos (v) + sen (v))|t0
2 0
2
1
(sen (t) t cos (t))
2
1

Ejemplo 25 Determine L1 (ln (1 + s))


112

Sea
g (t)

= L1 (ln (1 + s))
L (g (t)) (s) = ln (1 + s)
1
d

=
(L (g (t)) (s)) = L (tg (t)) (s)
1+s
ds

L (tg (t)) (s) = L et (s)


tg (t) = et
et
g (t) =
t

Por lo tanto
L1 (ln (1 + s)) =

3.1

et
t

Ejercicios

1. Compruebe las siguientes igualdades empleando la formula de derivacin para transformadas:


3e3s
10e3s
s2 +
s
2s(s2 3)
(s2 +1)3

(a) L(xsenh(3x)) =
(b) L(x2 cos(x)) =

(c) L(xe3x cos(3x)) =


(d) L1 (
(e) L1 (

(s+3)2 9
[(s+3)2 +9]2

(s+1)
1
x
sen(x)
(s2 +2s+2)2 ) = 2 xe
s
1
(s2 +1)2 ) = 2 x sen(x)

2. Muestre que L (cos2 (x)) =

s2 +2
s(s2 +4)

empleando la identidad d/dx(cos2 (x)) = sen(2x).

3. Determine la transformada de las funciones definidas por:


(a) f (x) = sen(x) si 0 x 2 y f (x + 2) = f (x) si x 2
(b) f (x) = cos(x) si 0 x 2 y f (x + 2) = f (x) si x 2
4. Determine la transformada de las funciones peridicas f (x), cuyas grficas se muestran a continuacin
a.-

b.-

113

c.-

d.-

e.-

f.-

5. Una masa de 32 [lb] se une al extremo libre de un largo y ligero resorte que es alargado 1 [pie] por una
fuerza de 4 [lb]. La masa est inicialmente en reposo en su posicin de equilibrio. A partir del instante
t = 0 (segundos), una fuerza externa F (t) = cos (2t) se aplica por 2 segundos y luego se permite que la
masa contine su movimiento sin impedimento. Determinar la funcin posicin resultante x (t) para esta
masa.
6. Considere un circuito RLC con R = 100 [Ohm] , L = 1 [Henry] , C = 0.001 [F araday] y un batera que
proporciona E0 = 90 [volts]. Inicialmente, no hay corriente en el circuito y no hay carga en el condensador.
En el instante t = 0, un interruptor en el circuito se cierra y se mantiene as durante 1 [segundo]. En
el instante t = 1 se abre y se mantiene as de ah en adelante. Determinar la corriente resultante en el
circuito.

El teorema de convolucin.

Definicin 6 Sean f y g funciones contnuas por tramos y de orden exponencial. La convolucin entre f y g
es la fuincin (f g) (t) definida por
Z t
(f g) (t) =
f (t u) g (u) du
0

Observacin 1
(f g) (t) =
=

f (t u) g (u) du, haciendo en cambio de variables s = t u, u = t s

f (s) g (t s) (ds) =

g (t s) f (s) ds = (g f ) (t)

114

Teorema 9 Sean f y g funciones contnuas por tramos de orden exponencial, entonces:


Z t

L ((f g) (t)) (s) = L


f (t u) g (u) du (s) = L (f (t)) (s) L (g (t)) (s)
0

o lo que es igual

f (t u) g (u) du = L1 (L (f (t)) (s) L (g (t)) (s))

Ejemplo 26 Calcular eat ebt


eat ebt =

ea(tu) ebu du =

eat e(ba)u du = eat

Ejemplo 27 Resolver la ecuacin integral


y (t) = 1

t
ebt eat

e(ba)u = eat e(ba)t 1 =


ba
ba
0

(t u) y (u) du

Aplicando transformada de Laplace:


L (y (t)) (s) = L (1) (s) L
L (y (t)) (s) =
L (y (t)) (s) =
L (y (t)) (s) =

1
=
L (y (t)) (s) 1 + 2
s
2

s +1
L (y (t)) (s)
=
s2
L (y (t)) (s) =
Ejemplo 28 Calcular L1

L1

1
s(s2 +1)

1
2
s (s + 1)

= L1

(t u) y (u) du (s)

1
L (t y (t)) (s)
s
1
L (t) (s) L (y (t)) (s)
s
1
1
L (y (t)) (s)
s s2
1
s
1
s
s
= L (cos (t)) y (t) = cos (t)
2
s +1

1
1
2
s s +1

= L1 (L (1) (s) L (sen (t)) (s))


= 1 sen (t)
Z t
Z t
t
=
1sen (u) du =
sen (u) du = cos|0
0

= 1 cos (t)

115

4.1

Ejercicios

1. En los siguientes problemas verifique las igualdades sin evaluar la integral


x

R
(a) L
cos(t)dt = s21+1
0

(b) L
(c) L

x
R

0
x
R
0

t sen(t) dt =

2
(s2 +1)2

sen(t) cos(t )dt =

R
(d) L x e dt =

3s+1
s2 (s+1)3

(e) L(1 x3 ) =

s
(s2 +1)2

6
s5

(f) L(1 e2x ) =

(g) L(x2 x3 ) =

1
s(s+2)

2
s3 (s1)2
1
(s2)(s2 +1)

(h) L(e2x sen(x)) =

2. Suponiendo que L1 (F (s)) = f (x) determine la validez de las siguientes igualdades a continuacin:
(a) L1 ( s2s+4 F (s)) =
1
F (s)) =
(b) L1 ( (s+1)

Rt
0

f ( ) cos(2(t )) dt

Rx

f ( )e5(x ) dt

3. Verifique las siguientes igualdades mediante convolucin:


1
(a) L1 ( s(s+1)
) = 1 ex
1
x
(b) L1 ( (s+1)
,
2 ) = xe

1
) = ex 12 x2 x 1
(c) L1 ( s3 (s1)

1
(d) L1 ( (s2 +4s+5)
2) =

1 2x
(sen(x)
2e

x cos(x))

4. Verifique las igualdades y trace la grfica de cada transformada inversa

x, 0x<1
1
es
1
(a) L ( s2 s2 ) =
1
, 1x

, 0x<1
2
s
s
3x + 5 , 1 x < 2
(b) L 1 ( s2 3 se2 + 5 se2 ) =

2x 5 ,
2x

116

Aplicaciones de la Transformada de Laplace a Ecuaciones Diferenciales

Ejemplo 29 Resolver
y 0 + 2y +

Solucin: Sea

t
0t<1
2t 1t<2
= h (t)
y (s) ds =

0
2t

h (t) = t (0 (t) 1 (t)) + (2 t) (1 (t) 2 (t))


= t0 (t) + 1 (t) (2 2t) 1 (t) (2 t) 2 (t)
= t0 (t) 2 (t 1) 1 (t) + (t 2) 2 (t)
entonces, aplicando transformada de Laplace
1
L (y 0 (t)) (s) + 2L (y (t)) (s) + L (y (t)) (s) = L (0 (t) (t 0)) 2L (1 (t) (t 1) + L (2 (t) (t 2)))
s
+
1
sL (y (t)) (s) y 0 + 2L (y (t)) (s) + L (y (t)))s = e0s L (t) (s) 2es L (t) (s) + e2s L (t) (s)
s


1
1
1 2es + e2s
L (y (t)) (s) y 0+ =
s+2+
s
s2
Si y (0+ ) = C, constante, entonces

L (y (t)) (s) =

1
s (s + 1)

1 2es + e2s +

sC
(s + 1)2

y, aplicando transformada inversa:


"
#
"
#
"
#
"
#
1
e2s
s
1
1
1
s
1
1
y (t) = L
2L
e
+L
+ CL
s (s + 1)2
s (s + 1)2
s (s + 1)2
(s + 1)2
Luego:
i
L1

"

1
s (s + 1)2

=
=
=
=
=
=
=

"
#

1
1
L1
L1
s
(s + 1)2

d
1
1 L1
ds s + 1

d
1 L1 L et
ds
t
1
1L
L te
t
1 te
Z t
1|(tu) ueu du
0
Z t
ueu du = 1 (1 + t) et
0

117

ii
1

"

s
(s + 1)2

= L

= L1

"
"

s+11
(s + 1)2

1
1

s + 1 (s + 1)2

= et L1 [L (t) (s + 1)]

= et L1 L tet
= et (1 t)
y entonces

y (t) = 1 (1 + t) et 2L1 es L 1 (1 + t) et (s) + L1 e2s L 1 (1 + t) et + Cet (1 t)

+ L2 L 2 (t) 1 (t 1) e2t
+ Cet (1 t)
= 1 (1 + t) et 2L1 L 1 (t) 1 te1t

finalmente

y (t) = 1 (1 + t) et 21 (t) 1 te1t + 2 (t) 1 (t 1) e2t + Cet (1 t)

Ejemplo 30 Resolver la ecuacin

y 00 2y 0 + y = tet sen (t)


Solucin: Aplicando la transformada de Laplace a la ecuacin, se tiene

L (y 00 (t)) (s) 2L (y 0 (t)) (s) + L (y (t)) (s) = L tet sen (t) (s)



d t
s2 Y (s) sy 0+ y 0 0+ 2 sY (s) y 0+ + Y (s) =
L e sen (t) (s)
ds
+
+

+
d
2
0
s Y (s) sy 0 y 0 2 sY (s) y 0
+ Y (s) = (L (sen (t)) (s 1))
ds
!

d
1
2
+
0
+
+
+ Y (s) =
s Y (s) sy 0 y 0 2 sY (s) y 0
ds (s 1)2 + 1


s2 Y (s) sy 0+ y 0 0+ 2 sY (s) y 0+ + Y (s) =

Sean y (0+ ) = a, y 0 (0+ ) = b, se tiene

2
s 2s + 1 Y (s) as b + 2a =
Y (s) =

2 (s 1)
2

2
(s 1) + 1

2 (s 1)
2

(s 1)2 + 1

2 (s 1)
b
a (s 2)
2 +

2 +
2
(s 1)
(s 1)2
(s 1) (s 1) + 1
2

118

Aplicando transformada inversa:

y (t) = L

Observe que

"
#
"
#
2 (s 1)
1
1
s1

1
1

+ aL

2 + bL
(s 1)2
(s 1)2
(s 1)2
(s 1)2 (s 1)2 + 1
2 (s 1)
2 = (s 1)

(s 1)2 (s 1)2 + 1

donde

(s) =

2s
s2 (s2 + 1)2

entonces, si (s) = L (f (t)) (s), se concluye que


t

2 (s 1)
2 = L (f (t)) (s 1) = L e f (t)

(s 1)2 (s 1)2 + 1

luego, solamente es necesario determinar f (t):

2s
1
d
1
(s) =
= L (t) L (tsen (t)) = L (t tsen (t))
2 = s2 ds
s2 + 1
s2 (s2 + 1)
Entonces
f (t) = t sen (t) =
Entonces

L1

Entonces

L
y entonces

(t u) u sen (u) du

2 (s 1)

t
2 = e

2
2
(s 1) (s 1) + 1

t
0

(t u) u sen (u) du

2 (s 1)
1

t
2 = e 1 cos (t) tsen (t)

2
(s 1)2 (s 1)2 + 1
Z

(t u) u sen (u) du + btet + a et tet


0

1
t
y (t) = e 1 cos (t) tsen (t) + bt + a at
2
y (t) = et

Ejemplo 31 Resolver
y 00 + 4y 0 + 4y =

0
e(t2)

,t 2
,2 < t
119

= h (t) , y (0) = 1, y 0 (0) = 1

Solucin: note que h (t) = 2 (t) e(t2) = 2 (t) g (t 2) , g (t) = et


Entonces:

L (y 00 (t)) (s) + 4L (y 0 (t)) (s) + 4L (y (t)) (s) = e2s L et (s)


e2s
s2 Y (s) sy 0+ y 0 0+ + 4 sY (s) y 0+ + 4Y (s) =
s+1
e2s
s2 Y (s) + s 1 + 4sY (s) 4 + 4Y (s) =
s+1

2
e2s
s + 4s + 4 Y (s) =
+5s
s+1

entonces, dividiendo por s2 + 4s + 4 , se tiene:


Y (s) =

e2s
5
s
+

(s + 1) (s2 + 4s + 4) s2 + 4s + 4 s2 + 4s + 4

Luego:
1
s2 + 4s + 4

8
1
1

s2 4s 4
8 (s 2)2 8

1
8t (s 2)
= L senh
8

1
= L e2t senh
8t (s)
8
2t

e
8t (s)
= L senh
8
=

adems:
s
s2 + 4s + 4

s
s
=
2
s2 4s 4
(s 2) 8


= L cos h
8t (s 2)


= L e2t cos
8t (s)


= L e2t cos h
8t (s)
=


e2t
1
1
1
t
8t (s)
=

= L e senh
(s + 1) (s2 + 4s + 4)
s + 1 s2 + 4s + 4
8

120

donde
e

Z


e2t
et t 3u
senh
8t
=
e senh
8u du
8
8 0

Z
et t 3u e 8u e 8u
=
e
du
2
8 0
Z t

et
e(3+ 8)u e(3 8)u du
=
2 8 0
t

et e(3+ 8)u
e(3 8)u


=
2 8 3+ 8
3 8
0

3+ 8)t
3 8)t
t
(
(
e
e
e

=
2 8 3+ 8
3 8

=
Por lo tanto:

e(2+ 8)t
e(2 8)t

6 8 16 16 + 6 8

5e2t

8t senh
8t + 2 (t)
y (t) = e cos h
8
2t

e(2 8)t
e(2+ 8)t

6 8 16 16 + 6 8

Ejemplo 32 Resolver
y 00 + y = t, y (1) = 1, y 0 (1) = 1
Observe que y (u + 1), es tal que y (0 + 1) = y (1). Considere w (u) = y (u + 1) = y (t), entonces
w (0) = y (0 + 1) = 1, w0 (0) = y 0 (0 + 1) = y 0 (1) = 1
y entonces
w00 (u) + w (u) = u + 1
Luego, aplicando la transformada de Laplace, se tiene:
L (w00 (u)) (s) + L (w (u)) (s) =


s2 W (s) sw 0+ w0 0+ + W (s) =

s2 + 1 W (s) s 1 =
W (s) =

1
1
+
s2
s
1
1
+
s2
s
1
1
+
s2
s
1
1
s+1
+
+ 2
2
2
2
s (s + 1) s (s + 1) s + 1

Por lo tanto
w (u) = L1 [L (u) (s) L (sen (u)) (s)] + L1 [L (1) (s) L (sen (u)) (s)] + L1 [L (cos (u)) (s)] + L1 [L (sen (u)) (s)]
w (u) = u sen (u) + 1 sen (u) + cos (u) + sen (u)
Z u
Z u
w (u) =
(u v) sen (v) dv +
1sen (v) dv + cos (u) + sen (u)
0

w (u) = u senu + 1 cos (u) + cos (u) + sen (u) = 1 + u


121

Realizando el cambio de variables: u + 1 = t :


y (t) = t
Ejemplo 33 Resolver
ty 00 + (3t 1) y 0 (4t + 9) y = 0, y (0) = 0 , y 0 (0) = 1
Aplicando transformada de Laplace, se tiene
L (ty 00 (t)) (s) + L ((3t 1) y 0 (t)) (s) L ((4t + 9) y (t)) (s) = 0

d
(L (y 00 (t)) (s)) + 3L (ty 0 (t)) (s) L (y 0 (t)) (s) 4L (ty (t)) (s) 9L (y (t)) (s)
ds


d 2
d
d

s Y (s) sy 0+ y 0 0+ + 3 (1)
(L (y 0 (t)) (s)) L (y 0 (t)) (s) 4 (1)
(Y (s)) 9Y (s)
ds
ds
ds


d
2sY (s) + s2 Y 0 (s) 0 3
sY (s) y 0+ sY (s) y 0+ + 4Y 0 (s) 9Y (s)
ds
2sY (s) s2 Y 0 (s) 3Y (s) 3sY 0 (s) sY (s) + 4Y 0 (s) 9Y (s)

Agrupando los trminos semejantes y dividiendo por s2 3s + 4 , se tiene:


2

s 3s + 4 Y 0 (s) = (3s + 12) Y (s)


3s + 12
Y 0 (s)
= 2
Y (s)
s + 3s 4
Y 0 (s)
s+4
= 3
Y (s)
(s + 4) (s 1)
Y 0 (s)
3
=
Y (s)
s1

Luego
d
d
(ln (Y (s))) = 3 (ln (s 1))
ds
ds
integrando, se tiene

Por lo tanto, la curva solucin es:

ln (Y (s)) = ln (s 1)3
1
Y (s) =
3
(s 1)
1
2!
L (y (t)) (s) =
2! (s 1)3
1 2
L (y (t)) (s) =
L t (s 1)
2!
1 t 2
L (y (t)) (s) =
L et
2!
y (t) =

122

t2 et
2

= 0
= 0
= 0
= 0

5.1

Ejercicios

1. Resuelva los siguientes problemas de valor inicial mediante el empleo de la transformada de Laplace.
Cuando sea apropiado haga uso de funciones escaln unitario.
(a) y 00 2y 0 + 5y = 1 + x , y(0) = 0 , y 0 (0) = 4

(b) y 00 2y 0 = ex senh(x) , y(0) = 0 , y 0 (0) = 0

(c) y 000 + 2y 00 y 0 2y = sen(3x) , y(0) = 0 , y 0 (0) = 0 , y 00 (0) = 1,

(d) y (4) y = x , y(0) = 0 , y 0 (0) = 0 , y 00 (0) = 0, y 000 (0) = 0


(e) y 00 + a2 y = f (x) ,

y(0) = y 0 (0) = 0

1 , 0x<1
1 , 1 x

x , 0x<1
0
(g) y + 2y = f (x) , y(0) = 0 , donde f (x) =
0 , 1x

1 , 0x<1
(h) y 00 + 4y = f (x) , y(0) = 0 , y 0 (0) = 1 , f (x) =
0 , 1x
0

(f) y + y = f (x) , y(0) = 0 , donde f (x) =

(i) y 00 + 4y = sen(x)U2 (x) , y(0) = 1 , y 0 (0) = 0

(j) y 00 5y 0 + 6y = U1 (x) , y(0) = 0 , y 0 (0) = 1

0 0x<
1 x < 2
(k) y 00 + y = f (x) , y(0) = 0 , y 0 (0) = 1 , f (x) =

0
2 x
(l) y 00 + 4y 0 + 3y = 1 U2 (x) U4 (x) + U6 (x) , y(0) = 0 , y 0 (0) = 0

2. Resuelva las siguientes ecuaciones integrales mediante la transformada de Laplace


(a) y(x) +

Zx

(x t)y(t) dt = x

(b) y(x) = 2x

Zx

sen(t)y(x t) dt

(c) y(x) = xe +

Zx

y(x ) d

(d) 3sen(2x) = y(x) + 2

Zx

(x t)y(t) dt

3. En este problema se da una aplicacin de la convolucin a un problema prctico. Considere un hilo en


forma de curva suave y un abalorio que se desliza por el hilo hacia el orgen, por accin de su propio peso
y sin rozamiento. Considere un sistema de coordenadas (x, y) donde y representa la altura. Si la forma del
hilo viene dada por una funcin y = y(x), el tiempo total de descenso ser una funcin T (y), de la altura
123

y. El problema mecnico de Abel es el inverso: dada la funcin T (y), encuentre la curva que determina
la forma del hilo que produce un tiempo de descenso igual a T (y). (Este problema puede ser bastante
difcil. Vea SIMMONS, ECUACIONES DIFERENCIALES con aplicaciones y notas histricas, seccin
52, pgina 418)

4. Hallar la ecuacin de la curva de descenso si T (y) = k y


5. El tanque mezclador
h dei la figura contiene inicialmente 500 litros de una solucin salina, con una concentracin de sal de 2 Kg
Lt . Durante los primeros 10 minutos de operacin, se abre la vlvula A, aadiendo
h i
Lt
12 min de una solucin con una concentracin de sal de 4 Kg
. Despus de 10 minutos se cambia a
h i Lt

Lt
Lt
la vlvula B, la cual agrega una concentracin de 6 Kg
a 12 min
. La vlvula C elimina 12 min
,
Lt
manteniendo constante el volumen. Determine la concentracin de sal en el tanque como funcin del
tiempo.

6. En
el problema 1 suponga, ahora, que slo se mantiene abierta la vlvula A y la vlvula C slo elimina
Lt
6 min
. Resuelva el problema.

7. Considere una masa de 1 [Kg] unida a un resorte con constante k = 4, al cual se le aplica una fuerza
externa f (t) = sen (3t) por 10 minutos y despus se elimina. Si el resorte parte del reposo en su posicin
de equilibrio, determine el desplazamiento del resorte en cualquier instante.

124

Parte IV: Sistemas de Ecuaciones Diferenciales.


1

Mtodos Bsicos.

Hasta ahora se ha estudiado ecuaciones diferenciales que involucran una variable independiente y una dependiente. Sin embargo, en las aplicaciones ocurren que una variable dependiente depende de ms de una variable
independiente, como se muestra en el siguiente ejemplo:

Ejemplo 1 Suponga una partcula de masa m que se mueve en el plano XY debido a una fuerza F = F (t, x, y)
que acta sobre ella. (es decir, la direccin de la fuerza y su magnitud dependen solamente de la posicin de
la partcula y del tiempo t) Suponga que inicialmente la partcula est en un punto cualquiera, el origen, en
reposo. Es posible realizar la siguiente pregunta: Cul es la posicin de la partcula en un instante posterior?

Solucin: Descomponga la fuerza F en sus componentes positivas F1 y F2 como se indica en la figura:

Figura 27: Descomposicin de fuerzas de F


De la segunda ley de Newton, se tiene:
d2 x
dt2
d2 y
m 2
dt

= F1 (t, x, y)
= F2 (t, x, y)

Adems
x (0) = y (0) = x0 (0) = y 0 (0) = 0
Para resolver este problema hay varios mtodos, en esta monografa consideraremos tres de ellos, dos que
se refieren a las tcnicas anteriormente estudiadas para resolver sistemas de ecuaciones, y una ms propio de la
teora de Matrices que estudiaremos con ms detencin.
125

i. Mtodo de Eliminacin
En el ejemplo supongamos que m = 1, y
F1 (t, x, y) = y e2t
F2 (t, x, y) = x + 3e2t
entonces
x00
y 00

= y e2t
= x + 3e2t

(1)
(2)

As de (1) se tiene
= x00 + e2t
= x000 2e2t
= x(4) + 4e2t

y
y0
y 00
reemplazando en (2), se tiene

d4 x
+ 4e2t = x + 3e2t
dt4
d4 x
x = e2t
dt4
as
luego

m4 1 = 0 m2 1 m2 + 1 = 0 m1 = 1, m2 = 1, m3 = i, m4 = i
xh = C1 et + C2 et + C3 cos (t) + C4 sen (t)

Y, por anuladores o coeficientes indeterminados:


xp = Ae2t xp =

1 2t
e
15

Luego
x (t) = C1 et + C2 et + C3 cos (t) + C4 sen (t)

1 2t
e
15

y reemplazando en (1) se tiene:


C1 et + C2 et C3 cos (t) C4 sen (t)

4 2t
e
15

= y e2t

y (t) = C1 et + C2 et C3 cos (t) C4 sen (t) +


Para determinar las constantes C1 , C2 , C3 y C4 se utilizan las condiciones iniciales:
1
=0
x (0) = C1 + C2 + C3
15
11
y (0) = C1 + C2 C3 +
=0
15
2
x0 (0) = C1 C2 + C4 +
=0
15
22
y 0 (0) = C1 C2 C4
=0
15
126

11 2t
e
15

de donde
C1 =

1
1
2
4
, C2 = , C3 = , C3 =
6
2
5
5

Luego
et et 2
4
1

+ cos (t) sen (t) e2t


6
2
5
5
15
et et 2
4
11
y (t) =

cos (t) + sen (t) + e2t


6
2
5
5
15
Representaciones grficas de las soluciones son las siguientes: primero las soluciones por separado:
x (t) =

ahora ambas soluciones determinando una curva paramtrica:

Otra idea, equivalente al mtodo de eliminacin, es la Triangulizacin con operadores:


Resolver

4
1
x0 = 50
x + 50
y
, x (0) = 25, y (0) = 0
4
4
y 0 = 50
x 50
y

reescribir como

4
1
x y
50
50
4
4
x + Dy + y
50
50
Dx +

127

= 0
= 0

o, mejor an:

Multiplicando (1) por

4
50

4
1
D+
x y = 0 (1)
50
50

4
4
x+ D+
y = 0 (2)
50
50

4
y (2) por D + 50
, se tiene

4
4
4
D+
x
y = 0 (1.1)
50
50
5050

2
4
4
4
y = 0 (2.1)

D+
x+ D+
50
50
50

sumando (1.1) y (2.1) se tiene

4
4
4
D+
x
y
50
50
5050

2
4
4
D+
y
y
50
5050

= 0 (1.2)
= 0 (2.2)

Resolvemos (2.2):

D2 +

8
16
4
D+
y
y
50
5050
5050

8
12
D2 + D +
y
50
5050

ecuacin auxiliar
8
16
m + m+
50
5050

50

64
5050

= 0
= 0

48
5050

0m=
2
r

3
4
1
4
2
4 2
16
x1 = 25

=
=
=
1
x2 = 25
50 2 5050
50 50
50
6

y (x) = C1 e 50 t + C2 e 50 t
reemplazando en (2)

6
2
4
4
x+ D+
C1 e 50 t + C2 e 50 t
50
50
6
2
6
2
C1 e 50 t C2 e 50 t
50
50
4
x
50
4
x
50

= 0

4
4C1 6 t 4C2 2 t
x
e 50
e 50
50
50
50
6
2
2
4C1 6 t 4C2 2 t
6
=
C1 e 50 t + C2 e 50 t
e 50
e 50
50
50
50
50
6
2
2
2
=
C1 e 50 t C2 e 50 t
50
50
6
2
1
1
50
t
x = C1 e
+ C2 e 50 t
2
2
=

128

por lo tanto
6
2
1
1
x (t) = C1 e 50 t + C2 e 50 t
2
2
6
2
y (x) = C1 e 50 t + C2 e 50 t

adems
y (0) = 0, x (0) = 25
1
1
C1 + C2 = 25
2
2
C2 + C1 = 0
luego
C1
C2

= 25
= C1

por lo tanto
25 6 t 25 2 t
e 50 e 50
2
2
6
2
y (x) = 25e 50 t 25e 50 t
x (t) =

ii. Mtodo de la transformada de Laplace


Para mostrar la forma en que acta este mtodo, utilizaremos el siguiente ejemplo:
1. Ejemplo 2 Resolver el sistema de ecuaciones diferenciales
dx
dt
dy
dt

= x (t) + y (t)
= 4x (t) 2y (t)

Se puede aplicar la transformada de Laplace para convertir el sistema en un sistema algebraico. Anotemos
x (0+ ) = C1 , y (0+ ) = C2 .

dx
L
(s) = L (x (t)) (s) + L (y (t)) (s)
dt

dy
L
(s) = 4L (x (t)) (s) 2L (y (t)) (s)
dt
anotando X (s) = L (x (t)) (s) y Y (s) = L (y (t)) (s), se tiene:
sX C1
sY (s) C2

= X (s) + Y (s)
= 4X (s) 2Y (s)
129

(s 1) X (s) Y (s) = C1
4X (s) + (s + 2) Y (s) = C2

sumando

4 (s 1) X (s) 4Y (s) = 4C1

4 (s 1) X (s) + s2 + s 2 Y (s) = (s 1) C2

2
s + s 6 Y (s) = 4C1 + (s 1) C2
4
s1
Y (s) = C1 2
+ C2 2
s +s6
s +s6

reemplazando en la primera ecuacin:


X (s) = C1

1
4
1
+ C1
+ C2 2
s1
(s 1) (s2 + s 6)
s +s6

Luego, calculamos las transformadas inversas:


h
i
1
1. (a) L1 s2 +s6
es:

1
1
1
1
L
= L
s2 + s 6
(s + 3) (s 2)

1
1
= L1 5 + 5
s+3 s2
1 3t 1 2t
+ e
= e
5
5
h
i
(b) L1 s2s1
+s6 es:

s1
s1
1
1
L
= L
s2 + s 6
(s + 3) (s 2)

s2+1
1
= L
(s + 3) (s 2)

1
1
1
1
= L
+L
s+3
(s + 3) (s 2)
1
1
= e3t e3t + e2t
5
5
h
i
(c) L1 (s1)(s12 +s6) es:

1
1
1
L1
=
L
(s 1) (s2 + s 6)
(s 1) (s 2) (s + 3)
1
1
1
4
1
5
20
+
+
= L
s1 s2 s+3
1
1
1
= et + e2t + e3t
4
5
20
130

Por lo tanto

4 2t
1
1
e e3t + C2 e3t e3t + e2t
5
5
5
4
1
(4C1 + C2 ) e2t + (C2 C1 ) e3t
5
5

y (t) = C1
y (t) =

4
1
1
x (t) = C1 e2t + C1 e3t + C2 e2t e3t
5
5
5
1
1
2t
x (t) =
(4C1 + C2 ) e + (C1 C2 ) e3t
5
5
1
1
y anotando a = 5 (4C1 + C2 ) ; b = 5 (C1 C2 ) se obtiene
x (t) = ae2t + be3t
y (t) = ae2t 4be3t

Ejemplo 3 Resolver
x00
y 00

= yx
= xy

con condiciones iniciales x (0) = 0, x0 (0) = 2, y (0) = 0, y 0 (0) = 1.


Aplicando la transformada de Laplace, se tiene
s2 L [x] s0 + 2 = L [y] L [x]
s2 L [y] s0 1 = L [x] L [y]
o

multiplicando (2) por s2 + 1

sumando

s2 + 1 L [x] L [y] = 2

L [x] + s2 + 1 L [y] = 1

2
s + 1 L [x] L [y] = 2

s2 + 1 L [x] + s2 + 1 L [y] = 1

2
s2 + 1 1 L [y] = s2 1
4

s + 2s2 + 1 1 L [y] = s2 1

4
s + 2s2 L [y] = s2 1
L [y] =

L [y] =
L [y] =
131

(1)
(2)

(1.1)
(2.1)

s2 1
s2 (s2 + 2)
s2 + 2
3
2 2
2
2
s (s + 2) s (s + 2)
3
1

s2 s2 (s2 + 2)

reemplazando en (2) se tiene


L [x] =
L [x] =
L [x] =
L [x] =
L [x] =

s + 1 L [y] 1

s2 1
s +1
1
s2 (s2 + 2)
s4 1
1
2
s (s2 + 2)
2s2 + 1
s4 1 s4 2s2
= 2 2
2
2
s (s + 2)
s (s + 2)
2
1
2

s + 2 s2 (s2 + 2)

Calculando las fracciones parciales:


Cs + D
n
A
B
= + 2+ 2
s2 (s2 + 2)
s
s
s +2
luego

As s2 + 2 + B s2 + 2 + (Cs + D) s2
3

As + 2As + Bs + 2B + Cs + Ds

= n
= n

luego tenemos el sistema


A+C
B+D
2A
2B
luego: A = 0, B =

n
2,C

=
=
=
=

0
0
0
n

= 0, D = n2 , luego
2
1 1
1
2
1 1
+
2 2
= 2

2
+ 2 s (s + 2)
s +2 2s
2 s2 + 2
3
3 1
3 1
1
1

+
= 2
s2 s2 (s2 + 2)
s
2 s2 2 s2 + 2

L [x] =
L [y] =

s2

por lo tanto

t
3 2
sen (2t)
x (t) =
4
2
t
3 2
y (t) = +
sen (2t)
2
4

Representaciones grficas de las soluciones son las siguientes: primero las soluciones por separado:
132

ahora ambas soluciones determinando una curva paramtrica:

Considere ahora el siguiente problema:


Ejemplo 4 El tanque A contiene 50 galones de agua en los cuales se disuelven 25 libras de sal. Un segundo
tanque B contiene 50 galones de agua pura. Se bombea lquido hacia y desde los tanques, en las proporciones
indicadas en la figura. Determine las cantidades de sal en ambos tanques en el instante t.

Figura 28: Sistema de Estanques Interconectados

133

Sean x (t) , y (t) las cantidades de sal en A y B respectivamente en el instante t. Observe que los volumenen
de solucin en A y B permanecen constantes:
A : VA (t) = 50 + 3t + t 4t = 50 [Galones]
B : VB (t) = 50 + 4t t 3t = 50 [Galones]
Ahora bien:
dx
dt
dy
dt




G
lb
G y (t)
G x (t)
0
+1

m
G
m
50
m
50



G x (t)
G y (t)
G y (t)
= entra sale = 4

m
50
m
50
m
50

= entra sale = 3

Por lo tanto
dx
dt
dy
dt

=
=

y (t)
4
x (t)
50
50
4
4
x (t) y (t)
50
50

o bien
4
1
x+ y
50
50
4
4
x y
50
50

x0

y0

adems x (0) = 25, y (0) = 0.


Para resolver este sistema se puede aplicar la transformada de Laplace:
4
1
L [x] + L [y]
50
50
4
4
L [x] L [y]
50
50

L [x0 ] =
L [y 0 ] =

Anotemos X = L [x] , Y = L [y]:


4
1
sL [x] x 0+ = L [x] + L [y]
50
50

4
4
L [x] L [y]
sL [y] y 0+ =
50
50
1
4
X+ Y
50
50
4
4
X Y
50
50

sX 25 =
sY 0 =

1
4
X Y
s+
50
50

4
4
X + s+
Y
50
50
134

= 25
= 0

multiplicando ambas ecuaciones por 50 se tiene:


(50s + 4) X Y
4X + (50s + 4) Y

= 1250
= 0

Multiplicando la primera por (50s + 4) se tiene


(50s + 4)2 X (50s + 4) Y
4X + (50s + 4) Y

= 1250 (50s + 4)
= 0

h
i
2
(50s + 4) 4 X

= 1250 (50s + 4)

sumando ambas ecuaciones:

[(50s + 4) 2] [(50s + 4) + 2] X
(50s + 2) (50s + 6) X
X

= 1250 (50s + 4)
= 1250 (50s + 4)
1250 (50s + 4)
=
(50s + 2) (50s + 6)

Por otra parte:


Y

4
1250 (50s + 4)
4X
=

50s + 4
50s + 4 (50s + 2) (50s + 6)
5000
(50s + 2) (50s + 6)

luego x = L1 [X] , y = L1 [y] :


X

1250 (50s + 4)
(50s + 2) + 2
= 1250
(50s + 2) (50s + 6)
(50s + 2) (50s + 6)

1
2
= 1250
+
50s + 6 (50s + 2) (50s + 6)

1
2
= 1250
+
50s + 6 (50s + 2) (50s + 6)
"
#
1
2
1
1

= 1250

6 + 502
2
6
50 s + 50
s + 50
s + 50
=

1
1
2

6 + 2
2
6
s + 50
s + 50 s + 50
1
1

= 25
6 +
2
6
s + 50
s + 50 s + 50
1
A
B
= 25
6 +
2 +
6
s + 50
s + 50
s + 50

= 25

135

resolviendo la suma de fracciones parciales


As +

6
2
A + Bs + B
50
50
A+B

= 1
6
2
A+ B =1
50
50
2
4
6
A A=1
A=1
50
50
50
50
25
25
=
, B=
4
2
2

= 0

A =
Por lo tanto
X
Y

1
25
25
1
1
6 + 2
2 2
6
s + 50
s + 50
s + 50
5000
1
=
= 5000 2
(50s + 2) (50s + 6)
50 s +

1
1
= 25
2
6
s + 50
s + 50
= 25

2
50

s+

6
50

luego
x (t) =

1
25 3 t
e 25 + e 25 t
2

y (t) = 25 e 25 t e 25 t

Representaciones grficas de las soluciones son las siguientes: primero las soluciones por separado:

ahora ambas soluciones determinando una curva paramtrica:

136

1.1

Ejercicios

1. Determine la solucin general de cada uno de los siguientes sistemas de ecuaciones diferenciales.

dx/dt
= 3x y
(a)
dy/d(t) = 9x 3y

dx/dt = 6x + 5y
(b)
dy/d(t) = 5x + 4y

dx/dt = 12x 9y
(c)
dy/d(t) =
4x
0
x = 3x + 2y + 4z
y 0 = 2x + 2z
(d)
0
z = 4x + 2y + 3z

2. Resuelva los siguientes sistemas de ecuaciones aplicando la transformada de Laplace a cada ecuacin para
transformar el sistema en un sistema de ecuaciones algebraicas.
0
x (t) = x(t) + 1
y 0 (t) = 2x(t)
(a)
x(0) = 0, y(0) = 1
0
x = 2y + et
y 0 = 8x t
(b)
x(0) = 1, y(0) = 1
0
x + 3x + y 0
= 1
x0 x + y 0 y = et
(c)
x(0) = 0, y(0) = 0
00
x + x0 + y 0
= 0
y 00 + y 0 4x0 = 0
(d)
x(0) = 1 , x0 (0) = 0
y(0) = 1 , y 0 (0) = 5
137

(e)

x0 4x + y (3)
= 6sen(t)
x0 + 2x 2y (3) = 0
x(0) = 0 , y(0) = 0
y 0 (0) = 0 , y 00 (0) = 0

Sistemas Lineales Homogneos.

Consideremos el sistema general de 2 ecuaciones diferenciales de primer orden con coeficientes constantes:
dx
dt
dy
dt
Si
U (t) =

x (t)
y (t)

= a11 x (t) + a12 y (t)

(2.1)

= a21 x (t) + a22 y (t)

, U (t) =

dx
dt
dy
dt

, A=

a11
a21

a12
a22

entonces el sistema (2.1) puede expresarse como


U 0 = AU
(2.2)

x1 (t)
x2 (t)
Observacin 1 Si U1 (t) =
y U2 (t) =
son soluciones de (2.1), y por lo tanto de (2.2),
y1 (t)
y2 (t)
entonces
C1 U1 (t) + C2 U2 (t)
tambin es solucin de (2.1) (y de (2.2)). Luego

x (t)
0
U (t) =
U es solucin de U = AU
y (t)
es un espacio vectorial.

x2 (t)
Observacin 2 Sean U1 (t) =
y U2 (t) =
:
y2 (t)

x1 (t) + x2 (t)
x1 (t) x2 (t)

U1 (t) + U2 (t) =
=
y1 (t) + y2 (t)
y1 (t) y2 (t)

x1 (t) x2 (t)
por lo tanto U1 y U2 son linealmente independientes si y slo si existe t tal que det
6= 0
y1 (t) y2 (t)
x1 (t)
y1 (t)

Definicin 1 Si U1 y U2 son soluciones de (2.1) (y de (2.2)) el Wronskiano de U1 y U2 es el determinante

x1 (t) x2 (t)
W [U1 (t) , U2 (t)] = det
y1 (t) y2 (t)

138

Observacin 3 Se verifica que


d
[x1 y2 x2 y1 ]
dt
0
x1 y2 + x1 y20 x02 y1 x2 y10
(a11 x1 + a12 y1 ) y2 + x1 (a21 x2 + a22 y2 ) (a21 x1 + a22 y1 ) x2 y1 (a11 x2 + a12 y2 )
a11 x1 y2 + a12 y1 y2 + a21 x1 x2 + a22 x1 y2 a21 x1 x2 a22 y1 x2 a11 y1 x2 a12 y1 y2
(a11 + a22 ) x1 y2 (a11 + a22 ) y1 x2
(a11 + a22 ) (x1 y2 y1 x2 )

d
W [U1 (t) , U2 (t)] =
dt
=
=
=
=
=
d
W [U1 (t) , U2 (t)] =
dt

(a11 + a22 ) W [U1 (t) , U2 (t)]

y entonces

W [U1 (t) , U2 (t)] = Ce

(a11 (t)+a22 (t))dt

Conclusin 1 U1 , U2 son linealmente independientes s y slo si t, W [U1 (t) , U2 (t)] 6= 0 y U1 , U2 son linealmente dependientes s y slo si t, W [U1 (t) , U2 (t)] = 0
Observacin 4 Considere la ecuacin de 2o orden lineal
y 00 + ay 0 + by = 0
defina x1 = y, x01 = x2 = y 0 , x02 = y 00 = ay 0 by, se obtiene el sistema de ecuaciones
x01
x02
o

note que si U =

x1 (t)
x2 (t)

0
1
b a

= x2
= ax2 bx1

x1
x2

x01
x02

(i)

es solucin de (i), entonces


x02
x02
x001

se concluye que x1 = y es solucin de

= ax01 bx1 y x01 = x2


= x001
= ax01 bx1
y 00 + ay 0 + by = 0

Ejercicio 1 Si {y1 , y2 } son soluciones linealmente independientes de y 00 +ay 0 +by, entonces U1 =

y2
son soluciones de (i)
y20

139

y1
y10

, U2 =

y1
y2
=
,
U
son soluciones linealmente independiente de (i), entonces {y1 , y2 }
2
y10
y20
son soluciones linealmente independientes de y 00 + ay 0 + by = 0, pues W [U1 , U2 ] = W [y1 , y2 ] . (Esto justifica la
escritura anterior del Wronskiano).
En efecto, si U1 =

Teorema 2 El espacio solucin del sistema de ecuaciones diferenciales de primer orden:


U 0 = AU
o
dx
dt
dy
dt

= a11 x (t) + a12 y (t)

(2.3)

= a21 x (t) + a22 y (t)

tiene dimensin 2, esto es, si U1 , U2 son soluciones linealmente independientes, entonces cualquier solucin se
expresa como
U = C1 U1 (t) + C2 U2 (t) , C1 , C2 R
Problema: Determinar la solucin general de:
U 0 = AU

(2.2)

Note que esta ecuacin es, al menos en apariencia, muy similar a la ecuacin
y 0 (x) = ay (x)
cuya solucin es
y (x) = eax
Nos preguntamos si existirn soluciones de (2.2) de la forma

mt
a
ae
mt
=
e
U=
b
bemt
Reemplazando en (2.2):

se obtiene

amemt
bmemt

a11 aemt + a12 bemt


a21 aemt + a22 bemt

a (a11 m) emt + a12 bemt


aa12 emt + b (a22 m) emt

= 0
= 0

y dividiendo por emt


a (a11 m) + a12 b = 0
aa12 + b (a22 m) = 0
El problema es determinar a, b y m. Observe que si a = 0 = b, entonces U =
sistema (2.4). Para determinar una solucin no nula por el teorema:
140

(2.4)

0
0

es una solucin obvia del

Teorema 3 Sea A una matriz de orden m n, las siguientes afirmaciones son equivalentes
i. A es singular (no tiene inversa).
ii. det (A) = 0
iii. AX = 0 tiene solucin no trivial
iv Las columnas (filas) de A forman un conjunto linealmente dependiente.
se requiere
det

a11 m
a12
a21
a22 m

(a11 m) (a22 m) a21 a12


m (a11 + a22 ) m + a11 a22 a21 a12
2

= 0
= 0
= 0

Esta ecuacin es llamada la ecuacin auxiliar del sistema (2.4)


Note que esta ecuacin es exactamente:

a11 a12
1 0
det (A mI) = 0, A =
, I=
0 1
a21 a22
Observacin 5 La ecuacin (2.4) puede expresarse


a
0
(A mI)
=
b
0

a
Si anotamos V =
, entonces m y V satisfacen
b
(A mI) V
AV

= 0
= mV

El nmero m se denomina valor propio de A y el vector V , vector propio de A asociado a m. Por esta razn
el mtodo se denomina de los valores y vectores propios: "Si m es valor propio de A y V es un vector propio
asociado a m, entonces X = emt V es una solucin de la ecuacin X 0 = AX".

2.1

Mtodo de solucin de U 0 = AU.

Si m1 y m2 son las soluciones de det (A mI), entonces, reemplazando m1 en (2.4) se puede determinar
a = a1 , b = b1 tales que a1 , b1 , m1 satisfacen (2.4) y por lo tanto

a1 em1 t
U1 =
b1 em1 t
es una solucin de U 0 = AU.

141

En forma anloga, reemplazando m2 en (2.4) se determinan a = a2 , b = b2 tales que a2 , b2 , m2 satisfacen


(2.4) y por lo tanto

a2 em2 t
U2 =
b2 em2 t
es una solucin de U 0 = AU.
Al aplicar este mtodo puede ocurrir cualquiera de las posibilidades siguientes:

i. det (A mI) = 0 tiene races reales distintas: m1 6= m2 . En estas condiciones es fcil ver que las soluciones

a1 em1 t
a2 em2 t
U1 =
=
,
U
2
b1 em1 t
b2 em2 t
son linealmente independientes y la solucin general del sistema es

C1 a1 em1 t + C2 a2 em2 t
U = C1 U1 (t) + C2 U2 (t) =
, C1 , C2 R
C1 b1 em1 t + C2 b2 em2 t
Por lo tanto
x (t) = C1 a1 em1 t + C2 a2 em2 t
y (t) = C1 b1 em1 t + C2 b2 em2 t , C1 , C2 R
Ejemplo 5 Resolver

El sistema se puede escribir como: U 0 =

dx
dt
dy
dt

= x+y
= 4x 2y

1 1
4 2

U, U =

x (t)
y (t)

La ecuacin auxiliar es:

1 1
1 0
1m
1
det
m
= det
=0
4 2
0 1
4
2 m

(1 m) (2 m) 4 = 0
m2 + m 2 4 = 0
m2 + m 6 = 0, o bien (m + 3) (m 2) = 0

luego, tenemos: m1 = 3, m2 = 2.
a) Para m1 = 3, reemplazando en (2.4)
(1 m) a + 1b = 0
4a (2 + m) b = 0
se tiene:
4a + b = 0
4a + b = 0
142

luego
4a + b = 0 b = 4a
es decir:
U1 (t) =
U1 (t) =

aem1 t
bem1 t

e3t
4e3t

ae3t
4ae3t

=a

e3t
4e3t

, luego para a = 1

b. Para m2 = 2, reemplazando en (2.4) se tiene


a + b = 0
4a 4b = 0
entonces a = b, luego

aem2 t
bem2 t
2t
e
U2 (t) =
e2t

U2 (t) =

ae2t
ae2t

=a

e2t
e2t

, luego para a = 1

Por lo tanto la solucin general del sistema dado es


U = C1 U1 (t) + C2 U2 (t)
por lo tanto
x (t) = C1 e3t + C2 e2t
y (t) = 4C1 e3t + C2 e2t , C1 , C2 R
Representaciones grficas de las soluciones son las siguientes: primero las soluciones por separado:

ahora ambas soluciones determinando una curva paramtrica:

143

Algunas curvas solucin para diferentes condiciones iniciales:

(Compare con el ejemplo del mtodo de la Transformada de Laplace)


Ejemplo 6 Resolver
4
1
x+ y
50
50
4
4
x y
50
50

x0

y0

Escribiendo el sistema en forma matricial se tiene

1
4
x
x
50
50
=
4
4
y

y0
| 50 {z 50 }
A

144

luego

2
4
4
=
+m 2
0 = det (A mI) = det
4
4

m
50
50
50
50

2
4
4
4
2
4
2
50

+m = 2
+m=
m=
=
6
50
50
50
50
50
50 50
6
2
m1 = , m2 =
50
50

4
m
50

1
50

6
:
a) Vector propio para m1 = 50

4
50
+
4
50

6
50

(A m1 I) V = 0

1
a1
0
50
=
4
6
0
b1
50
+ 50

2
1
2
50 a1 + 50 b1 = 0
b1 = 50 a1 = 2a1
4
2
50
50 a1 + 50 b1 = 0

6
6
m1 t
a1 e 50 t
e 50 t
a1 e
=
=
a
, luego para a1 = 1
U1 (t) =
6
6
1
b1 em1 t
2a1 e 50 t
2e 50 t

6
e 50 t
U1 (t) =
6
2e 50 t

2
b. Vector propio para m2 = 50
:

4
50
+
4
50

2
50

(A m2 I) V = 0

1
a2
0
50
=
4
2
0
50
+ 50
b2

2
1
50 a2 + 50 b2 = 0
b2 = 2a2
4
2
50 a2 50 b2 = 0

2t
2
a2 em1 t
a2 e 50 t
e 50
U2 (t) =
=
= a2
, luego para a2 = 1
2
2
b2 em1 t
2a2 e 50 t
2e 50 t
2t
e 50
U2 (t) =
2
2e 50 t

Por lo tanto
6

x (t) = C1 e 50 t + C2 e 50 t
6

y (t) = 2C1 e 50 t + 2C2 e 50 t


ii. det (A mI) = 0 tiene dos races reales iguales: m1 = m2 = m R.
Determinamos un vector propio V1 asociado a m y entonces U1 = emt V1 es una solucin del sistema U 0 = AU .
Para determinar una segunda solucin, buscamos una solucin U2 de la forma
U2 = (W2 + W1 t) emt
145

Para determinar W1 y W2 , suponga que U2 satisface la ecuacin U 0 = AU, esto es:


U20
0
(W2 + W1 t) emt + (W2 + W1 t) memt
W1 + mW2 + mW1 t
entonces, W1 + mW2
AW1

=
=
=
=
=

AU2 , o bien
(AW2 + AW1 t) emt , cancelando emt y resolviendo la derivada se tiene
AW2 + AW1 t
AW2 y mW1 = AW1 , es decir
mW1 y W1 = (A mI) W2

por tanto, W1 es un vector propio de A asociado a m, entonces W1 = V1 y W2 es un vector que satisface


(A mI) W2 = V1
en conclusin, si det (A mI) = 0 tiene dos races reales iguales, digamos m1 = m2 = m, entonces una base
para el espacio solucin de U 0 = AU est dada por
U1
U2

= emt V1
= (W2 + V1 t) emt

donde V1 es el vector propio asociado a m y W2 es un vector que satisface (A mI) W2 = V1 y se le denomina


vector propio generalizado de m
Ejemplo 7 Determine la solucin general de
x0
y0

= 3x 4y
= xy

El problema puede escribirse tambin como

3 4
x
0
U =
U, donde U =
1 1
y

3 4
La matriz A =
tiene valores propios: det (A mI) = 0 :
1 1

3m
4
det
= 0
1
1 m

(3 m) (1 m) + 4 = 0
3 2m + m2 + 4 = 0
2
m2 2m + 1 = 0 (m 1) = 0
m = m1 = m2 = 1

El vector propio asociado a m = 1 :

AV1 = 1V1 (A I) V1 = 0


31
4
a
0
=
2a 4b = 0 a = 2b
1
1 1
b
0


a
2b
2
V1 =
=
=b
,
b
b
1

2
considerando b = 1, se tiene V1 =
1

146

Luego U1 = emt V1 = et

2
1

. Para determinar U2 , se necesita determinar W2 , tal que


(A mI) W2 = V1

como m = 1, entonces

2
1

U = C1 U1 (t) + C2 U2 (t) =

C1 2et + C2 (1 + 2t) et
C1 et + C2 tet

2 4
1 2

luego U2 = (W2 + tV1 ) emt

c
d

c 2d = 1 c = 1 + 2d



c
1 + 2d
1
2
=
=
+d
,
W2 =
d
d
0
1

1
si d = 0, entonces W2 =
0

1
2
=
+
t et , es decir:
0
1

(1 + 2t) et
U2 =
tet

y la solucin general es:

x (t)
y (t)

esto es
x (t) = (2C1 + C2 + 2C2 t) et
y (t) = (C1 + C2 t) et , C1 , C2 R
Representaciones grficas de las soluciones son las siguientes: primero las soluciones por separado:

ahora ambas soluciones determinando una curva paramtrica:

147

Algunas curvas solucin para diferentes condiciones iniciales:

Ejemplo 8 Resolver
U0 =

3 18
2 9

Los valores propios son:


(3 m) (9 m) + 36
27 + 6m + m2 + 36
m2 + 6m + 9
(m + 3)2
vector propio asociado:

6 18
a
2 6
b

0
0

V1 =

=
=
=
=

0
0
0
0 m1 = 3 = m2

2a 6b = 0 a = 3b



a
3b
3
3
=
=b
, para b = 1, V1 =
b
b
1
1
148

Por lo tanto U1 =

3
1

e3t , para determinar U2 se tiene que

1
2c 6d = 1 c = + 3d
2

1
1
1

c
3
2 + 3d
2
2
=
W2 =
=
+d
, para d = 0, W2 =
d
1
d
0
0
1
3
2
+
t e3t y la solucin general es
por tanto U2 =
1
0

1
3
3
3t
2
+ C2
e
+
t e3t , C1 , C2 R
U (t) = C1
1
0
1
6 18
2 6

c
d

3
1

x (t) =

1
3C1 + C2
+ 3t
e3t
2

y (t) = (C1 + C2 t) e3t , C1 , C2 R


Representaciones grficas de las soluciones son las siguientes: primero las soluciones por separado:

ahora ambas soluciones determinando una curva paramtrica:

149

Algunas curvas solucin para diferentes condiciones iniciales:

iii. det (A mI) = 0 tiene races complejas m1 = a + bi, m2 = a bi, entonces la solucin es de la forma
Ui (t) = Vi (t) emi t , i = 1, 2
Observacin
6 Recuerde que si m = a + ib, entonces m = a bi es el complejo conjugado de m. Si V =

u1
..
. es una matriz columna donde cada ui es un nmero complejo, se anota V la matriz conjugada de
un

u1

V : V = ... . Suponga que m = a + ib es un valor propio complejo de la matriz A de orden n n con


un
coeficientes reales y que V sea un vector propio asociado al valor propio m. Entonces m es un vector propio de
A y adems V es un vector propio asociado a m, en efecto:

AV = mV AV = mV AV = mV
Ahora bien, es posible preguntarse si, como en el caso real, las expresiones
U1 = emt V y U2 = emt V
representan soluciones del sistema de ecuaciones diferenciales de primer orden
U 0 = AU
Observe que en este contexto, las expresiones
U10 = AU1 , U20 = AU2
carecen de sentido, ya que U1 y U2 son expresiones que asumen valores en el plano complejo y no tiene ningn
sentido en este curso las expresin U10 , U20 . Sin embargo, es posible preguntarse por alguna combinacin conveniente de las expresiones U1 y U2 determinen una expresin real que si sea solucin de U 0 = AU . Para esto
recuerde que
ea+ib = ea (cos (b) + isen (b))
150

y considere:
Y1
Y2

= emt V = eat (cos (bt) + isen (bt)) V


= emt V = eat (cos (bt) isen (bt)) V

entonces

V + V cos (bt) + i V V sen (bt)


i (Y1 Y2 ) = eat i V V cos (bt) V + V sen (bt)

son funciones reales de la variable t, ya que V + V = 2 Re (V ), i V V = 2 Im (V ) donde Re (V ) =

Im (u1 )
Re (u1 )

..
..
e Im (V ) =
, V = Re (V ) + i Im (V ). Observe que

.
.
Y1 + Y2

Re (un )

= eat

Im (un )

AV

= A (Re (V ) + i Im (V )) = AV = mV = (a + bi) (Re (V ) + Im (V ))


= (a Re (V ) b Im (V )) + i (b Re (V ) + a Im (V ))

se concluye que
A Re (V ) = a Re (V ) b Im (V )
A Im (V ) = b Re (V ) + a Im (V )
Proposicin
reales de ordenn n, y sea

1 Sea m = a + ib un valor propio de la matriz A con coeficientes

Im (u1 )
Re (u1 )
u1

..
..
V = ... un vector propio asociado al valor propio m, si Re (V ) =
e Im (V ) =
,
.
.
un
Re (un )
Im (un )
entonces las funciones
Z1 (t) = eat (Re (V ) cos (bt) Im (V ) sen (bt))
Z2 (t) = eat ( Im (V ) cos (bt) Re (V ) sen (bt))
son soluciones del sistema de ecuaciones diferenciales de primer orden U 0 = AU , en general cualquier combinacin lineal Z = C1 Z1 + C2 Z2 , con C1 , C2 R es solucin de U 0 = AU
Por lo tanto, entonces,
U1 (t) = eat (Re (V ) cos (bt) Im (V ) sen (bt))
U2 (t) = eat (Im (V ) cos (bt) + Re (V ) sen (bt))
son las soluciones del sistema de ecuaciones U 0 = AU , y la solucin general es:
U = C1 U1 (t)+C2 U2 (t) = C1 eat (Re (V ) cos (bt) Im (V ) sen (bt))+C2 eat (Im (V ) cos (bt) + Re (V ) sen (bt)) , C1 , C2 R
Ejemplo 9 Resolver el sistema
y1 2y2
5y1 y2
151

= y10
= y20

Reescribiendo el sistema en forma matricial:


0

y1
1 2
y1
=
5 1
y2
y20

1 2
luego, los valores propios de
son
5 1

1m
2
0 = det
= (1 m) (1 + m) + 10 = 1 m2 + 10 = m2 + 9 m = 3i
5
1 m
Los vectores propios asociados a m1 = 0 + 3i :

1 2
a
a
= 3i
, o bien
5 1
b
b
a 2b = 3ia
5a b = 3ib

de donde 2b = (1 3i) a, o bien b = 12 (1 3i) a. Por tanto

a
a
1
=
V1 =
=
a
, para a = 1
1
1
3
b
2 (1 3i) a
2 2i


1
0
1
V1 =
+
i
=
1
3
1
32
2 2i
2
es un vector propio asociado a m1 = 0 + 3i.
Para el caso m2 = 0 3i :

1 2
a
a
= 3i
, o bien
5 1
b
b
a 2b = 3ia
5a b = 3ib
de donde 2b = (1 + 3i) a, o bien b = 12 (1 + 3i) a. Por tanto

a
a
1
=
V2 =
=
a
, para a = 1
1
1
3
b
2 (1 + 3i) a
2 + 2i


1
0
1
V2 =
+
i
= V1
=
1
3
1
3
2 + 2i
2
2
es un vector propio asociado a m2 = 0 3i..
Por lo tanto, la solucin del sistema de ecuaciones es:
U = C1 U1 (t)+C2 U2 (t) = C1 eat (Re (V ) cos (bt) Im (V ) sen (bt))+C2 eat (Im (V ) cos (bt) + Re (V ) sen (bt)) , C1 , C2 R
donde
U1 (t) = e0t (Re (V ) cos (3t) Im (V ) sen (3t))
U2 (t) = e0t (Im (V ) cos (3t) + Re (V ) sen (3t))
152

0
2 cos (3t)
cos (3t)
sen (3t) =
3
cos (3t) + 3sen (3t)

0
2
2sen (3t)
U2 (t) =
cos (3t) +
sen (3t) =
3
1
3 cos (3t) + sen (3t)
U1 (t) =

2
1

y la solucin del sistema es


y1
y2

= 2C1 cos (3t) + 2C2 sen (3t)


= C1 (cos (3t) + 3sen (3t)) + C2 (sen (3t) 3 cos (3t)) , C1 , C2 R

Representaciones grficas de las soluciones son las siguientes: primero las soluciones por separado:

ahora ambas soluciones determinando una curva paramtrica:

Algunas curvas solucin para diferentes condiciones iniciales:

153

Ejemplo 10 Considere

2 3
3 2

x
y

x0
y0

, determine las soluciones.

Determinar los valores propios:

2m
3
0 = det
= (2 m)2 + 9 (2 m)2 = 9
3
2m

2 m = 9 2 m = 3i m = 2 3i
Determinar los vectores propios: m = 2 3i


2 (2 3i)
3
a
0
=
3
2 (2 3i)
b
0


3i 3
a
0
=
3 3i
b
0
3ai 3b = 0 ai = b
por tanto:
V1
V1

=
=

a
b
1
i

=
=

a
ai
1
0

=a

0
+i
1

1
, para a = 1
i

y la solucin general del sistema est dada por:


U = C1 U1 (t) + C2 U2 (t) , C1 , C2 R
U1
U2

1
= e
cos (3t)
+ sen (3t)
0

0
2t
= e
cos (3t)
+ sen (3t)
1
2t

154

0
1
1
0

por tanto
x (t) = e2t (C1 cos (3t) + C2 sen (3t))
y (t) = e2t (C1 sen (3t) + C2 cos (3t)) , C1 , C2 R
Representaciones grficas de las soluciones son las siguientes: primero las soluciones por separado:

ahora ambas soluciones determinando una curva paramtrica:

Algunas curvas solucin para diferentes condiciones iniciales:

155

2.2

Sistemas de n n Ecuaciones Diferenciales

Ahora consideraremos sistemas de ecuaciones diferenciales de primer orden con coeficientes constantes:
x01
x02
x0n

= a11 x1 + a12 x2 + ... + a1n xn


= a21 x1 + a22 x2 + ... + a2n xn
..
.
= an1 x1 + an2 x2 + ... + ann xn

o bien, en forma matricial:

U 0 = AU, A = (aij )nn , U =

x1
x2
..
.
xn

Proposicin 2 {U U 0 = AU } es un espacio vectorial de dimensin n.

Definicin 2 Si U1 , ..., Un son soluciones de AU = U 0 , el Wronskiano de U1 , ..., Un es


W [U1 (t) , ..., Un (t)] = det (U1 (t) Un (t))
y, como antes, U1 , ..., Un son linealmente independientes s y slo si W [U1 (t) , ..., Un (t)] 6= 0.
Como en el caso de 2 2, se define
m el valor propio de A si

a1
a2

a m cualquier vector V = . tal que (A mInn ) V =


..

det (A mInn ) = 0 y vector propio asociado

0
.. .
.

0
an
Si m es un valor propio real y V es el vector propio asociado a m, entonces U = V emt es una solucin de
0
U = AU.
156

Definicin 3 Si det (A mI) = (m mi ) q (m), pero (m mi )


que mi es un valor propio de multiplicidad k de la matriz A.

k+1

no es un factor de det (m mi ), se dice

A partir de esta definicin, podemos tener dos situaciones generales para un autovalor propio mi de la matriz
A del sistema de ecuaciones diferenciales:
i. k = 1, es decir, la multiplicidad de mi es uno, luego si Vi es el vector propio asociado a mi y Ui = Vi emi t es
una solucin linealmente independiente de U 0 = AU.
ii. Si k 6= 1, es decir, la multiplicidad de mi es mayor que 1, podemos distinguir tres casos:
a. Existen k vectores propios linealmente independientes Vi,1 , ..., Vi,k asociados a mi , entonces:
Vi,1 emi t , Vi,2 emi t , ..., Vi,3 emi t
son soluciones linealmente independientes de la ecuacin U 0 = AU.
b. Existe un nico vector propio Vi (salvo por multiplicidad por un escalar) asociado al valor propio mi ,
entonces existen vectores W2 , W3 , ..., Wk , tales que:
(A mI) Vi
(A mI) W2
(A mI) W3
(A mI) Wk

= 0,
= Vi ,
= W2 ,
..
.
= Wk1

y los siguientes vectores columnas son soluciones linealmente independientes de U 0 = AU :


Ui,1
Ui,2
Ui,3
Ui,4

Ui,k

= Vi emi t
= (W2 + tVi ) emi t

t2
=
W3 + tW2 + Vi emi t
2!

t2
t3
=
W4 + tW3 + W2 + Vi emi t
2!
3!
..
.

tk2
tk1
tk
=
Wk + tWk1 + +
W3 +
W2 + Vi emi t
(k 2)!
(k 1)!
k!

c. Existen r < k vectores propios linealmente independientes Vi,1 , ..., Vi,r asociados a mi . En este caso, no
podemos utilizar la regla de construccin de vectores del caso anterior, luego no podemos determinar
los k vectores soluciones linealmente independientes de U 0 = AU.
Observacin 7 Si W2 , W3 , ..., Wk son los vectores como en el caso ii.b., entonces
2

0 = (A mI) Vi = (A mI) (A mI) W2 = (A mI) W2 = (A mI) (A mI) W3 = (A mI) W3 =


3

k1

= (A mI) (A mI) W4 = (A mI) W4 = ... = (A mI)

cada Wj se denomina vector propio generalizado asociado al valor propio mi .


157

(A mI) Wk = (A mI) Wk

Ejemplo 11 Resolver

1 2 2
U 0 = 2 1 2 U
2 2 1

Los valores propios son:

1m
2
2

1m
2
det 2

2
2
1m

1m
2
2

2
1m
2

2
2
1m

1 m 2
2

0
1
1
(1 m)

2
2 1 m

1m

(1 m)
2
3m

(1 m) m2 4m 5

= 0

1m
2
2

0
1 m 1 m
=

2
2
1m

1 m 2
4

0
1
0
= (1 m)

2
2 3 m
= (1 m) [(1 m) (3 m) 8]

= 0 m1 = 1 = m2 , m3 = 5

Los autovectores son:


m1 = m2 = 1, es un valor propio de multiplicidad 2. Los vectores

1 (1)
2
2
a

b
2
1 (1)
2
2
2
1 (1)
c
2a 2b + 2c
2a + 2b 2c
2a 2b + 2c

propios asociados:

0
= 0
0
= 0
= 0
= 0

se observa que las ecuaciones son equivalentes, luego el sistema no es tal, sino una ecuacin:
ab+c=0b=a+c
por tanto






a
a
a
0
1
0
V = b = a + c = a + c = a 1 + c 1
c
c
0
c
0
1

Luego, escogiendo a = 1, c = 0, se tiene

y escogiendo a = 0, c = 1, se tiene

1
V1 = 1
0

0
V2 = 1
1
158

que, claramente, son linealmente independientes.


Para m = 5, el vector propio es:

4 2 2
a
2 4 2 b
2 2 4
c
4a 2b + 2c
2a 4b 2c
2a 2b 4c

=
=
=
=

0
0
0
0
0
0

diviendo por 2 cada ecuacin del sistema, se tiene


2a b + c = 0 (1)
a 2b c = 0 (2)
a b 2c = 0 (3)
sumando (3) + (2) se tiene
2a b + c = 0 (1.1)
a 2b c = 0 (2.1)
3b 3c = 0 (3.1)
despejando c en (3.1)
c = b

reemplazando en (1.1) y (2.1) se tiene

2a 2b = 0
a b = 0
entonces
a = b

para b = 1 se tiene

Luego, la solucin general es:

a
b
1
V3 = b = b = b 1
c
b
1

1
V3 = 1
1

1
0
1
U (t) = C1 et 1 + C2 et 1 + C3 e5t 1 , C1 , C2 , C3 R.
0
1
1
x (t) = C1 et C3 e5t
y (t) = C1 et + C2 et + C3 e5t
z (t) = C2 et C3 e5t

159

Representaciones grficas de las soluciones son las siguientes: primero las soluciones por separado:

ahora las soluciones determinando una curva paramtrica:

Algunas curvas solucin para diferentes condiciones iniciales:

160

Ejemplo 12 Resolver
x0
y0
z0

= x+y
= y+z
= z

Escribiendo la ecuacin en forma matricial, se tiene


0

x
1 1 0
x
y0 = 0 1 1 y
0 0 1
z
z0

Tiene un nico valor propio:

1m
1
0
= (1 m)3 = 0 m1 = m2 = m3 = 1,
0
1m
1
det
0
0
1m

m = 1, tiene multiplicidad 3.
Determinamos, ahora, los vectores propios

0 1
0 0
0 0

asociados a m = 1:


0
a
0
1 b = 0
0
c
0
b = 0
c = 0
0 = 0

1
por lo tanto V1 = 0 , es el nico vector propio asociado a m = 1. Para determinar dos vectores propios
0
linealmente independientes, utilizamos la frmula de construccin dada anteriormente:
det(A I)W2
det (A I) W3
Para determinar W2 :

y el vector propio:

0 1 0
a
0 0 1 b
0 0 0
c
b
c
0

= V1
= W2

=
=
=
=

1
0
0
1
0
0


a
1
0
W2 = 1 = a 0 + 1
0
0
0
161

escogiendo a = 0, se tiene

0
W2 = 1
0

Para determinar W3 :

0 1 0
a
0 0 1 b =
0 0 0
c
b =
c =
0 =
y el vector propio:

escogiendo a = 0, se tiene

0
1
0
0
1
0


a
1
0
W3 = 0 = a 0 + 0
1
0
1

0
W3 = 0
1

Luego, la solucin general del sistema se determina como una combinacin lineal de los vectores:

t
1
e
U1 = 0 et = 0
0
0


t
0
1
te
U2 = 1 + t 0 et = et
0
0
0


t2

t
0
0
1
2
2e
t
t
t

0
1
0
+t
+
e =
U3 =
te
2
1
0
0
et

Por lo tanto

2
C1 + tC2 + t2 C3
et , C1 , C2 , C3 R.
U (t) =
C2 + tC3
C3

Representaciones grficas de las soluciones son las siguientes: primero las soluciones por separado:

162

ahora las soluciones determinando una curva paramtrica:

Algunas curvas solucin para diferentes condiciones iniciales:

163

Ejemplo 13 Resolver el sistema


y1 + y2 + y3
y3
y2

= y10
= y20
= y30

Escribiendolo en notacin matricial, tenemos;


0

y1
1 1 1
y1
0 0 1 y2 = y20
y3
y30
0 1 0

El polinomio asociado es:

1m
1
1

m 1

0
m 1
0 = det
= (1 m)
= (1 m) m2 + 1
1
m
0
1
m

luego:m1 = 1, m2 = m3 = i.
Los vectores propios asociados son:
para m = 1 :


0 1
1
a
0
0 1 1 b = 0 b + c = 0 b = c = 0.
bc=0
0 1 1
c
0



a
a
1
1
V1 = b = 0 = a 0 , para a = 1, V1 = 0
c
0
0
0
por lo tanto

para m = i :

1i 1
1
a
0
i 1 b
0
1 i
c
(1 i) a + (1 i) b

1
U1 = 0 et
0

0
(1 i) a + b + c = 0
(1 i) a + b + c = 0
ib c = 0
= 0

c = bi
0
b ic = 0
= 0a=b

b
b
0
1
0
V1 = b = b + 0 = b 1 + bi 0 ,
bi
0
bi
0
1

1
0
para b = 1, V1 = 1 + i 0
0
1
164

por lo tanto

y la solucin general es:

1
0
U2 (t) = 1 cos (t) 0
0
1

0
1
U3 (t) = 0 cos (t) + 1
1
0

cos (t)
sen (t) = cos (t)
sen (t)

sen (t)
sen (t) = sen (t)
cos (t)

1
cos (t)
sen (t)
U (t) = C1 0 et + C2 cos (t) + C3 sen (t) , C1 , C2 , C3 R.
0
sen (t)
cos (t)
y1 (t) = C1 et + C2 cos (t) + C3 sen (t)
y2 (t) = C2 cos (t) + C3 sen (t)
y3 (t) = sen (t) C2 C3 cos (t)
Representaciones grficas de las soluciones son las siguientes: primero las soluciones por separado:

ahora las soluciones determinando una curva paramtrica:

165

Algunas curvas solucin para diferentes condiciones iniciales:

Ejemplo 14 Resolver el sistema


y1 + y2 + y3
y2 y3
y2 + y3

= y10
= y20
= y30

Escribiendolo en notacin matricial, tenemos;

0
1 1 1
y1
y1
0 1 1 y2 = y20
0 1 1
y3
y30

El polinomio asociado es:

1m
1
1

1m
1

0
1m
1
0 = det
= (1 m)
= (1 m) m2 2m + 2
1
1m
0
1
1m
166

luego:m1 = 1, m2 = m3 = 1 + i.
Los vectores propios asociados son:
para m = 1 :


0 1 1
a
0
b+c=0
0 0 1 b = 0 c = 0 b = c = 0.
0 1 0
c
0
b=0



a
a
1
1
V1 = b = 0 = a 0 , para a = 1, V1 = 0
c
0
0
0
por lo tanto

para m = 1 + i :

i 1
1
a
0 i 1 b
0
1 i
c
ia + (1 i) b

por lo tanto

1
U1 = 0 et
0

0
ia + b + c = 0
ia + b + c = 0
ib c = 0
= 0

c = bi
0
b ic = 0
= 0 a = (1 + i) b

(1 + i) b
b
bi
1
1
= b + 0 = b 1 + bi 0 ,
b
V1 =
bi
0
bi
0
1

1
1
para b = 1, V1 = 1 + i 0
0
1

1
1
U2 (t) = 1 et cos (t) 0
0
1

1
1
U3 (t) = 0 et cos (t) + 1
1
0

et cos (t) et sen (t)


et sen (t) =

et cos (t)
et sen (t)

e cos (t) + et sen (t)


et sen (t) =

et sen (t)
et cos (t)

y la solucin general es:


1
cos (t) sen (t)
cos (t) + sen (t)
+ C3 et
, C1 , C2 , C3 R.
cos (t)
sen (t)
U (t) = C1 0 et + C2 et
0
sen (t)
cos (t)
o

y1 (t) = (C1 + (C2 + C3 ) cos (t) + (C3 C2 ) sen (t)) et


y2 (t) = (C2 cos (t) + C3 sen (t)) et
y3 (t) = (sen (t) C2 C3 cos (t)) et
167

Representaciones grficas de las soluciones son las siguientes: primero las soluciones por separado:

ahora las soluciones determinando una curva paramtrica:

Algunas curvas solucin para diferentes condiciones iniciales:

168

En general, si U1 , ..., Un son soluciones linealmente independientes de U 0 = AU , entonces la solucin general


del sistema puede expresarse como

C1

C1 U1 + + Cn Un = [U1 Un ] ... = U C
Cn

Note, tambin, que si C (t) =

C1 (t)

..
, entonces
.
Cn (t)

C1 (t)

..
U 0 C (t) = [U10 Un0 ]

.
Cn (t)
=
=
=
=

C1 (t) U10 + Cn (t) Un0


C1 (t) AU1 + Cn (t) AUn
A {C1 (t) U1 + Cn (t) Un }
AU C (t)

Definicin 4 Si U1 (t) , ..., Un (t) son n soluciones linealmente independientes del sistema de n ecuaciones con
n incgnitas U 0 = AU la matriz U (t) = [U1 (t) Un (t)] se denomina matriz fundamental del sistema U 0 = AU,
la denotaremos (t).

2.3

Ejercicios

1. Empleando el mtodo de los valores prpios, determine las soluciones de los siguientes sistemasde ecuacionesa diferenciales.

dx/dt = 2y(t)
(a)
dy/d(t) = 8x(t)

dx/dt = 12 x + 9y
(b)
dy/dt = 12 x + 2y
0
x = 2x 7y
y 0 = 5x + 10y + 4z ,
(c)
0
z = 5y + 2z

6 2
X
(d) X 0 =
3 1

1 0 1
(e) X 0 = 0 1 0 X
1 0 1

1 4 1
(f) X 0 = 4 1 2 X
0
0 6
169

2. Resuelva cada sistema sujeto a la condicin inicial dada.


1/2
0
3
0
(a) X =
X , X(0) =
1 1/2
5


1 1 4
1
(b) X 0 = 0 2 0 X , X(0) = 3
1 1 1
0

3. Determine la solucin general de cada uno de los siguientes sistemas de ecuaciones diferenciales.

dx/dt
= 3x y
(a)
dy/d(t) = 9x 3y

dx/dt = 6x + 5y
(b)
dy/d(t) = 5x + 4y

dx/dt = 12x 9y
(c)
dy/d(t) =
4x
0
x = 3x + 2y + 4z
y 0 = 2x + 2z
(d)
0
z = 4x + 2y + 3z

1 0 0
(e) X 0 = 0 3 1 X
0 1 1

4 1 0
(f) X 0 = 0 4 1 X
0 0 4

dx/dt =
x+y
(g)
dy/d(t) = 2x y

dx/dt = 4x + 5y
(h)
dy/d(t) = 2x + 6y


1 8
3
0
(i) X =
X , X(0) =
1 3
5
0
x = 2x + y + 2z
y 0 = 3x + 6z
(j)
0
z = 4x 3z

4 0 1
(k) X 0 = 0 6 0 X
4 0 4

2
4
4
(l) X 0 = 1 2 0 X
1 0 2

170

1 12 14
4
2
3 X , X(0) = 6
(m) X 0 = 1
1
1
2
7

6 1
2
(n) X 0 =
X , X(0) =
5 4
8

Sistemas de lineales no homogneos.

Son sistemas de la forma


U 0 (t) = AU (t) + F (t)

f1 (t)
x1 (t)

..
..
donde A = (aij )nn , U =
, F (t) =
.
.
.
xn (t)
fn (t)
Se verifica fcilmente que si (t) es una matriz fundamental para U 0 (t) = AU (t), y Up (t) es alguna solucin
(particular) de U 0 (t) = AU (t) + F (t) entonces, toda solucin del sistema no homogneo se expresa como

U (t) = (t) C + Up (t)

C1

donde C = ... es un vector constante.


Cn

3.1

Mtodo de variacin de parmetros.

Problema: Determinar una solucin particular Up (t) del sistema U 0 (t) =AU (t) +F (t) .
v1 (t)

..
0
Sea (t) una matriz fundamental de U (t) = AU (t) y sea V (t) =
. Intentaremos determinar
.

vn (t)
una solucin particular de la forma Up (t) = (t) V (t) reemplazando en la ecuacin:

0 (t) V (t) + (t) V 0 (t) = A (t) V (t) + F (t) (t) V (t) = F (t)
{z
}
| {z }
|
&

(t) V (t) = A (t) V (t)

y como (t) es una matriz fundamental, ella es invertible:


V 0 (t) = 1 (t) F (t)
Z
V (t) =
1 (t) F (t) dt
Z
Up (t) = (t) 1 (t) F (t) dt
Conclusin:

171

La solucin general del sistema no homogneo


U 0 (t) = AU (t) + F (t)
est dada por
U (t) = (t) C + (t)

1 (t) F (t) dt

C1

donde (t) es la matriz fundamental de U 0 (t) = AU (t) y C = ... es un vector constante arbitrario.
Cn

Observacin 8 Recuerde que si (t) = ij (t) es tal que det ( (t)) 6= 0, entonces
1 (t) =

donde

ij

1
i+j
(1)
det ( (t))

ij

es el determinante de la matriz obtenida de eliminando la fila i y la columna j.

Ejemplo 15 Resolver el problema con condiciones iniciales


4 2
15
7
0
2t
U =
U
te , U (0) =
3 1
4
3
La solucin del sistema homogneo asociado es:

4m
2
det
3
1 m
(4 m) (1 m) 6
m2 3m 10
(m + 2) (m 5)

= 0
= 0
= 0
= 0

luego, los valores propios son: m1 = 2 y m2 = 5.


Los vectores propios asociados son:
Para m1 = 2 :

4 (2)
2
a
3
1 (2)
b

6 2
a
3 1
b
6a + 2b
3a + b
luego, b = 3a y el vector propio es:
V1 =

a
b

a
3a
172

=a

=
=

= 0
= 0

1
3

0
0
0
0

considerando el valor a = 1, se tiene:


V1 =
Para m1 = 5 :

1
3

45
2
a
3
1 5
b

1 2
a
3 6
b
a + 2b
3a 6b

luego, a = 2b y el vector propio es:


V2 =
considerando el valor b = 1, se tiene:

a
b

V2 =
y las soluciones son
U1
U2
por lo tanto, la matriz fundamental es

2b
b

2
1

0
0
0
0

= 0
= 0

=b

2
1

1
e2t
e2t =
3
3e2t

5t
2
2e
=
e5t =
1
e5t
=

(t) =

e2t
3e2t

2e5t
e5t

R
La solucin Up (t) est dada por:Up (t) = (t) 1 (t) F (t) dt, donde

2e5t
e2t
(t) =
3e2t e5t

15te2t
F (t) =
4te2t

1 2t

1
e5t
2e5t
27 e2t
7e
1 (t) =
=
3 5t
1 5t
3e2t e2t
7e3t
7e
7e

Z
Z 1 2t
27 e2t
15te2t
7e
1 (t) F (t) dt =
dt
3 5t
1 5t
4te2t
7e
7e

Z
t
12 t2
=
dt
=
7te7t
te7t + 17 e7t

Z
12 t2
e2t
2e5t
(t) 1 (t) F (t) dt =
3e2t e5t
te7t + 17 e7t

1 2 2t
+ 2te2t + 27 e2t
2t e
Up (t) =
3 2 2t
+ te2t + 17 e2t
2t e
173

y la solucin general:
Z

U (t) = (t) C + (t) 1 (t) F (t) dt

1 2 2t

e2t
C1
2t e
2e5t
+ 2te2t + 27 e2t
U (t) =
+
3 2 2t
3e2t e5t
C2
+ te2t + 17 e2t
2t e
Aplicando la condicin inicial:

C1 + 2C2

3C1 + C2

7
3

47
7
20
7

1 2
3 1

C1
C2

2
7
1
7

las constantes son:


47
7
40
=
7
1
=
7
23
=
7

C1 + 2C2

6C1 2C2
C1
C2
Luego la solucin es
U (t) =
=

e2t
3e2t

2e5t
e5t

1
7
23
7
2t

46 5t
1 2 2t
+ 2te
7 e 2t e
3 2 2t
23 5t
+ te2t
7 e + 2t e

12 t2 e2t + 2te2t + 27 e2t


3 2 2t
+ te2t + 17 e2t
2t e

2t

+ 37 e
27 e2t

Representaciones grficas de las soluciones son las siguientes: primero las soluciones por separado:

174

ahora las soluciones determinando una curva paramtrica:

Algunas curvas solucin para diferentes condiciones iniciales:

Una aplicacin de este tipo de ecuacin es la siguiente:


Ejemplo 16 Considere el sistema de estanques A, B y C interconectados como en la figura. Un mecanismo
de bombeo permite transferir 3 galones por minuto desde el estanque A al B y 2 galones por minuto desde el
estanque B al C, y vlvulas de desage en los estanques B y C permiten drenar del sistema 1 galn por minuto
desde el estanque B y 3 galones por minuto desde el estanque C. A partir de cierto instante se hace ingresar a los
1
estanques A y C una solucin salina con concentracin variable dada por e 20 t libras de sal por galn a razn
de 3 galones por minuto en el estanque A y a razn de 1 galn por minuto en el estanque C. Simultneamente se
activa el mecanismo de bombeo en el sistema de estanques y se abren las vlvulas de desage en los estanques B
y C. Si inicialmente el estanque A contena 60 galones de salmuera con 20 libras de sal y el estanque B contena
60 galones de agua pura y el estanque C contena 60 galones de salmuera con 30 libras de sal, determine las
cantidades de sal en los estanques A, B y C en un instante cualquiera. Suponga que en cada instante la sal est
uniformemente distribuida en cada estanque.

175

El sistema de ecuaciones que modela el problema es


1

x0

= 3e 20 t 3

y0

= 3

z0

x
60

x
y
3
60
60
1
y
z
= e 20 t + 2 3
60
60

o,
x0
y0
z0

1
1
x + 0y + 0z + 3e 20 t
20
1
1
=
x y + 0z
20
20
1
1
1
= 0x + y z + e 20 t
30
20

El sistema de forma matricial es

Luego la matriz:

Vectores propios:

0
0

1
20

Existen valores propios

1
20

0
1
20
1
30

1t
0
3e 20

0 X +
0
1
1
20
e 20 t

0
1
0 tiene un nico valor propio m = 20
, con multiplicidad 3.
1
1

30

20
0 0
0
a
0
1
1
0 0 b = 0 20
a = 0, 30
b = 0 0 = V1 .
1
1
c
0
0
30


0
0 0
a
0
1
1
1
0 0 b = 0 20
generalizados: 20
a = 0, 30
b = 1 b = 30, a =
1
c
1
0 30 0

1
20
1
20

X0 =

1
20

0
1
20

0
0 V2 = 30
0

0
0 0
a
0
1
0 0 b = 30
20
1
c
0
0 30
0

600
1
1
0
20 a = 30, 30 b = 0 a = 600, b = 0 V3 =
0
176

Solucin homognea:

0
0
0
0
600
0

1
1
1
1
X1 = 0 e 20 t , X2 = 30 + t 0 e 20 t , X3 = 0 + t 30 + t2 0 e 20 t

2
1
1
0
1
0
0

1 2

1
0 0 600
t 30
t 1
1200
1
1
1
1
t
0
La matriz fundamental es: (t) = e 20 t 0 30 30t 1 (t) = e 20 t 600
30
1 2
1
1 t 2t
0
0
600
Para calcular la solucin no homognea:
1 2

1
t 30
t 1
1200
1
1
1
t
0
1 (t) = e 20 t 600
30
1
0
0
600
1t 1 2

1 2 1t
1
1
1
20
30
te 20 t e 20 t
3e 20
1200 t e 1
400 t + 1
1
1
1 20 t
= 1 t
0
1 (t) F (t) = 600
te 20 t
0
200
30 e
1
1
1

t
1
t
20
e 20
0
0
200
600 e
1 3

Z
1200 t + t
1 2

t
1 (t) F (t) dt = 400
1
t
200

1 3


0 0 600 C1

1200 t + t
1
1 2

t
Solucin general: X (t) = e 20 t 0 30 30t C2 + 400

1
1
1 t
C
2t

3
200 t
20
0 0 600
C1
20
1
Como X (0) = 0 , entonces 0 30 0 C2 = 0 C1 = 30, C2 = 0, C3 = 30
30
1 0
0
30
C3
Por lo tanto, la
solucin
es:

1 3

0 0 600 1200
3t + 20
t + t + 30
1
1
3 2
1 2
= e 20 t

400
t
X (t) = e 20 t 0 30 30t
40 t + t

1 2
1
1
1 2
1
3
1 t 2t
1200 t + 60 t + t + 30
200 t + 30
t

x (t) = e 20 (3t + 20)


2

t
3t
y (t) = e 20
+t
40
3

t
t
t2
z (t) = e 20
+
+ t + 30
1200 60

Representaciones grficas de las soluciones son las siguientes: primero las soluciones por separado:

177

ahora las soluciones determinando una curva paramtrica:

Algunas curvas solucin para diferentes condiciones iniciales:

178

3.2

La matriz exponencial.

Si A es una matriz cuadrada de orden n n, se define

t
t3
t2
A + A2 + A3 +
1!
2!
3!

X
1
k
(At)
=
k!

eAt

= I+

k=0

Proposicin 3 eAt es realmente una matriz, cualquiera que sea la matriz A.

A =
A3

eAt

(2)
0

"

(2)
0

e2t
0


2 0
2 0
(2)2
, A =
=
0 3
0 3
0

3
2 0
(2)
0
0
=
,
0 3
(3)2
0
(3)3

2 0
0 3

0
k
(3)
0
e3t

en general, si

At

1 0
0 1

t
+
1!

2 0
0 3

0
(3)2

t2
+
2!

(2)
0

0
2
(3)

A=

m1
0
..
.

0
m2
..
.

..
.

0
0
..
.

mn

eAt =

em1 t
0
..
.

em2 t
..
.

..
.

0
0
..
.

emn t

Propiedades: Sean A y B matrices de orden n n y r, s, t R ( C) Entonces:

+ =

1.- eA0 = e0 = I

2.- eA(t+s) = eAt eAs


1
3.- eAt
= eAt , (eAt es invertible, cualquiera que sea A).
4.- Si AB = BA e(A+B)t = eAt eBt
5.- erIt = ert I
6.- Como eAt = I +

t
1! A

t2 2
2! A

t3 3
3! A

d At
e
dt

+ , entonces, se puede definir la derivada como:


t
t3
t2
= A + 2 A + 3 A2 + 4 A3 +
3!
4!

2!
t
t2 2 t3 3
= A I + A + A + A +
1!
2!
3!

= AeAt

179

X
(2t)k

k!

k=0

X
(3t)k
k!

k=0

y entonces, dado

v1
v2
..
.

vn

d At
d
d

e V = eAt V + eAt V

dt
dt
dt

= AeAt V + eAt 0
= AeAt V

Note que eA0 V = IV = V . Por lo tanto eAt V es la solucin del P.V.I.


U 0 = AU, U (0) = V
Esto demuestra la siguiente proposicin

v1
v2
..
.

Proposicin 4 Si A es una matriz de orden n n y V =

vn

entonces

U (t) = eAt V, es solucin de U 0 = AU

y adems, U (0) = eA0 V = IV = V.


a.- eAt es una matriz fundamental para U 0 = AU
1

b.- Si U (t) es cualquier matriz fundamental para U 0 = AU entonces eAt = U (t) (U (0))
c.- La solucin general de U 0 = AU + F (t) est dada por
U (t) = eAt C + eAt

eAt F (t) dt

C1

donde C = ... es un vector columna constante arbitrario.


Cn

En general el clculo de eAt no es simple. Sin embargo el clculo se facilita si A es una matriz nilpotente,
esto es, existe k N tal que Ak+1 = 0. Si este es el caso, entonces
eAt = I +

t
tk
t2
A + A2 + + Ak + 0 + 0 +
1!
2!
k!

Ejemplo 17 Resolver
0

U =

2 1
1 2

U+

et
et

Para resolver la ecuacin homognea podemos utilizar dos formas:


180

i. Valores Propios:
det

2m
1
1
2m

= 0 (2 m) 1 = 0 2 m = 1 m = 2 1 =

a
b

y los vectores propios son:


m=1:

m=2:

1 1
1 1

1 1
1 1

y la solucin homognea es

Uh (t) = C1

a
b

1
1

0
0

0
0

a + b = 0 a = b v1 =

e + C2

1
1

C1
C2

a + b = 0 a = b v2 =

1
1

3t

et
et

e3t
e3t

1
1

ii. Determinando la matriz eAt : Observe que

2 1
1 0
0 1
A=
=2
+
= 2I + B
1 2
0 1
1 0
y como (2I) B = B (2I), entonces eAt = e2tI+tB = e2t etB , adems

0 1
0 1
1 0
=
= I B 3 = B, B 4 = I, B 5 = B, ...
B2 =
1 0
1 0
0 1
entonces B 2n = I, B 2n+1 = B, y

t2
t3
1 0
0 1
1 0
0 1
tB
=
+t
+
e
+
+
0 1
1 0
0 1
1 0
2!
3!
t3
t4
t5
t2
= I + tB + I + B + I + B +
2!
3!
4!
5!

t4 t6
t3
t5
t2
=
1 + + + + I + t + + + B
2! 4! 6!
3! 5!
sean
t2 t4 t6
+ + +
2! 4! 6!
t5
t3
b (t) = t + + +
3! 5!

a (t) = 1 +

observe que
et
et

t2 t3 t4
t5
+ + + +
2! 3! 4! 5!
t2 t3 t4
t5
= 1 t + + +
2! 3! 4! 5!
= 1+t+

181

1
3

luego
et + et
et et
entonces
eBt

=
=

y
At

e2t
0

y la solucin homognea es

0
e2t

1 t
e + et
2

1 t
= 2b (t) b (t) =
e et
2
= 2a (t) a (t) =

1 t
e + et
2
t
1
e + et
et et
2

1
2

et + et
et et

Uh (t) = C1

1 t
+
e et
2

t
t
e e
et + et
1 0
0 1

et et
et + et

e3t + et
e3t et

+ C2

1
=
2

0 1
1 0

e3t + et
e3t et

e3t et
e3t + et

e3t et
e3t + et

La solucin de la ecuacin no homognea la realizaremos utilizando la matriz eAt , cuya inversa es simple de
calcular:

3t
1
+ et e3t et
e
At
e
=
e3t et e3t + et
2
t

3t
2t
1
1
e
+ et e3t et
+ 1 + e4t e2t
e
e
At
e
F (t) =
=
e3t et e3t + et
et
e2t 1 + e4t + e2t
2
2

1
e4t + 1
=
4t
e
+ 2e2t 1
2

Z
1
14 e4t + t
At
F (t) dt =
e
14 e4t e2t t
2

3t
Z
1
1
e + et e3t et
14 e4t + t
At
At
Up = e
F (t) dt =

e
e3t et e3t + et
14 e4t e2t t
2
2

1 t
1
4 e + te3t 14 e3t + tet 14 et et te3t + 14 e3t + et + tet
=
1 et + te3t + 14 e3t tet 14 et et te3t 14 e3t et tet
4

14 t
1
e + 2tet et
2
Up =
32 et 2tet et
4
y la solucin general es

1 t
1
C1
e3t et
e + 2tet et
2
U (t) =
+
e3t + et
C2
32 et 2tet et
4

C1 + C2
C1 C2 + t 1
1
x (t) = e3t
+ et

+ et
2
2
4
8

C2 C1 t 1
3
3t C1 + C2
t
y (t) = e
+e

et
2
2
4
8
1
2

e3t + et
e3t et

182

Representaciones grficas de las soluciones son las siguientes: primero las soluciones por separado:

ahora las soluciones determinando una curva paramtrica:

Algunas curvas solucin para diferentes condiciones iniciales:

183

Ejemplo 18 Calcular eBt , para

1
1
1
1
1
B= 1
2 2 2

Calculando la potencias de B se tiene:

1
1
1
1
1
1
1
1
1
0 0 0
1
1 , B2 = 1
1
1 1
1
1 = 0 0 0
B= 1
2 2 2
2 2 2
2 2 2
0 0 0

y entonces

eBt
Note que

1 0 0
1
1
1
1+t
t
t

1
1 = t
1+t
t
= 0 1 0 +t 1
0 0 1
2 2 2
2t 2t 1 2t
eBt

Ejemplo 19 Determine eAt de la matriz

1 t t
t
t
= t 1 t
2t
2t
1 + 2t

1
20

A=

0
1
20

1
20

1
30

0
0
1
20

Note que la matriz A puede descomponerse en la siguiente suma de matrices:


1
0
0
20
0
0
1 0 0
1
1
1
1
0
20
0 = 0 1 0 + 20
A = 20
20
1
1
1
0
0
1
0

0
30
20
30

y la matriz bajo-triangular es nilpotente:

1
20

1
20

entonces:
eAt

eAt

0
0

1
30

0
0

1
30

0
0
0

0
0
0

1 0 0
1
= e 20 t 0 1 0

0 0 1

1
0
1
t
1
= e 20 t 20
t2
1200

t
30

0
0 =
1
0
30

0 0
0 0 =
0 0
0
0

1
20

0
0
1
600

+t

0
1
20

0
0
1

184

0
0
1
30

0
0
0

0 0
0 0
1
0 0
600

0 0 0
0 0 0
0 0 0
0
0

0
0
2
t
0 + 0
2
1
0
600

0 0
0 0

0 0

En general no podemos esperar que A sea nilpotente, pero note que si m es un valor propio de A entonces
eAt = eAtmIt+mIt = emt e(AmI)t
y si v es un vector columna de orden n 1, entonces
U (t) = eAt v = emt e(AmI)t v
y si escogemos v como un vector propio generalizado, asociado al autovalor m, tal que (A mI)k+1 v = 0,
entonces

t
tk
t2
U (t) = emt Inn v + (A mI) v + (A mI)2 v + + (A mI)k v
1!
2!
k!
es una solucin de U 0 = AU.
Ejemplo 20 Considere

1 1 0
x
U0 = 0 1 1 y
0 0 1
z

Segn se ha visto anteriormente, m = 1 es el nico autovalor propio



1
0
v1 = 0 , v2 = 1 , v3 =
0
0

0
0
1

son vectores propios generalizados (v1 es vector propio) asociados a m = 1. Ellos satisfacen
i. (A 1I) v1 = 0 (A 1I)2 v1 = 0 = (A 1I)3 v1 =

t
t
2
= et v1 = et 0
U1 (t) = et e(AI)t v1 = et
v
+
+
v
+

(A

I)
v
(A

I)
1
1
1

|1! {z
} |2!
{z
}
0
2

= 0

= 0


0 1 0
0 1 0
0
1
ii. (A 1I) v2 = 0 0 1 v2 = 0 0 1 1 = 0 = v1 (A I)2 v2 = (A I) v1 = 0 =
0 0 0
0 0 0
0
0
3
(A I) v2 =


1
0

U2 (t) = et e(AI)t v2 = et v2 + 1!t (A I) v2 = et 1 + t 0

0
0

iii. (A 1I) v3 = v2 (A I)2 v3 = (A I) v2 = v1 , (A I)3 v2 = 0 =




0
0

2
2
U3 (t) = et e(AI)t v3 = et v3 + 1!t (A I) v3 + t2! (A I) v3 = et 0 + t 1 +

1
0
185

t
2!

1
0

Conforme a la solucin obtenida en el ejemplo anterior.


Ejemplo 21 Resuelva el siguiente sistema de ecuaciones con condicin inicial:
X 0 (t) = AX (t) + F (t)
donde

2

0 0 0
t
1
A = 1 0 0 , F (t) = t1 , X (1) = 2
2 3 0
3
3

0 0 0
Sea A = 1 0 0 , entonces las potencias
2 3 0

0 0
A2 = 0 0
3 0

o sea, A es nilpotente de orden 3. Entonces

de A, son

0
0 0 0
0 , A3 = 0 0 0
0
0 0 0

1 0
t
eAt = I + tA + A2 = 0 1
2
0 0

1
0 0
t
1 0
eAt =
2t + 32 t2 3t 1

1
0
t
1
Por lo tanto la inversa es: eAt =
3 2
t

2t
3t
2
Como la solucin de la ecuacin es
2

At

0
0 0 0
0 0 0
2
t
0 + t 1 0 0 + 0 0 0
2
1
2 3 0
3 0 0

0
0 .
1
At

X (t) = e C + e

eAu F (u) du

Se tiene que
eAu F (u) =
Z
eAt

eAu F (u) du =
1
t

eAu F (u) du =
1

Xp

1
0
0
u2
u2

u
1
0 u1 = 2u1
3 2
15
1
3
2 u 2u 3u 1
2 2u

Z t
u2
1 t1
2u1 du =

2 ln (t)
15
15
15
1
1

2u
t

2
ln
(t)

2
2

1
0 0
1 t1

=
t
1 0
2 ln (t)
3 2
15
15
3 2
2t + 2 t 3t 1
2 t 2 ln (t) 2
2t +

1
1t

t 1 2 ln (t)
3 2
19
15
2 t + 2 t 6t ln (t) 2 ln (t) 2
186

1 t1
t 1 2 ln (t)
19
t

6t ln (t) 2 ln (t)
2

15
2

Por lo tanto:

1
t
X (t) =
2t + 32 t2

determinando las constantes

0 0
C1
1 0 C2 +
3t 1
C3

1
X (1) = 1

C1
C1 + C2
7
2 C1 + 3C2 + C3 +

7
2

7
2

1 t1
t 1 2 ln (t)
3 2
19
t
+
t

6t ln (t) 2 ln (t)
2
2

15
2

0 0
0
1
C1
1 0 C2 + 0 = 2
7
3
C3
3 1
2

1
= 2
3

C1 = 1, C2 = 1, C3 = 7 y la solucin es:


1
0 0
1
t
1 0 1 +
X (t) =
3 2
2t + 2 t 3t 1
7

1 t1
t 1 2 ln (t)
3 2
19
t
+
t

6t ln (t) 2 ln (t)
2
2

15
2

x (t) = 2 t1
y (t) = 2t 2 ln (t)
29
29
z (t) = 3t2 + t 6t ln (t) 2 ln (t)
2
2
Representaciones grficas de las soluciones son las siguientes: primero las soluciones por separado:

ahora las soluciones determinando una curva paramtrica:

187

Ejemplo 22 Considere el sistema de estanques interconectados como lo muestra la figura.Suponga que inicialmente cada estanque contiene 100 galones de agua pura y que la solucin entra y sale de cada estanque a razn
de 5 galones por minuto. Si la solucin que entra en el estanque A tiene una concentracin de 2 kilogramos de
sal por galn, determine la concentracin de sal en cada estanque despus de 10 minutos. Es cierto que si el
hsistema
i continua funcionando indefinidamente las concentraciones en cada estanque sern aproximadamente 2
kgs
gal . Explique su respuesta.

188

Sean x, y, z las cantidades de sal en A, B y C respectivamente. x (0) = 0 = y (0) = z (0).

0
5
1
x0 = 10 100
20
x
0
0
x
10
x
5
5
1
1
y 0 = 100
x 100
y y 0 = 20
20
0 y + 0
5
5
1
1
0
0
z
0
z
z = 100 y 100 z
0
20
20
{z
}
|
A

note que

A=

1
I +
20

0
1
20

0
0
1
20

0
0
0

0
1
1
0 es nilpotente de orden 2, luego como eAt = e 20 tI+Bt
donde B = 20
1
0 20
0
t
e 20 eB , y B es nilpotente de orden 2, se tiene que:

0
0 0
0 0
1 0 0
1
1
0 0 t + 0 0
eAt = e 20 t 0 1 0 + 20

1
1
0 0 1
0 20
0
0
400

1
0 0
1
t
1 0
eAt = e 20 t 20
t2
t
1
800
20
0

0
0


0
t2
0
2
0

1
0
0
10
10
10
1
1
t
1
0 0 = e 20 t 2t ,
Entonces eAt 0 = e 20 t 20
2
t2
t
t
0
0
20 1
800
80

1
200e 20 t
10
200
R At
1
1
1
= e 20 t

200 10t
luego e 0 dt =
10te 20 t + 200e 20 t
1
1
1
1
2
1
2 20 t
0
20 t + 200e 20 t
4 t 10t + 200
4 t e 10te

1
0 0
200
10
R
1
1
t
=
1 0 e 20 t
200 10t
Xp = eAt eAt 0 dt = e 20 t 20
2
1 2
t
t
0
1
4 t 10t + 200
800
20

1
0 0
200
t
=
1 0
200 10t
Xp = 20
1 2
t
t2
t

10t
+
200
4
800 20 1

200
200
= 200
10t 10t + 200
Xp =
1 2
1 2
1 2
200
4 t 2 t + 10t + 4 t 10t + 200
La solucin general es:

1
0 0
C1
200
1
t
1 0 C2 + 200
X (t) = e 20 t 20
t2
t
200
C3
1
800
20

189

= e 20 tI eBt = e 20 IeBt =

0
Como X (0) = 0
0

200
0
1 0 0
C1
0 = 0 1 0 C2 + 200 C1 = C2 = C3 = 200
200
C3
0
0 0 1

Por lo tanto

x (t) = 200 200e 20 t

1
t
y (t) = 200 200e 20 t 1 +
20

1
t
t2
20
t
z (t) = 200 200e
1+
+
20 800

y la concentracin en 10 minutos es:


x (t)
100
y (t)
100
z (t)
100

= 2 2e 2
1

= 2 3e 2
= 2

13 1
e 2
4

Como e 20 t 0 si t entonces la cantidad de sal en cada estanque (se aproxima) a 200 [kg] y la
kg
kg
concentracin tiende a 200
100 gal = 2 gal .
Representaciones grficas de las soluciones son las siguientes: primero las soluciones por separado:

ahora las soluciones determinando una curva paramtrica:

190

3.3

Relacin entre las ecauciones de orden superior y sistemas de ecuaciones de


primer orden

Consideremos la ecuacin D2 + 1 y = 0, o equivalentemente


y 00 + y = 0

(1)

o tambin
y 00 = y

Si se anota x1 (t) = y1 (t) ; x2 (t) = y10 (t), entonces se tiene


x01
x02

= x2
= x001 = y 00 = y = x1

Conclusin: La ecuacin y 00 + y = 0 determina el sistema de ecuaciones


x01
x02

= x2
= x1

y a su vez este sistema determina la ecuacin x002 = x01 = x2 , esto es x002 = x2 o bien x002 + x2 = 0, que no es
otra que la ecuacin (1).
Pero entonces la ecuacin y 00 + y = 0 puede reinterpretarse como

0 1
x1
0
U , donde U =
.
U =
1 0
x2
Las soluciones del sistema son:
Por el mtodo de valores propios:

m
1
det
= 0 m2 + 1 = 0 m = i
1 m
191

Los vectores propios asociados a m = i

i 1
a
0
ai + b = 0
=

b = ai
1 i
b
0
a bi = 0


1
1
0
Si a = 1, entonces V =
=
+i
y las soluciones son
i
0
1

x1 (t)
1
0
U1 =
cos (t)
sen (t)
= e0t
0
1
x2 (t)

x1 (t)
0
1
U2 =
cos (t) +
sen (t)
= e0t
1
0
x2 (t)

cos (t)
sen (t)
cos (t)
sen (t)
U1 =
+ C2
, U2 (t) =
; U = C1
sen (t)
cos (t)
sen (t)
cos (t)
Se pueden graficar estas soluciones, como curvas parametrizadas por t, como en los ejemplos anteriores.

Note que el vector velocidad de la curva t U (t) es U 0 (t) = AU (t) . De esta manera podemos asociar a la
ecuacin y 00 + y = 0 el campo de vectores A. Resolver la ecuacin y 00 + y = 0 es equivalente a resolver el sistema
U 0 = AU . Esto es, a su vez, equivalente a determinar curvas en el plano tales que sus vectores velocidad U 0
coinciden con AU .
En general, la ecuacin y 00 = f (y, y 0 ) es equivalente al sistema
x01
x02

= x2
= f (x1 , x2 )

donde x1 = y, x01 = x2 = y 0 , x02 = x001 = y 00 , y ms generalmente, la ecuacin y (n) = f y, y (1) , y (2) , ..., y (n1) es

192

equivalente al sistema
x01
x02
x03
x0n1
x0n

3.4

= x2
= x3
= x4
..
.
= xn
= f (x1 , x2 , ..., xn1 )

Ejercicios.

1. Aplique el mtodo de variacin de parmetros para determinar una solucin particular de cada sistema
dado a continuacin.

dx/dt
= 2x y
(a)
dy/d(t) = 3x 2y + 4t

2 1
sen(2t)
(b) X 0 =
X+
e2t
4 2
2 cos(2t)

0 2
2
(c) X 0 =
X+
1 3
e3t

t
1 8
e
X+
(d) X 0 =
1 1
tet

3
2
1
(e) X 0 =
X+
2 1
1

1 1
3
0
(f) X =
X+
et
1 1
3

2 2
1
e2t
0
(g) X =
X+
t
8 6
3

0 1
1
0
(h) X =
X+
1 0
cot(t)

1 2
tan(t)
0
(i) X =
X+
1 1
1

3 1 1
0
(j) X 0 = 1 1 1 X + t
1 1 1
2et
2. Determine las matrices exponenciales etA y etA para las siguientes matrices.

1 0
(a) A =
0 2

0 1
(b) A =
1 0
193

3. Determine las matrices exponenciales etA para las siguientes matrices.

1
1
1
1
1
(a) A = 1
2 2 2

0 0 0
(b) A = 3 0 0
5 1 0

4. Determine la solucin de cada sistema empleando la matriz exponencial.

0 1
0
(a) X =
X
1 0

0 0 0
(b) X 0 = 3 0 0 X
5 1 0

1 0
t
(c) X 0 =
X +
0 2
e4t

0 1
cosh(t)
(d) X 0 =
X +
1 0
senh(t)

5. Considere dos tanques con salmuera hconectados


entre s de tal forma que la salmuera se bombea desde el
i
gal
tanque 1 al tanque 2 a razn de 30 min por una caera y por una segunda caera se bombea desde
h
i
gal
el tanque 2 al tanque 1 a razn de 10 min
. El tanque 1 contiene x (t) libras de sal en 100 galones de
salmuera y el tanque 2 contiene y (t) libras de sal en 200 galones de salmuera. La salmuera en cada tanque
se mantiene uniforme por medio de agitacin. Determine la
de sal que hay en el tiempo t en
h cantidad
i
gal
cada tanque, si al tanque 1 entra agua pura a razn de 20 min , y la salmuera del tanque 2 sale a una
h
i
gal
velocidad de 20 min
.
6. Considere una masa m1 de 2[Kg] unida a un resorte de constante k1 = 4, ubicada sobre un plano horizontal.
A la masa m1 se adosa un resorte de constante k2 = 2. Si a este segundo resorte se adosa, en su extremo
libre una segunda masa m2 de 1[Kg], y sobre esta ltima se aplica una fuerza externa f (t) = 40sen (3t).
(a) Determine el sistema de ecuaciones que modela el comportamiento de este sistema masa-resorte.
(b) Determine las posiciones de las masas m1 y m2 en cualquier instante t. Si cada una de las masas se
encuentran en reposo en su posicin de equilibrio.
7. Dos tanques A y B, cada uno de los cuales contienes 50 litros de lquido, estn conectados entre s mediante
tubos. El lquido pasa del tanque A al tanque B a razn de 4 litros por minuto, y del tanque B a tanque A
a 1 litro por minuto. Elhlquido
i dentro de cada tanque se mantiene bien revuelto. Una solucin salina con
Kg
una concentracin de 3 lt de sal entra al tanque A a razn de 4 litros por minuto. Una solucin salina
h i
con una concetracin de 1 Kg
de sal entra al tanque B a razn de 1 litro por minuto. Las soluciones
lt
194

salen del sistema por ambos tanques, del tanque A a 1 litro por minuto y del tanque B a 4 litros por
minuto. Si en un principio el tanque A contiene agua pura y el tanque B contiene 5 [Kg] de sal, determine
la masa de sal en cada tanque en el instante t 0. Despus de transcurrir varios minutos, cul tanque
tiene la mayor concentracin de sal? Cul es la concentracin lmite?
8. Considere
h
iun sistema de tres tanques de salmuera, de tal forma que del tanque
h Ai sale salmuera a razn
gal
gal
de 10 min hacia el tanque B; del tanque B sale la solucin a razn de 10 min
hacia el tanque C, y
h
i
gal
desde este ltimo sale salmuera hacia el tanque A a la misma razn, 10 min . Si los volmenes iniciales
son 50 galones, 25 galones y 50 galones respectivamente, determine la cantidad de sal en cada estanque
en cualquier instante t, si el estanque A contena 50 libras de sal diluidas y los estanques B y C slo agua
pura. R.
9. Dos tanques, cada uno con 50 litros de lquido, estn conectados entre s mediante tubos, de modo que
el lquido pasa del tanque A al tanque B a razn de 4 litros por minuto, y del tanque B a tanque A a 1
litro por minuto. El lquido dentro de cada tanque se mantiene bien revuelto. Por otro lado, entra agua
pura al tanque A a razn de 3 litros por minuto, y la solucin sale del tanque B a 3 litros por minuto. Si
en un principio el tanque A contiene 25 [Kg] de sal y el tanque B no contiene sal (slo agua), determine
la masa de sal en cada tanque en el instante t 0.
10. Considere un sistema de tres tanques, con 1000 litros de salmuera cada uno, conectados entre s de modo
que del tanque A pasa al tanque B, por medio de una caera, la solucin bien mezclada a razn de 100
litros por minuto, y desde el tanque B al tanque C, la solucin bien mezclada, pasa por una caera a
razn de 100 litros por minuto, y desde este ltimo tanque, la solucin sale fuera del sistema a razn de
100 litros por minuto. Si al tanque A ingresa 1 kilo de sal por litro, a razn de 100 litros por minuto,
determine las concentraciones de sal en cada tanque, si en un principio cada tanque contena 10, 20 y 30
kilos de sal respectivamente.

R
R
at
(Sugerencias: Matriz exponencial, teat dt = a1 teat ea , t2 eat dt = a1 t2 eat 2 teat dt ).

Ecuaciones Diferenciales con coeficientes no


constantes.
1

Ecuacin de Cauchy-Euler.

La ecuacin de Cauchy-Euler es la ecuacin


an xn y (n) (x) + an1 xn1 y (n1) (x) + + a1 xy (1) (x) + a0 y (x) = r (x)
La sustitucin x = et o equivalentemente t = ln (x) transforma esta ecuacin en una ecuacin lineal con
coeficientes constantes.
Ejemplo 1 Resolver
x2 y 00 xy 0 + y = ln (x)
195

(1.1)

Sea x = et y denotemos ye (t) = y (x) = y (et ), entonces


de
y
dt
d2 ye
dt2

dy
dy dx
dy t
de
y
=
e
= et
dx dt
dt
dt
dt
d2 y dx t dy t
d2 y 2t de
y
e +
e = 2e +
dx2 dt
dx
dx
dt
2

dy 2
y
d ye de
2t
=e

dt2
dt2
dt

Reemplazando en la ecuacin dada, se tiene:


2

y
d ye de
de
y
2t 2t

e e
et et
+ ye = t
dt2
dt
dt

esto es

La solucin homognea es

de
y
d2 ye
2
+ ye = t
2
dt
dt

ye (t) = C1 et + C2 tet

Para la solucin particular utilizamos el mtodo de los aniquiladores. El aniquilador de t es D2 , luego


determinamos la solucin particular como solucin de la ecuacin homognea
2

D2 (D 1) ye = 0
yep (t) = a + bt

Se reemplaza en la ecuacin no homognea yep0 (t) = b; yep00 (t) = 0, entonces


0 2b + a + bt = t

y se tiene el sistema
b = 1
2b + a = 0 a = 2
por lo tanto
yep (t) = 2 + t

Luego, la solucin general de la ecuacin lineal con coeficientes constantes asociada a (1.1) es
ye (t) = 2 + t + C1 et + C2 tet

y como t = ln (x), la solucin general de la ecuacin (1.1) es

y (x) = 2 + ln (x) + C1 x + C2 x ln (x)

196

Solucin de ecuaciones diferenciales en series de potencias.

Definicin 1 Una serie de potencias en x a es una serie de la forma


f (x) =

n=0

Cn (x a)

Definicin 2 El intervalo de convergencia es el conjunto de los puntos x tales que la serie converge. Normal|
|x a| = L, esto para aquellos valores
mente el radio de convergencia se determina con L < 1 donde lim |C|Cn+1
n|
n
de x para los cuales L < 1.
Definicin 3 Si
f (x) =

n=0

entonces, la derivada se define como


f 0 (x) =

n=1

Cn (x a)

nCn (x a)

Ejemplo 2 Determinar, si existe, una solucin y (x) =

n1

Cn xn de la ecuacin

n=0

y 0 2xy = 0
Sea y (x) =

Cn xn , calculando su derivada y 0 (x) =

n=0

nCn xn1 , reemplazamos las series en la ecuacin:

n=1

n=1

n=1

nCn xn1 2x

nCn xn1 2

n=0

Cn xn

Cn xn+1

n=0
n+1

C1 + 2C2 x + 3C3 x2 + + nCn xn1 + 2C0 x + 2C1 x2 + 2C2 x3 + + 2Cn x

o bien

= 0
= 0, desarrollando
= 0

C1 + (2C2 2C0 ) x + (3C3 2C1 ) x2 + + (?) xi + = 0


para determinar el coeficiente entre parntesis (?) expresamos el exponente n 1 como i en la primera serie:
n 1 = i, por lo tanto n = i + 1, de manera que
nCn xn1 = (i + 1) Ci+1 xi
Ahora expresamos el exponente n + 1 como i en la segunda serie: n + 1 = i, por lo tanto n = i 1, de manera
que
2Cn xn+1 = 2Ci1 xi

197

por lo tanto el coeficiente (?) es:


(?) xi = ((i + 1) Ci+1 2Ci1 ) xi
de manera que
C1 + (2C2 2C0 ) x + (3C3 2C1 ) x2 + + ((i + 1) Ci+1 2Ci1 ) xi + = 0
o bien
C1 +

X
i=1

{(i + 1) Ci+1 2Ci } xi = 0

(2.1)

Es claro que si C1 = 0 e (i + 1) Ci+1 2Ci1 = 0, i = 1, 2, ... entonces (2.1) es satisfecho y por lo tanto la

X
correspondiente serie y (x) =
Ci xi es una solucin de la ecuacin dada. Determinamos C0 , C1 , C2 , ...
i=0

C1 = 0
i = 1, 0 = (1 + 1) C1+1 2C11 = 2C2 2C0 C2 = C0
i = 2, 0 = (2 + 1) C2+1 2C21 = 3C3 2C1 C3 = C1 = 0
1
1
1
i = 3, 0 = 4C4 2C2 C4 = C2 = C0 C4 = C0
2
2
2
2
i = 4, 0 = 5C5 2C3 C5 = C3 = 0 C5 = 0
5
1
1
1
i = 5, 0 = 6C6 2C4 C6 = C4 =
C0 C6 = C0
3
32
3!
2
i = 6, 0 = 7C7 2C5 C7 = C5 = 0 C7 = 0
7
1
1
1
i = 7, 0 = 8C8 2C6 C8 = C6 =
C0 C8 = C0
4
432
4!

etc., luego

Cn =
Por lo tanto

0 si i es impar
1
n! si i = 2n

y (x) = C0 + C1 x + C2 x2 + C3 x3 + C4 x4 + =

X
X
2
1 2n
1 2 n
y (x) = C0
= C0 ex
x = C0
x
n!
n!
n=0
n=0

2.1

n=0

C0

1 2n
x
n!

Puntos ordinarios y puntos singulares

Consideremos la ecuacin de segundo orden


a2 (x) y 00 (x) + a1 (x) y 0 (x) + a0 (x) y (x) = 0
y 00 (x) + p (x) y 0 (x) + q (x) y (x) = 0
donde p (x) =

a1 (x)
a2 (x) ,

q (x) =

a0 (x)
a2 (x) .

198

(2.2)
(2.3)

Definicin 4 Se dice que x = x0 es un punto singular de la ecuacin (2.2) o (2.3) si p (x) y q (x) son funciones
analticas en x0 , es decir p (x) y q (x) admiten desarrollo en series de potencias de x x0 en una vecindad de
x0 . Si un punto no es punto ordinario, se dice que es un punto singular
Ejemplo 3 Todo valor real de x es un punto ordinario de la ecuacin
y 00 (x) + ex y 0 (x) + (sen (x)) y (x) = 0
ya que
x
ex = 1 + 1!
+
sen (x) = x

x2
2!
x3
3!

+ x3! +
5
+ x5! +

convergen para todo x en R

Ejemplo 4 La ecuacin
y 00 (x) + ln (x) y (x) = 0
singular en x = 0 ya que ln (x) no tiene desarrollo en serie de potencias de (x 0) = x
tiene un0 punto
(ln (x)) = x1 , (ln (x))0 x=0 @
Teorema 1 Si x = x0 es un punto ordinario de la ecuacin (2.2) entonces siempre es posible encontrar dos
soluciones distintas en serie de potencias, soluciones que son de la forma
y (x) =

n=0

Cn (x x0 )

Una solucin en serie converge por lo menos para |x x0 | < R1 , donde R1 es la distancia de x0 al punto
singular real o complejo ms cercano.
Observacin 1 Siempre podemos suponer que x = 0 es un punto ordinario, pues si esto no es as, el cambio
de variables t = x x0 traslada el valor x = x0 a t = 0

Ejemplo 5 Resolver la ecuacin x2 + 1 y 00 + xy 0 y = 0


Como p (x) = x2x+1 , q (x) = x1
2 +1 son funciones analticas en x0 = 0, se concluye que x0 = 0 es un punto
ordinario de la ecuacin y segn el teorema anterior existen dos soluciones de la forma
y (x) =

Cn xn

n=0

Se tiene que
y 0 (x) =
y 00 (x) =

n=1

X
n=2

nCn xn1
n (n 1) Cn xn2

199

reemplazando en la ecuacin:

x2

n=2

X
X
X
2
n (n 1) Cn xn2 + x
nCn xn1
Cn xn
x +1

n (n 1) Cn xn2 +

n=2

n=2

X
n=2

n (n 1) Cn xn +

n (n 1) Cn xn2 + x

n=2

n=1

nCn xn1

n=1

n (n 1) Cn xn2 +

n=1

nCn xn

n=0

X
n=0

= 0

Cn xn

= 0

Cn xn

= 0

n=0

Para poder unir las series en una nica sumatoria, primero se debe "homogeneizar" el exponente de la variable
x, de la siguiente forma:

n=2

n (n 1) Cn xn +

{z
}
k=n
n=2k=2

k=2

n=2

n (n 1) Cn xn2

nCn xn

n=1

|
{z
}
k =n2n=k+2
n=2k=0

k (k 1) Ck xk +

{z }
k=n
n=1k=1

(k + 2) (k + 1) Ck+2 xk +

k=0

k=1

Cn xn

= 0

n=0

{z }
k=n
n=0k=0

kCk xk

Ck xk

= 0

k=0

ahora es necesario que todas las series comienzen desde el mismo ndice, luego las series de inicio menor se
expanden hasta el mayor valor de inicio, en este caso k = 2

k=2

k (k 1) Ck xk +21C2 +32C3 x+

(k + 2) (k + 1) Ck+2 xk +1C1 x+

k=2

k=2

kCk xk C0 C1 x

Ck xk = 0

k=2

agrupando trminos se tiene


2 1 C2 C0 + (3 2 C3 + C1 C1 ) x +

[k (k 1) Ck + (k + 2) (k + 1) Ck+2 + kCk Ck ] xk

= 0

[(k + 2) (k + 1) Ck+2 + {k (k 1) + k 1} Ck ] xk

= 0

k=2

2 1 C2 C0 + 3 2 C3 x +

k=2

2 1 C2 C0 + 3 2 C3 x +

k=2

[(k + 2) (k + 1) Ck+2 + (k 1) (k + 1) Ck ] xk

Luego se tienen las siguientes ecuaciones


2 1 C2 C0
3 2 C3
(k + 2) (k + 1) Ck+2 + (k 1) (k + 1) Ck

200

= 0
= 0
= 0, k 2

= 0

de donde se tienen las siguientes igualdades


C2
C3
Ck+2

1
C0
21
= 0
(k 1) (k + 1)
k1
=
Ck =
Ck , k 2
(k + 2) (k + 1)
k+2
=

note que en la tercera igualdad el trmino k + 1 se simplific porque el valor k = 1, que anula el trmino en
el numerador y denominador, no pertenece al dominio del valor k, que son los enteros mayores que 2.
La tercera igualdad, llamada ley de recurrencia, genera las siguientes igualdades:
k
k
k
k
k
k
k

1
1
1
1
1
= 2, C4 = C2 =
C0 =
C0 = 2
C0 = C4 = 2
C0
4
42
2221
2 21
2 2!
2
= 3, C5 = C3 = 0 C5 = 0
5
3
31
31
31
31
= 4, C6 = C4 =
C0 =
C0 = 3
C0 = C6 = 3
C0
2
2
6
6 2 2!
3 2 2 2!
2 3 2!
2 3!
4
= 5, C7 = C5 = 0 C7 = 0
7
5
531
531
531
531
= 6, C8 = C6 =
C0 =
C0 = 4
C0 = C8 = 4
C0
3
3
8
8 2 3!
4 2 2 3!
2 4 3!
2 4!
6
= 7, C9 = C7 = 0 C9 = 0
9
7
7531
7531
7531
= 8, C10 = C8 =
C0 = 5
C0 = C8 = 5
C0
4
10
10 2 4!
2 5 3!
2 5!

etc. y se tiene en general:

C2n+1

= 0
= (1)n

C2n

1 3 5 7 (2n 3)
C0
2n n!

y entonces
y (x) = C0 + C1 x + C2 x2 + C3 x3 + C4 x4 + + C2n x2n + C2n+1 x2n+1 +
1
1
1 3 5 7 (2n 3)
C0 x4 + 0x5 + + (1)n
C0 x2n
= C0 + C1 x + C0 x2 + 0x3 2
n n!
2
2

2!
2

X
n 1 3 5 7 (2n 3) 2n
+ C1 x
y (x) = C0 1 +
(1)
x
2n n!
n=1
luego
y1 (x) = C0

y2 (x) = C1 x

1 3 5 7 (2n 3) 2n
(1)
1+
x
2n n!
n=1
n

son las dos soluciones linealmente independientes.


201

Para determinar el radio de convergencia de la serie se buscan los puntos singulares de la ecuacin

2
x + 1 y 00 + xy 0 y = 0
x
1
p (x) =
, q (x) = 2
2
x +1
x +1
como x2 + 1 = 0 s y slo si x = i, la expresin x2x+1 no tiene desarrollo en serie de potencias de x en x0 = i
y como la distancia de x0 = 0 a i (o i) es 1 entonces, segn el teorema, el radio de convergencia es 1 (o por lo
menos 1), luego y1 (x) converge x, |x| < 1.
Observacin 2 Como y2 (x) = x es una solucin de la ecuacin, utilizando el mtodo de reduccin de orden
se puede determinar que la solucin y1 (x) es
Z
x2
x2
e 2
2

y1 (x) = e
dx
x2 + 1
luego, se puede concluir que la serie converge a la funcin
1+

(1)

n=1

1 3 5 7 (2n 3) 2n
x2
x =e2
n
2 n!

x2

e 2

dx
x2 + 1

y la integral puede escribirse como


Z

1+

2
x2

dx =
x2 + 1

(1)n

1357(2n3) 2n
x
2n n!

n=1

1
2n
2n n! x

n=0

Ejemplo 6 Resolver la ecuacin

y 00 (1 + x) y = 0
Como x0 = 0 es un punto ordinario, (1 + x es su propio desarrollo en serie de potencias de x) existen dos

X
soluciones en serie de potencias y (x) =
Cn xn . Reemplazando en la ecuacin:
n=0

n=2

n2

n (n 1) Cn x

|
{z
}
k =n2n=k+2
n=2k=0

k=0

n=2

n (n 1) Cn xn2 (1 + x)
n

Cn x

n=0

{z }
k=n
n=0k=0

2 1 C2 C0 +

k=1

Cn xn

= 0

n=0

Cn xn+1

= 0

n=0

|
{z
}
k =n+1n=k1
n=0k=1

(k + 2) (k + 1) Ck+2 xk

Ck1 xk

= 0

{(k + 2) (k + 1) Ck+2 Ck Ck1 } xk

= 0

202

k=0

Ck xk

k=1

Por lo tanto
2C2 C0
(k + 2) (k + 1) Ck+2

= 0
= Ck + Ck1 , k = 1, 2, 3, ...

o bien
C2

Ck+2

1
C0
2
Ck + Ck1
, k = 1, 2, 3, ...
(k + 1) (k + 2)

luego, para los valores de k se tiene


k
k
k
k
k
k

1
1
1
(C1 + C0 ) C3 = C1 + C0
23
3!
3!
1
1
1
2
1
= 2, C4 =
(C2 + C1 ) =
C0 +
C1 C4 = C1 + C0
34
2 3 4
34
4!
4!

1
1
1
1
1
1+3
1
= 3, C5 =
(C3 + C2 ) =
C1 + C0 + C0 C5 = C1 +
C0
45
4 5 3!
3!
2!
5!
5!

1
1
1
1
1
2+4
1+4
2
= 4, C6 =
(C4 + C3 ) =
C1 + C0 + C1 + C0 C6 =
C1 +
C0
56
5 6 4!
4!
3!
3!
6!
6!

1
1
1+3
2
1
1+25
1+3+5
1
= 5, C7 =
(C5 + C4 ) =
C1 +
C0 + C1 + C0 C7 =
C1 +
C0
67
6 7 5!
5!
4!
4!
7!
7!

1
1
1+4
1
1+3
2+4
= 6, C8 =
(C6 + C5 ) =
C1 +
C0 + C1 +
C0
78
78
6!
6!
5!
5!
2+4+6
1 + 4 + 6 (1 + 3)
C8 =
C1 +
C0
8!
8!

1
1
1+3+5
2+4
1+4
1+25
= 7, C9 =
(C7 + C6 ) =
C1 +
C0 +
C1 +
C0
89
89
7!
7!
6!
6!
1 + 2 5 + 7 (2 + 4)
1+3+5+7+74
C9 =
C1 +
C0
9!
9!

1
1
1 + 4 + 6 (1 + 3)
1+25
1+3+5
2+4+6
= 8, C10 =
(C8 + C7 ) =
C1 +
C0 +
C1 +
C0
9 10
9 10
8!
8!
7!
7!
2+4+8+258
1+4+6+8+63+83+85
C10 =
C1 +
C0
10!
10!
=

1, C3 =

etc.
Luego

1
1
1
2
1
1
1+3
2
3
4
y (x) = C0 + C1 x + C0 x +
C1 + C0 x +
C1 + C0 x +
C1 +
C0 x5 +
2
3!
3!
4!
4!
5!
5!

2+4
1+4
1+25
1+3+5
6
+
C1 +
C0 x +
C1 +
C0 x7 +
6!
6!
7!
7!

2+4+6
1 + 2 5 + 7 (2 + 4)
1 + 4 + 6 (1 + 3)
8
+
C1 +
C0 x +
C1 +
8!
8!
9!
203

1+3+5+7+74
2+4+8+258
1+4+6+8+63+83+85
C0 x9 +
C1 +
C0 x10 + ...
9!
10!
10!
entonces, las soluciones son, entonces

1
1
1 + 3 5 1 + 4 6 1 + 3 + 5 7 1 + 4 + 6 (1 + 3) 8
1
y1 (x) = C0 1 + x2 + x3 + x4 +
x +
x +
x +
x
2
3!
4!
5!
6!
7!
8!

2 4
1 5 2 + 4 6 1 + 2 5 7 2 + 4 + 6 8 1 + 2 5 + 7 (2 + 4) 9
1 3
y2 (x) = C1 x + x + x + x +
x +
x +
x +
x
3!
4!
5!
6!
7!
8!
9!
... +

Un ejemplo de ecuacin, cuya solucin es importante en el rea de la fsica es la llamada ecuacin de Legendre:

1 x2 y 00 2xy 0 + r (r + 1) y = 0, r N

Sea y =

Cn xn y reemplazamos en la ecuacin:

n=0

n=2

X
X

X
1 x2
n (n 1) Cn xn2 2x
nCn xn1 + r (r + 1)
Cn xn

n (n 1) Cn xn2

|
{z
}
k =n2n=k+2
n=2k=0

k=0

... +

k=2

n=2

n=2

n (n 1) Cn xn

{z
}
k=n
n=2k=2
k

(k + 2) (k + 1) Ck+2 x

k=2

n=1

2nCn xn

n=1

{z
}
k=n
n=1k=1

k (k 1) Ck x

k=1

r (r + 1) Cn xn

|
k

2kCk x +

{z
}
k=n
n=0k=0

r (r + 1) Ck xk

= 0

k=0

2 1 C2 + 3 2 C3 x 2 1 C1 x + r (r + 1) C0 + r (r + 1) C1 x + ... =

X
X
X
(k + 2) (k + 1) Ck+2 xk
k (k 1) Ck xk
2kCk xk +
r (r + 1) Ck xk = 0
k=2

k=1

k=0

2 1 C2 + r (r + 1) C0 + (3 2 C3 2 1 C1 + r (r + 1) C1 ) x + ... =

X
+
{(k + 2) (k + 1) Ck+2 k (k 1) Ck 2kCk + r (r + 1) Ck } xk = 0
k=2

y entonces:

= 0

n=0

= 0

n=0

2C2 + r (r + 1) C0
6C3 + (r 1) (r + 2) C1
(k + 2) (k + 1) Ck+2 + (r k) (r + k + 1) Ck

= 0
= 0
= 0, k = 2, 3, 4, ...

o bien
C2
C3
Ck+2

r (r + 1)
C0
2!
(r 1) (r + 2)
=
C1
3!
(r k) (r + k + 1)
=
Ck , k = 2, 3, 4, ...
(k + 1) (k + 2)
=

204

y entonces
(r 2) (r + 3)
(r 2) (r + 3) r (r + 1)
(r 2) r (r + 1) (r + 3)
C2 =
C0 C4 =
C0
34
3 4 2!
4!
(r 3) (r + 4)
(r 3) (r + 4) (r 1) (r + 2)
(r 3) (r 1) (r + 2) (r + 4)
3, C5 =
C3 =
C1 C5 =
C1
45
4 5 3!
5!
(r 4) (r + 5)
(r 4) (r + 5) (r 2) r (r + 1) (r + 3)
4, C6 =
C4 =
C0
56
5 6 4!
(r 4) (r 2) r (r + 1) (r + 3) (r + 5)

C0
6!
(r 5) (r + 6)
(r 5) (r + 6) (r 3) (r 1) (r + 2) (r + 4)
5, C7 =
C5 =
C1
67
6 7 5!
(r 5) (r 3) (r 1) (r + 2) (r + 4) (r + 6)

C1
7!

= 2, C4 =

C6

C7

en general
C2n
C2n+1

(r (2n 2)) (r 4) (r 2) r (r + 1) (r + 3) (r + (2n 1))


C0
(2n)!
n (r (2n 1)) (r 3) (r 1) (r + 2) (r + 4) (r + 2n)
= (1)
C1
(2n + 1)!
= (1)n

finalmente, la solucin y (x) =

Cn xn es:

n=0

r (r + 1)
(r 1) (r + 2)
(r 2) r (r + 1) (r + 3)
C0 x2
C1 x3 +
C0 x4 +
2!
3!
4!
(r 3) (r 1) (r + 2) (r + 4)
(r 4) (r 2) r (r + 1) (r + 3) (r + 5)
+
C1 x5
C0 x6
5!
6!
(r 5) (r 3) (r 1) (r + 2) (r + 4) (r + 6)

C1 x7 +
7!
(r (2n 2)) (r 4) (r 2) r (r + 1) (r + 3) (r + (2n 1))
... + (1)n
C0 x2n +
(2n)!
(r (2n 1)) (r 3) (r 1) (r + 2) (r + 4) (r + 2n)
+ (1)n
C1 x2n+1 +
(2n + 1)!

r (r + 1) 2 (r 2) r (r + 1) (r + 3) 4 (r 4) (r 2) r (r + 1) (r + 3) (r + 5) 6
x +
x
x +
= C0 1
2!
4!
6!

n (r (2n 2)) (r 4) (r 2) r (r + 1) (r + 3) (r + (2n 1)) 2n


+ (1)
x + +
(2n)!

(r 1) (r + 2) 3 (r 3) (r 1) (r + 2) (r + 4) 5
+ C1 x
x +
x
3!
5!
(r 5) (r 3) (r 1) (r + 2) (r + 4) (r + 6) 7
...
x + ...
7!

n (r (2n 1)) (r 3) (r 1) (r + 2) (r + 4) (r + 2n) 2n+1


+
+ (1)
x
(2n + 1)!

y (x) = C0 + C1 x

205

luego

r (r + 1) 2 (r 2) r (r + 1) (r + 3) 4 (r 4) (r 2) r (r + 1) (r + 3) (r + 5) 6
y1 (x) = C0 1
x +
x
x +
2!
4!
6!

(r (2n 2)) (r 4) (r 2) r (r + 1) (r + 3) (r + (2n 1)) 2n


+ (1)n
x +
(2n)!

(r 1) (r + 2) 3 (r 3) (r 1) (r + 2) (r + 4) 5
y2 (x) = C1 x
x +
x
3!
5!
(r 5) (r 3) (r 1) (r + 2) (r + 4) (r + 6) 7
...
x + ...
7!

n (r (2n 1)) (r 3) (r 1) (r + 2) (r + 4) (r + 2n) 2n+1


+
x
+ (1)
(2n + 1)!
Observacin 3 Si r es par, es decir r = 2m, para algn m (fijo) entonces y1 (x) es un polinomio de grado
r = 2m, en efecto el coeficiente de x2(m+1) , donde m + 1 = n, tiene el factor
n

z }| {
r 2(m + 1) 2 = r 2m = 0,

adems, el coeficiente de x2n , n > m + 1 contiene al coeficiente


n

z }| {
r 2(m + 1) 2 = r 2m = 0,

Observacin 4 Si r es impar, es decir r = 2m + 1, para algn m (fijo) entonces y2 (x) es un polinomio de


grado r = 2m + 1, segn el argumento dado en la observacin anterior.
Resumiendo, si r es un entero positivo, la ecuacin de Legendre siempre posee una solucin que es un
polinomio de grado r. Estos polinomios son llamados polinomios de Legendre cuando las constantes C0 y C1 se
escogen como:
C0
C1
C0
C1

= 1 si r = 0
= 1 si r = 1
r 1 3 5 (r 1)
= (1) 2
si r = 2, 4, 6, 8, ...
2 4 6 r
r1
1 3 5 r
= (1) 2
si r = 3, 5, 7, 9, ...
2 4 6 (r 1)

As, por ejemplo


r
r
r
r
en general, Pr (x)

= 0, P0 (x) = 1 es solucin de 1 x2 y 00 2xy 0 = 0

= 1, P1 (x) = x es solucin de 1 x2 y 00 2xy 0 + 2y = 0

1
= 2, P2 (x) = 1 3x2 es solucin de 1 x2 y 00 2xy 0 + 6y = 0
2

3
10
= 3, P3 (x) =
x x3 es solucin de 1 x2 y 00 2xy 0 + 12y = 0
2
6

es el polinomio de Legendre, solucin de la ecuacin 1 x2 y 00 2xy 0 + r (r + 1) y = 0.


206

2.2

Puntos Singulares Regulares e Irregulares

Definicin 5 Un punto singular (no ordinario) x = x0 de la ecuacin


a2 (x) y 00 + a1 (x) y 0 + a0 y = 0
o de la ecuacin normalizada
y 00 + p (x) y 0 + q (x) y = 0,
se denomina punto singular regular si (x x0 ) p (x) y (x x0 )2 q (x) son funciones analticas en x0 , es decir
pueden expresarse como series de potencias de x x0 en una vecindad de x0 .
Teorema 2 Si x0 es un punto singular regular de la ecuacin
y 00 + p (x) y 0 + q (x) y = 0,
entonces existe al menos una solucin de esta ecuacin en serie de potencias
y (x) = (x x0 )r

n=0

Cn (x x0 )n =

n=0

Cn (x x0 )n+r

donde r es una cantidad que se debe determinar.


Si x0 = 0, entonces y (x) =

n=0

ecuacin indicial)
donde

Cn xn+r = xr

Cn xn y r es solucin de la ecuacin cuadrtica (llamada

n=0

r (r 1) + p0 r + q0 = 0
xp (x) = p0 + p1 x + p2 x2 +
x2 q (x) = q0 + q1 x + q2 x2 +

En la resolucin de la ecuacin
a (x) y 00 + b (x) y 0 + c (x) y = 0
mediante el mtodo de Frobenius (series del tipo xr

Cn xn ), el valor de r lo obtenemos como solucin de la

n=0

ecuacin indicial

r (r 1) + rp0 + q0 = 0

Si las races de este polinomio son reales, digamos r2 < r1 , distinguimos tres posibilidades:
i. r2 < r1 y r1 r2 no es un entero (natural). En este caso la ecuacin tiene dos soluciones linealmente
independiente en forma de series de potencias de la forma
y1 (x) =
y2 (x) =

n=0

X
n=0

207

Cn xn+r1
Bn xn+r2

ii. r2 < r1 y r1 r2 es un entero (natural). En este caso existen dos soluciones linealmente independientes de
la ecuacin (5.3) y son de la forma:

y1 (x) =

n=0

Cn xn+r1 , C0 6= 0

y2 (x) = Cy1 (x) ln (x) +

Bn xn+r2

n=0

note que C podra ser igual a 0 en y2 (x). Si este es el caso, la ecuacin tiene dos soluciones en forma de
serie de Frobenius. Esto no se puede saber por anticipado.
iii. Si r1 = r2 = r, siempre existen dos soluciones linealmente independientes de la ecuacin (5.3), que tienen
la forma:
y1 (x) =

n=0

Cn xn+r , C0 6= 0

y2 (x) = y1 (x) ln (x) +

bn xn+r

n=0

Para determinar y1 (x) se procede de la forma habitual y para determinar y2 (x) se utiliza la frmula de
Abel:
Z R p(x)dx
e
y2 (x) = y1 (x)
2 dx
(y1 (x))
A continuacin resolveremos un ejemplo particular del mtodo de Frobenius.
Ejemplo 7 Resolver la ecuacin de Bessel

x2 y 00 + xy 0 + x2 2 y = 0

La ecuacin de Bessel tiene un punto singular regular en x0 = 0, en efecto


(x 0)

x
x2

x2 2
(x 0)
x2
2

= 1=1+

0xn

n=1
2

= x = 2 x0 + 0x + 1x2 + 0x3 +

Segn el teorema anterior, existe una solucin de esta ecuacin de la forma y (x) =

n=0

p0 = 1, q0 = 2 , la ecuacin indicial es
r (r 1) + p0 r + q0 = r (r 1) + r 2 = r2 2 = 0

208

Cn xn+r . Como

Reemplazando y (x) =

Cn xn+r en la ecuacin, se tiene:

n=0

x2 y 00 + xy 0 + x2 2 y

X
X

X
x2
(n + r) (n + r 1) Cn xn+r2 + x
(n + r) Cn xn+r1 + x2 2
Cn xn+r
n=0

xr

n=0

n=0

n=0

(n + r) (n + r 1) Cn xn+r +

(n + r) (n + r 1) Cn xn +

(n + r) Cn xn+r +

n=0

n=0

(n + r) Cn xn +

Cn xn+r+2

Cn xn+2

n=0

Cn 2 xn+r

n=0

Cn 2 xn

= 0
= 0
= 0

= 0

{z
} |
{z
}
|
{z
}
|
{z
}

k=n
k=n
k =n+2n=k2
k=n

n=0k=0
n=0k=0
n=0k=2
n=0k=0
(
)

X
X
X
X
r
k
k
k
2 k
x
(k + r) (k + r 1) Ck x +
(k + r) Ck x +
Ck2 x
Ck x
= 0
n=0

n=0

k=0

n=0

k=0

k=2

k=0

k=0

k=2

k=0

xr r (r 1) C0 + r (r + 1) C1 x + rC0 + (r + 1) C1 x 2 C0 2 C1 x + ... =
)

X
X
X
X
k
k
k
2 k
... +
(k + r) (k + r 1) Ck x +
(k + r) Ck x +
Ck2 x
Ck x
= 0
k=0

x
r (r 1) + r 2 C0 + r (r + 1) + (r + 1) 2 C1 x =
)

k
2
+
= 0
(k + r) (k + r 1) Ck + (k + r) Ck + Ck2 Ck x
r

k=2

(
)
h

i
X

2
xr r2 2 C0 + r2 + 2r + 1 2 C1 x +
= 0
(k + r) 2 Ck + Ck2 xk
k=2

Por lo tanto, si hacemos r = 0, entonces r = y reemplazando r = en la igualdad anterior obtenemos:


(
)

k
[k (k + 2) Ck + Ck2 ] x
=0
x C1 (1 + 2) x +
k=2

y por lo tanto, es suficiente:

C1
Ck k (k + 2) + Ck2

= 0
= 0, k = 2, 3, 4, ...

o bien
C1

= 0

Ck

Ck2
, k = 2, 3, 4, ...
k (k + 2)
209

evaluando los valores de k se tiene


C0
C0
C0
=
C2 = 2
2 (2 + 2)
2 2 (1 + )
2 (1 + )
C1
3, C3 =
=0
3 (3 + 2)
C2
C0
1
C0
4, C4 =
=
C4 =
4 (4 + 2)
4 2 (2 + ) 22 (1 + )
2 24 (2 + ) (1 + )
C3
5, C5 =
=0
5 (5 + 2)
C4
C0
1
C0
6, C6 =
=
C6 =
4
6
6 (6 + 2)
6 2 (3 + ) 2 2 (2 + ) (1 + )
3 2 2 (3 + ) (2 + ) (1 + )
C5
7, C7 =
=0
7 (7 + 2)
C6
1
C0
8, C8 =
=
8 (8 + 2)
8 2 (4 + ) 3 2 26 (3 + ) (2 + ) (1 + )
C0
8
4 3 2 2 (4 + ) (3 + ) (2 + ) (1 + )

= 2, C2 =

C8

se verifica fcilmente que


C2n
C2n+1

= (1)n

22n

= 0

C0
(n!) (1 + ) (2 + ) (n + )

y la solucin es
y (x) =

n=0

Cn xn+r = C0

(1)

n=0

1
x2n+
22n (n!) (1 + ) (2 + ) (n + )

recuerde que r = .
Definicin 6 La funcin gamma est definida por: : (0, ) R
Z
et t1 dt, > 0
() =
0

Al integrar por partes, es "fcil" ver que


(1)
(1) = 1
(2) (1 + ) = ()
en particular, de (2), se tiene (n + 1) = n!
Empleando esta propiedad y como
n

C2n = (1)

C0
22n (n!) (1 + ) (2 + ) (n + )
210

entonces:
(2 + ) = (1 + (1 + )) = (1 + ) (1 + )
(3 + ) = (1 + (2 + )) = (2 + ) (2 + ) = (2 + ) (1 + ) (1 + )
(4 + ) = (1 + (3 + )) = (3 + ) (3 + ) = (3 + ) (2 + ) (1 + ) (1 + )
luego
(1 + n + ) = (n + ) (n + ) = (n + ) (n 1 + ) (3 + ) (2 + ) (1 + ) (1 + )
y entonces, si se considera
C0 =
se obtiene:
y1 (x) =
o bien

C0
2 (1 + )

X
C0
1
(1)n 2n
x2n+

2 (1 + ) n=0
2 (n!) (1 + ) (2 + ) (n + )

y1 (x) = C0

(1)n

n=0

o bien

x 2n+
1
(n!) (1 + ) (2 + ) (n + ) (1 + ) 2

y1 (x) = C0

(1)n

n=0

x 2n+
1
n! (1 + n + ) 2

Cuando C0 = 1, la funcin anterior se denomina funcin de Bessel de 1a clase.


Todos los clculos anteriores, con r = , determinan la otra solucin
y2 (x) = C0

(1)

n=0

x 2n
1
n! (1 + n ) 2

Si C0 = 1, y2 (x) se denomina funcin de Bessel de 1a clase.


Resolveremos ejemplos que muestran la aplicacin del mtodo de Frobenius.
Ejemplo 8 Resolver la ecuacin
2xy 00 + (1 + x) y 0 + y = 0
en torno a x0 = 0.
Normalizando la ecuacin se tiene:
y 00 +
p (x) =

1
2x

+ 12 , q (x) =

1
2x ,

1+x 0
1
y +
y=0
2x
2x

luego x0 = 0 es un punto singular. Adems

p (x) =
q (x) =

1
1
1 x
1 x
+ xp (x) = + = + + 0x2 + 0x3 +
2x 2
2 2
2 2
1
x
x
x2 q (x) = = 0 + + 0x2 + 0x3 +
2x
2
2
211

luego, la ecuacin indicial es


r (r 1) + p0 r + q0
1
r (r 1) + r + 0
2
2r (r 1) + r
2r2 r
r (2r 1)

= 0
= 0
= 0
= 0
= 0

por lo tanto r1 = 12 , r2 = 0, son las soluciones de la ecuacin indicial.

X
Entonces, buscamos soluciones del tipo y (x) =
Cn xn+r .
n=0

2x

xr

n=0

(n + r) (n + r 1) Cn xn+r2 + (1 + x)

(n + r) Cn xn1

(n + r) Cn xn+r1 +

n=0

Cn xn+r

n=0

k=1

(n + r) Cn xn +

= 0

= 0
Cn xn

n=0
n=0
n=0
n=0

|
{z
}
|
{z
}
|
{z
}
| {z }

k
=
n

n
=
k
+
1
k
=
n

n
=
k
+
1
k
=
n
k
=
n

n = 0 k = 1
n = 0 k = 1
n=0k=0
n=0k=0
(
)

X
X
X
X
r
k
k
k
k
x
2 (k + r + 1) (k + r) Ck+1 x +
(k + 1 + r) Ck+1 x +
(k + r) Ck x +
Ck x
= 0

xr

2 (n + r) (n + r 1) Cn xn1 +

k=1

2r (r 1) C0 x1 + rC0 x1 +
r

k=0

k=0

k=0

[{2 (k + r + 1) (k + r) + (k + 1 + r)} Ck+1 + {(k + r) + 1} Ck ] xk


1

r (2r 1) C0 x

[(k + r + 1) (2 (k + r) + 1) Ck+1 + (k + r + 1) Ck ] x

k=0

)
)

como r (2r 1) es la ecuacin indicial, entonces para r1 = 12 , r2 = 0, la ecuacin se anula y C0 6= 0, luego


(k + r + 1) (2 (k + r) + 1) Ck+1 + (k + r + 1) Ck = 0 Ck+1 =

(k + r + 1)
Ck
(k + r + 1) (2 (k + r) + 1)

Entonces,
i) r1 =

1
2

Ck+1

k + 32
1

=
Ck Ck+1 =
Ck , k = 0, 1, 2, 3...
3
2 (k + 1)
k + 2 (2k + 2)
212

= 0

= 0

luego
k
k
k
k

1
= 0 C1 = C0
2

1
1
1
1
= 1 C2 =
C1 =
C0 C2 = 2
C0
22
22
2
2 2!
1
1 1
1
= 2 C3 =
C2 =
C0 C3 = 3
C0
23
2 3 22 2!
2
3!

1
1
1
1
= 3 C4 =
C3 =
4
C0 C4 = 4
C0
24
24
2 3!
2 4!
..
.
n
(1)
C0
= n 1 Cn = n
2 n!

y entonces

C0 n
C0
C0 3
C0 4
C0 2
n
y1 (x) = x
C0
x+ 2
x 3
x + 4
x + + (1) n
x +
2 1!
2 2!
2 3!
2 4!
2 n!

1
1
1 1+ 1
1
1
1
n
2+ 12
3+ 12
4+ 12
n+ 12
2
2
y1 (x) = C0 x
+ 2
3
+ 4
+ + (1) n
+
x
x
x
x
x
2 1!
2 2!
2 3!
2 4!
2 n!

X
1
1
1
y1 (x) = C0 x 2 +
(1)n n
xn+ 2
2

n!
n=1

!
X
1
1
n
(1) n
y1 (x) = C0
xn+ 2 , definida para x > 0
2

n!
n=0
1
2

ii) r = 0
Ck+1 =

(k + 1)
1
Ck Ck+1 =
Ck , k = 0, 1, 2, 3...
(k + 1) (2k + 1)
2k + 1

luego
k
k
k
k

1
= 0 C1 = C0
1

1
1
1
1
= 1 C2 =
C1 =
C0 C2 =
C0
3
3
1
31
1
1 1
1
= 2 C3 =
C2 =
C0 C3 =
C0
5
5 3 1
5

31

1
1
1
1
= 3 C4 =
C3 =

C0 C4 =
C0
7
7
531
7531
..
.
n
(1)
C0
= n 1 Cn =
(2n 1) 7 5 3 1
213

y entonces

C0
C0
C0
C0 2
C0
n
x+
x
x3 +
x4 + + (1)
xn +
y2 (x) = x0 C0
1
31
531
7531
(2n 1) 7 5 3 1

X
1
n
y2 (x) = C0 1 +
(1)
xn
(2n

1)

1
n=1
Entonces la solucin general es

X
X
1
1
n
n
n+ 12
n
(1) n
(1)
+ C2 1 +
y (x) = C1
x
x , C1 , C2 R
2 n!
(2n 1) 7 5 3 1
n=0
n=1
Ejemplo 9 Resolver
xy 00 + (x 6) y 0 3y = 0

en torno a x0 = 0.

Se verifica que x0 = 0 es un punto singular regular, normalizando


y 00 +

x6 0
3
y
y=0
x
x
| {z }
|{z}
p(x)

q(x)

como p (x) y q (x) no son analticas en torno x0 = 0, el punto es singular, pero


xp (x) = 6 + x + 0x2 + 0x3 + p0 = 6
x2 q (x) = 0 3x + 0x2 + 0x3 + q0 = 0
La ecuacin indicial es:
r (r 1) + (6) r + 0 = 0
r2 r 6r = r2 7r = 0
r (r 7) = 0 r2 = 0, r1 = 7
r1 r2 = 7 N
Se determina una solucin y (x) =

Cn xn+r . Reemplazando en la ecuacin y haciendo los cambios usuales

n=0

en los ndices, se obtiene:


(
xr

r (r 7) C0 x1 +

k=0

[(k + r + 1) (k + r 6) Ck+1 + (k + r 3) Ck ] xk

=0

de donde
r (r 7) C0
(k + r + 1) (k + r 6) Ck+1 + (k + r 3) Ck

= 0
= 0

Ck+1

=
214

k+r3
Ck , k = 0, 1, 2, 3...
(k + r + 1) (k + r 6)

i. r1 = 7
Ck+1

k+4
Ck , k = 0, 1, 2, 3...
(k + 8) (k + 1)
4
7!4!
0 C1 =
C0 =
C0
81
3!8!1!

5
5
7!5!
7!4!
1 C2 =
C1 =

C0 C2 =
C0
92
92
3!8!1!
3!9!2!

6
6
7!6!
7!5!
2 C3 =
C2 =
C0 C3 =
C0
10 3
10 3 3!9!2!
3!10!3!

7
7
7!7!
7!6!
3 C4 =
C3 =

C0 C4 =
C0
11 4
11 4
3!10!3!
3!11!4!

8
8
7!8!
7!7!
4 C5 =
C4 =
C0 C5 =
C0
12 5
12 5 3!11!4!
3!12!5!

9
9
7!9!
7!8!
5 C6 =
C5 =

C0 C6 =
C0
13 6
13 6
3!12!5!
3!13!6!

10
10
7!10!
7!9!
6 C7 =
C6 =
C0 C7 =
C0
14 7
14 7 3!13!6!
3!14!7!
..
.
7! (n + 3)!
(1)n
C0
3! (n + 7)!n!

X
7! (n + 3)!
y1 (x) =
(1)n
C0 xn+7
3!
(n
+
7)!n!
n=0

Cn

slo hemos obtenido una solucin. La experiencia recomienda considerar primero r2 = 0, la menor raz,
observe por qu.
ii. r2 = 0
Ck+1

k3
Ck , k = 0, 1, 2, 3..., k 6= 6
(k + 1) (k 6)
3
1
0 C1 =
C0 = C0
1 (6)
2

2
2
1
1
1 C2 =
C1 =
C0 C2 =
C0
2 (5)
25
2
25

1
1
1
1
2 C3 =
C2 =
C0 C3 = C0
3 (4)
34 25
5!
3 C4 = 0
1
4 C5 =
C4 = 0
5 (2)
2
5 C6 =
C5 = 0
6 (1)
6 0C7 = 3C6 0 = 0

215

k
k
k

C7+n

4
7!4!
C7 =
C7
81
3!8!1!

5
5
7!5!
7!4!
= 8 C9 =
C8 =

C7 C9 =
C7
92
92
3!8!1!
3!9!2!

7!5!
6
6
7!6!
= 9 C10 =
C9 =
C7 C10 =
C7
10 3
10 3 3!9!2!
3!10!3!
..
.
7! (n + 3)!
= (1)n
C7 , n = 1, 2, 3, ...
3! (n + 7)!n!

1 3
1
1
7! (n + 3)!
(1)n
x +
C7 xn+7
y2 (x) = C0 1 x + x2
2
10
120
3!
(n
+
7)!n!
n=0
= 7 C8 =

luego, la solucin y2 (x) no contiene el logaritmo natural, pues C la constante que lo acompaa es cero,
entonces la solucin general sera

7! (n + 3)!
1 3
1
1 2
n 7! (n + 3)!
n+7
y (x) =
(1)
+ C0 1 x + x
(1)
C0 x
x +
C7 xn+7
3!
(n
+
7)!n!
2
10
120
3!
(n
+
7)!n!
n=0
n=0

X
1 3
1
1
n 7! (n + 3)!
= C1
(1)
xn+7 + C2 1 x + x2
x
3! (n + 7)!n!
2
10
120
n=0

Ejemplo 10 Resolver
xy 00 + 3y 0 y = 0
en torno a x0 = 0.
Normalizando la ecuacin:
y 00 +

3 0 1
y y=0
x
x

luego
p (x) =

3
x

q (x) =

1
x

y x0 = 0 es un punto singular. Adems:


xp (x) = 3 + x + 0x2 + 0x3 + p0 = 3
x2 q (x) = 0 x + 0x2 + 0x3 + q0 = 0
por lo tanto, x0 = 0 es un punto singular regular. La ecuacin indicial es:
r (r 1) + 3r + 0 = 0 r2 + 2r = 0 r1 = 0, r2 = 2

216

Entonces, buscamos soluciones del tipo y (x) =

Cn xn+r .

n=0

n=0

(n + r) (n + r 1) Cn xn+r2 + 3

n=0

n1

(n + r) Cn xn+r1

Cn xn+r

n=0

= 0

= 0
(n + r) (n + r 1) Cn x
3 (n + r) Cn x
Cn xn

n=0
n=0
n=0

|
{z
}
|
{z
}
| {z }

k
=
n

n
=
k
+
1
k
=
n

n
=
k
+
1
k
=
n

n = 0 k = 1
n = 0 k = 1
n=0k=0
(
)

X
X
X
r
k
k
k
x
(k + r + 1) (k + r) Ck+1 x +
3 (k + 1 + r) Ck+1 x
Ck x
= 0
k=1

r (r 1) C0 x

n1

k=1

+ 3rC0 x
r

k=0

[{(k + r + 1) (k + r) + 3 (k + 1 + r)} Ck+1 Ck ] x


1

r (r + 2) C0 x

k=0

de donde

k=0

[(k + r + 1) (k + r + 3) Ck+1 Ck ] x

r (r + 2) = 0
(k + r + 1) (k + r + 3) Ck+1 Ck = 0, k = 0, 1, 2, . . .
luego
Ck+1
r1

Ck
, k = 0, 1, 2, . . .
(k + r + 1) (k + r + 3)
= 0, r2 = 2
=

reemplazando los valores de r, se tiene


i. r1 = 0
Ck+1

1
Ck , k = 0, 1, 2, 3...
(k + 1) (k + 3)
1
2
0 C1 =
C0 =
C0
13
1!3!

1
1
2
2
1 C2 =
C1 =
C0 C2 =
C0
24
2 4 1!3!
2!4!

1
1
2
2
2 C3 =
C2 =
C0 C3 =
C0
35
3 5 2!4!
3!4!

2
1
1
2
3 C4 =
C3 =
C0 C4 =
C0
46
4 6 2!4!
4!6!
217

)
)

= 0

= 0

k
k
k

Cn

1
1
2
2
C4 =
C0 C5 =
C0
57
5 7 4!6!
5!7!

1
1
2
2
= 5 C6 =
C5 =
C0 C6 =
C0
68
6 8 6!8!
6!8!

2
1
1
2
= 6 C7 =
C6 =
C0 C7 =
C0
79
7 9 6!8!
7!9!
..
.
2!
=
C0
n! (n + 2)!

X
2
y1 (x) =
C0 xn
n!
(n
+
2)!
n=0
= 4 C5 =

y se ha obtenido una solucin. Para la segunda solucin, consideremos el siguiente valor de r


ii. r2 = 2
Ck+1

Cn

1
Ck , k = 0, 1, 2, 3..., k 6= 1
(k 1) (k + 1)
1
1
0 C1 =
C0 = C0 C1 = C0
1 (1)
1
1 0C2 = C1 C1 = 0 C0 = 0
1
2
2 C3 =
C2 C3 =
C2
13
1!3!

1
1
2
2
3 C4 =
C3 =
C2 C4 =
C2
24
2 4 1!3!
2!4!

1
1
2
2
4 C5 =
C4 =
C2 C5 =
C2
35
3 5 2!4!
3!5!

1
1
2
2
5 C6 =
C5 =
C2 C6 =
C2
46
4 6 2!4!
4!6!

1
1
2
2
6 C7 =
C6 =
C2 C7 =
C2
57
5 7 4!6!
5!7!

1
1
2
2
7 C8 =
C7 =
C2 C8 =
C2
68
6 8 5!7!
6!8!

1
1
2
2
8 C9 =
C8 =
C2 C9 =
C2
79
7 9 6!8!
7!9!

1
1
2
2
9 C10 =
C9 =
C2 C10 =
C2
8 10
8 10 7!9!
8!10!
..
.
2
C2 , n = 2, 3, 4, ...
n! (n + 2)!

218

y2 (x) =

X
2
2
C2 xn2 =
C2 xn
n!
(n2)!
n!
(n
+
2)!
n=2
n=0

luego, la solucin y2 (x) es idntica a y1 (x), segn este mtodo, luego emplearemos la frmula de Abel para
determinar una segunda ecuacin, es decir:
Z R p(x)dx
e
y2 (x) = y1 (x)
dx
(y1 (x))2
aqu
R
1
3
e p(x)dx = 3
x
x
2
2 3
2 4
2 2
y1 (x) = 1 +
x+
x +
x +
x , si C0 = 1
1!3!
2!4!
3!5!
4!6!
1
1 3
1 4
1
y1 (x) = 1 + x + x2 +
x +
x + ...
6
24
360
8640

p (x) =

luego:
[y1 (x)]2

2
1
1 3
1 4
1
1
1 + x + x2 +
x +
x +
x5 ...
6
24
360
8640
302400

1
1 3
1 4
1
1
1 3
1 4
1
1
1
=
1 + x + x2 +
1 + x + x2 +
x +
x +
x5 ...
x +
x +
x5 ...
6
24
360
8640
302400
6
24
360
8640
302400
1 3
11 4
143 5
2
7 2
= 1+ x+ x + x +
x +
x +
3
36
30
2880
453600
=

para determinar

1
,
(y1 (x))2

se efecta la divisin

1
11 4
143 5
2
19 3
433 4
1613 5
2
7
1
1 : 1 + x + x2 + x3 +
x +
x + = 1 x + x2
x +
x
x +
3
36
30
2880
453600
3
4
270
25920
453600
luego integrando, se tiene:
y2 (x) =
=
=
=

1
19 3
433 4
1613 5
2
1 2
y1 (x)
1 x+ x
x +
x
x + dx
x3
3
4
270
25920
453600

1 2 2 1 1
1613 3
19
433 2
y1 (x) x + x + ln(x)
x+
x
x +
2
3
4
270
51840
1360800

1
1 2 2 1
19
1613 3
433 2
y1 (x) ln (x) + y1 (x) x + x
x+
x
x +
4
2
3
270
51840
1360800
1
1
25
209 3
1
23
473 2
y1 (x) ln (x) x2 + x1 +
+
x
x
x +
4
2
3
72 540
25920
30240
Z

Por lo tanto, las soluciones son:

1 3
1 4
1
1 2
2
n
y1 (x) =
C0 x = C0 1 + x + x +
x +
x + ...
n! (n + 2)!
6
24
360
8640
n=0

y2 (x) =

1
1
1
25
209 3
23
473 2
y1 (x) ln (x) x2 + x1 +
+
x
x
x +
4
2
3
72 540
25920
30240
219

y la solucin general es:

1
1 3
1 4
1
1
25
1
23
1
y (x) = C1 1 + x + x2 +
x +
x + ... + C2
y1 (x) ln (x) x2 + x1 +
+
x
6
24
360
8640
4
2
3
72 540

209 3
473 2
...
x
x +
25920
30240
Ejemplo 11 Resolver la ecuacin
x2 y 00 + 3xy 0 + (1 + x) y = 0
en torno a x0 = 0.
La ecuacin normalizada es:
y 00 +

3 0 1+x
y=0
y +
x
x2

luego
p (x) =
q (x) =

3
xp (x) = 3
x
1
1
+ x2 q (x) = 1 + x
x2
x

entonces x0 = 0 es un punto singular regular, cuya ecuacin indicial es


r (r 1) + 3r + 1 = 0 (r + 1)2 = 0
por lo tanto, tiene una nica raz: r = 1. Luego, admite una solucin de la forma y (x) =
Reemplazando en la ecuacin, se tiene:
x2

n=0

xr

xr

n=0

(n + r) (n + r 1) Cn xn+r2 + 3x

n=0

(n + r) (n + r 1) Cn xn +

(n + r) (n + r 1) Cn xn +

(n + r) Cn xn+r1 + (1 + x)

n=0

3 (n + r) Cn xn +

n=0

Cn xn +

n=0

3 (n + r) Cn xn +

Cn xn

n=0

Cn xn+1

Cn xn+r .

n=0

Cn xn+r

= 0

= 0

Cn xn+1

n=0

= 0

{z
} |
{z
}
| {z }
|
{z
}

n=k
n=k
n=k
k =n+1n=k1

n=0k=0
n=0k=0
n=0k=0
n=0k=1
(
)

X
X
X
X
r
k
k
k
k
x
(k + r) (k + r 1) Ck x +
3 (k + r) Ck x +
Ck x +
Ck1 x
= 0
n=0

n=0

k=0

k=0

{r (r 1) + 3r + 1} C0 xr +
220

k=1

n=0

k=0

k=1

{[(k + r) (k + r + 2) + 1] Ck + Ck1 } xk

= 0

luego
(r + 1)2 C0
{(k + r) (k + r + 2) + 1} Ck + Ck1
entonces
Ck =

= 0
= 0, k = 1, 2, 3, ...

1
Ck1 , k = 1, 2, 3, ...
(k + r) (k + r + 2) + 1

para r = 1, se tiene la siguiente ley de recurrencia:


Ck
k
k
k

1
1
Ck = 2 Ck1 , k = 1, 2, 3, ...
(k 1) (k + 1) + 1
k
1
1
= 1 C1 = 2 C0 C1 =
C0
1
(1!)2
1
1
1
= 2 C2 = 2 C1 = 2 2 C0 C2 =
C0
2
2 1
(2!)2

!
1
1
1
1
= 3 C3 = 2 C2 = 2
C0 C3 =
C0
3
3
(2!)2
(3!)2
=

..
.
Cn

= (1)

2 C0

(k!)

y la solucin es
1

y1 (x) = x

n
X
(1)

n=0

(n!)2

x =

n
X
(1)

n=0

(n!)2

xn1

Otra forma, adems de la frmula de Abel para obtener la segunda solucin, es reemplazar la forma conocida
de la segunda solucin en la ecuacin y determinar los trminos de la segunda serie, es decir, reemplazando:

1X
b0 X
n
y2 (x) = y1 (x) ln (x) +
bn x = y1 (x) ln (x) +
bn xn1
+
x n=0
x n=1

y20 (x) = y10 (x) ln (x) +


y200 (x) = y100 (x) ln (x) +

y1 (x)
b0 X
(n 1) bn xn2
2+
x
x
n=2

2y10 (x) y1 (x) 2b0 X


+ 3 +
(n 1) (n 2) bn xn3

x
x2
x
n=3

221

se tiene:
)

2y10 (x) y1 (x) 2b0 X


n3
x
+ ... =
(x) ln (x) +
+ 3 +
(n 1) (n 2) bn x

x
x2
x
n=3
(
)
(
)

X
X
(x)
y
b
b
1
0
0
(n 1) bn xn2 + (1 + x) y1 (x) ln (x) +
bn xn1
+3x y10 (x) ln (x) +
= 0
2+
+
x
x
x
n=2
n=1
2

y100

x2 y100 (x) ln (x) + 2xy10 (x) y1 (x) +


... + 3x

n=2

x2 y100

(x) ln (x) +

2xy10

... +

X
2b0
3b0
(n 1) (n 2) bn xn3 + 3xy10 (x) ln (x) + 3y1 (x)
+ x2
+ ... =
x
x
n=3

(n 1) bn xn2 + y1 (x) ln (x) +

X
b0 X
bn xn1 + xy1 (x) ln (x) + b0 + x
bn xn1
+
x n=1
n=1

= 0

2b0 X
3b0
(x) y1 (x) +
(n 1) (n 2) bn xn1 + 3xy10 (x) ln (x) + 3y1 (x)
+
+ ... =
x
x
n=3

n=2

3 (n 1) bn xn1 + y1 (x) ln (x) +

X
b0 X
bn xn1 + xy1 (x) ln (x) + b0 +
bn xn
+
x n=1
n=1

= 0

agrupando:
2b0 3b0 b0

+
+ ... = 0
x
x
x

X
X
X
X
(n 1) (n 2) bn xn1 +
3 (n 1) bn xn1 +
bn xn1 + b0 +
bn xn = 0
... +

x2 y100 (x) ln (x) + 3xy10 (x) ln (x) + (1 + x) y1 (x) ln (x) + 2xy10 (x) y1 (x) + 3y1 (x) +
n=3

2xy10 (x) + 2y1 (x) + b0 +

n=2

n=3

b0 +

n=3

(n 1) (n 2) bn xn1 +

(n 1) (n 2) bn xn1 +

n=2

bn xn1 +

n=1

b0 +

n=3

(n 1) (n 2) bn xn1 +

n=2

3 (n 1) bn xn1 +

bn xn1 +

n=1

n=1

bn xn

n=1

3 (n 1) bn xn1 + ... =

... +

n=1

bn xn

n=1

= 2xy10 (x) 2y1 (x)

3 (n 1) bn xn1 + ... =

n=2

... +

n=1

bn xn1 +

n=1

222

bn xn

X (1) (n 1)
1
+
xn2
x2 n=2
(n!)2
)
(

1 X (1)n n1
= ... 2
x
+
x n=1 (n!)2
= 2x

= 0

b0 +

n=3

(n 1) (n 2) bn xn1 +

3 (n 1) bn xn1 + ...

n=2

bn xn1 +

bn xn

3 (n 1) bn xn1 + ...

n=1

b0 +

n=3

(n 1) (n 2) bn xn1 +

n=2

n=1

n1

bn x

n=1

bn x

n=1

2 X (1)
2 (n 1) n2 2 X 2 (1)
x
+
+
x n=2
x n=1 (n!)2
(n!)2
n+1

X
(1)n+1 2 (n 1)
2

(n!)

n=2

n2

n+1

X
2 (1)n+1

(n!)

n=1

xn1

xn1

homogeneizando potencias:
b0 +

n=3

... +

bn xn

| {z }
k=n
n=1k=1

k=2

k (k 1) bk+1 xk +

3kbk+1 xk +

k=1

k=2

bk+1 xk +

k=1

iniciando del mismo ndice:


b0 + (4b2 + b1 ) x +

xn2 +

(n!)2
n=2
|
{z
}
k =n2n=k+2
n=2k=0

n=1

b0 +

{z
}
k =n1n=k+1
n=3k=2

n+1
X
(1)
2 (n 1)

(n 1) (n 2) bn xn1 +

n=2

3 (n 1) bn xn1

| {z }
k =n1n=k+1
n=1k=1

n+1
X
2 (1)

xn1
2
(n!)
n=1
|
{z
}
k =n1n=k+1
n=1k=0

bk xk =

X
(1)k+3 2 (k + 1)
2

((k + 2)!)

k=0

[k (k 1) bk+1 + 3kbk+1 + bk+1 + bk ] xk

2
(2!)

... +

(1!)

xk +

23 2
2

(3!)

X
2 (1)k+2

k
2x

((k + 1)!)
k=0

x + ...

"

k+3
X
(1)
2 (k + 1)
2

((k + 2)!)

k=2

luego
b0

4b2 + b1

k + 2k + 1 bk+1 + bk
bk+1

=
=

(2!)
2 (3 1)
2

(3!)

(1)

k+3

3!
(2!)

b2 =
2 (k + 1)

((k + 2)!)2

2 (3 1)
2

4 (3!)

b1
b1
1
=
2 22
4
(3!)

k+2

2 (1)

((k + 1)!)2

2 (1)k+2 ((k + 2) (k + 1) + 1)
2

(k + 1) ((k + 2)!)
223

+ ...

n=1

|
{z
}
k =n1n=k+1
n=2k=1

k=1

2 22 1

bn xn1

bk

2,k

(k + 1)

2, k N

2 (1)

k+2
2

((k + 1)!)

xk

luego:
k

b3

2, b3 =

3, b4 =

b4

4,

b5
k

32

(4!)
52
b1
+
(3!)2 (4!)2 (3!)2

2 (43 + 1)

5,

b6

b2
2 (43 + 1)
1
2 =
2 32
2
3
3 (4!)

2 (54 + 1)
2

42 (5!)

b1
2 22
(3!)
1

b3
2 (54 + 1)
1
2 =
2
2
4
4
42 (5!)

2 (54 + 1) (3!)2 + (52) 52


2

(4!) (5!)

2 62 5

52
2

(3!) (4!)

2 (43 + 1) 42
32

b1
2

(3!)

(4!)

b1
2

(3!)

b1

(4!)

2 62 5

2 52 4 (3!)2 + 42 3!
b4
1
b1
b5 =
2 =
2

5
5
52 (6!)2
52 (6!)2
(4!)2 (5!)2
(4!)2

3 2

6
2 62 5 (4!)2 + 2 52 4 (3!)2 + 42 3!
b1
=
+
2
2
(5!) (6!)
(5!)2

2
3 2

2
2
2
2
2
2
6

5
(4!)
+
2
5

4
(3!)
+
4

3!
6
2 7 6
2 7 6
b4
1
b1
b6 =
2 =
2
+

2
2
2
2
2
2
2
6
6
6 (7!)
6 (7!)
(5!) (6!)
(5!)

3 2 2

6 7
2 72 6 + 2 62 5 (4!)2 2 52 4 (3!)2 + 42 3!
b1
=
+
2
2
(6!) (7!)
(5!)2

..
.
luego la solucin y2 (x) sera:
y2 (x) = y1 (x) ln (x) +

3!
(2!)2

+ b1 +

1
(3!)2

2 (54 + 1) (3!)2 + (52) 52

b1

b1
(2!)2
!

x+

52
(3!)2 (4!)2

b1
(3!)2

x2 + ...

x3 + ...
2
(4!) (5!)
(4!)

3 2
2
2
2 62 5 (4!) + 2 52 4 (3!) + 42 3!
6
b1 4
x + ...
+
... +
2
2
(5!) (6!)
(5!)2
... +

1
52
2 (54 + 1) (3!)2 + (52) 52 3
1
2
x
+
x
+
x

x + ...
2
2
2
2
2
2
(2!)
(3!)
(3!) (4!)
(4!) (5!)

3 2

2
2
6
2 62 5 (4!) + 2 52 4 (3!) + 42 3!
x4 + ...
... +
2
2
(5!) (6!)

= y1 (x) (ln (x) + b1 ) +

3!

224

y la solucin general sera:

1
52
2 (54 + 1) (3!)2 + (52) 52 3
1
2
x
+
x
+
x

x + ...
2
2
2
2
2
2
(2!)
(3!)
(3!) (4!)
(4!) (5!)

3 2

6
2 62 5 (4!)2 + 2 52 4 (3!)2 + 42 3!
x4 + ...
... +
2
2
(5!) (6!)

y (x) = y1 (x) (C1 + C2 ln (x)) + C2

2.3

3!

Ejercicios

1. Determine dos soluciones linealmente independientes en forma de series de potencias para las siguientes
ecuaciones.
(a) y 00 + x2 y = 0
(b) y 00 xy 0 + 2y = 0

(c) y 00 + 2xy 0 + 2y = 0

(d) (x + 2)y 00 + xy 0 y = 0
(e) (x2 + 1)y 00 6y = 0

(f) (x2 1)y 00 + xy 0 y = 0

(g) y 00 xy 0 (x + 2)y = 0

2. Resuelva mediante series de potencias:


(a) y 00 xy 0 = 1

(b) y 00 4xy 0 4y = ex
3. Resuelva los siguientes problemas de valores iniciales empleando el mtodo de las series de potencias.
(a) (x + 1)y 00 (2 x)y 0 + y = 0 , y(0) = 2 , y 0 (0) = 1

(b) (x2 + 1)y 00 + 2xy 0 = 0 , y(0) = 20 , y 0 (0) = 1

4. Obtenga los primeros trminos de las series que son soluciones de las siguientes ecuaciones diferenciales:
(a) xy 00 + sen(x)y = 0, (Normalice la ecuacin antes de desarrollar)
(b) y 00 + ex y 0 y = 0
5. En los siguientes problemas muestre que las races indicatvas no difieren en un entero. Use el mtodo de
Frobenius para obtener dos soluciones linealmente independientes en forma de series alrededor del punto
singular regular x0 = 0 . Exprese la solucin general en (0, )
(a) 2xy 00 + 5y 0 + xy = 0
(b) 2x2 y 00 xy 0 + (x2 + 1)y = 0
(c) x2 y 00 (x 2/9)y = 0

225

(d) x2 y 00 + xy 0 + (x2 4/9)y = 0

(e) 2x2 y 00 + 3xy 0 + (2x 1)y = 0


(f) x(x 2)y 00 + y 0 2y = 0

6. En los siguientes problemas muestre que las races indicatvas difieren en un entero. Use el mtodo de
Frobenius para obtener dos soluciones linealmente independientes en forma de series alrededor del punto
singular regular x0 = 0 . Exprese la solucin general en (0, )
(a) x2 y 00 + xy 0 + (x2 1/4) = 0

(b) y 00 + x3 y 0 2y = 0
(c) x y 00 + y = 0

(d) xy 00 + xy 0 + y = 0
7. Determine la solucin de los siguientes problemas de valor inicial.
(a) x2 y 00 + 3xy 0 = 0 , y(1) = 0 , y 0 (1) = 4
(b) x2 y 00 5xy 0 + 8y = 0 , y(2) = 32 , y 0 (2) = 0
(c) x2 y 00 3xy 0 + 4y = 0 , y(1) = 5 , y 0 (1) = 3

226

Vous aimerez peut-être aussi